Contents 1 함수와 극한 1.1 수열 수열의 극한값 등비수열과 복리 선형점화식 무한급수 무한급수

Size: px
Start display at page:

Download "Contents 1 함수와 극한 1.1 수열 수열의 극한값 등비수열과 복리 선형점화식 무한급수 무한급수"

Transcription

1 Contents 함수와 극한 수열 수열의 극한값 등비수열과 복리 선형점화식 무한급수 무한급수 수렴하는 급수의 성질 수렴판정법 거듭제곱급수 5 거듭제곱급수의 수렴반경 함수의 극한과 연속성 좌극한과 우극한 극한법칙 점근선 연속함수 5 중간값정리와 최대 최소 정리 지수함수 지수함수의 정의와 성질 자연상수 연속복리 제한된 성장 5 로그함수 5 로그함수의 성질 5 자연로그 5 지수방정식 5 로그선형 6 삼각함수 6 삼각함수의 성질 6 삼각함수의 덧셈정리 6 삼각함수의 합성 i

2 ii CONTENTS 6 삼각함수의 극한값 65 함수와 미분 미분계수와 접선 평균변화율과 순간변화율 미분가능성 접선의 방정식 선형근사식 도함수와 미분법칙 도함수 함수의 연산과 미분법칙 연쇄법칙 : 합성함수의 미분법칙 연쇄법칙의 응용 5 고계도함수 평균값 정리와 극값 평균값정리 도함수와 함수의 증감 최대와 최소 : 전체에서와 부분에서 임계점 미분의 응용 최적화 문제 한계함수와 평균함수 탄력성 5 역함수정리 5 연쇄법칙의 응용 5 음함수미분법 5 역함수 정리 6 지수, 로그함수의 도함수 6 자연상수 6 지수함수의 도함수 6 로그함수의 도함수 6 로그미분법 7 함수의 그래프 7 함수의 볼록성 7 변곡점 7 그래프 그리기 7 로피탈의 법칙 8 테일러급수 8 테일러다항식 8 테일러정리 8 특수함수의 테일러급수 8 도함수와 테일러급수

3 CONTENTS iii 85 테일러급수 구하기 6 적분과 응용 부정적분 역도함수 여러 함수의 부정적분 부정적분의 성질 적분기법 치환적분법 f ()/f ()의 적분 부분적분법 부정적분의 응용: 변수분리법 변수분리법 자연성장, 자연감소 뉴턴의 냉각법칙 연속복리 5 시장가격의 결정 모형 정적분과 넓이 정적분과 넓이 정적분의 성질 정적분의 계산 두 곡선 사이의 영역의 넓이 5 y에 대한 적분 5 적분기법과 응용 5 치환적분법 5 대칭함수의 적분 5 정적분의 부분적분법 5 함수의 평균 55 연속흐름 6 특이적분 6 무한구간에서 정의된 특이적분 6 불연속함수의 특이적분 6 특이적분의 비교판정법 7 연속확률변수 7 확률밀도함수 7 누적분포함수 7 기댓값과 분산 7 정규분포 8 부피 8 부피 8 회전체의 부피 I 8 회전체의 부피 II

4 iv 행렬과 응용 행렬 행렬의 정의 행렬의 합과 상수곱 행렬의 곱 전치행렬 선형방정식계 선형방정식계 기본행연산과 기약행사다리꼴 선형계의 해집합 행렬의 활용 역행렬 역행렬 선형계와 역행렬 역행렬 구하기 가역행렬의 열벡터 5 투입산출모형 행렬식과 특성벡터 : 점화계와 행렬 특성치와 특성벡터 행렬식의 성질 점화계의 일반해 5 행렬식과 특성벡터 : 일반 5 행렬식의 정의 5 행연산과 행렬식 5 특성치와 행렬식 5 점화계의 일반해 55 사례연구: 레슬리 모형 6 마코프 모형 6 마코프연쇄 6 분포벡터 6 n 단계 추이확률 6 n 단계 추이행렬의 극한 65 극한안정분포 CONTENTS

5 Chapter 함수와 극한 수열 수열의 극한값 수를 순서대로 배열한 것을 수열(sequence)이라 부른다 이를 수학적으로 표현하면, 수열이란 자연수의 집합 N {,,, } 에서 정의된 함수이다 수열 (a, a,, an, )은 다음과 같이 간단히 나타내기도 한다 {an } n, {an } 이때 an 을 일반항이라고 한다 수열 {an }은 다음과 같이 그래프를 이용하여 나타낼 수도 있다 an n 이 그래프에서 an /n은 n이 커질수록 에 가까워진다 이때 수열의 극한값(limit of sequence)은 이라고 하고 lim an lim n n n 꼭 자연수의 집합일 필요는 없다 뒤에 나오는 거듭제곱급수나 테일러급수의 경우는 음이 아닌 정수의 집합 N {,,,, } 을 주로 사용한다

6 CHAPTER 함수와 극한 으로 표현한다 일반적으로 n이 커짐에 따라 an 의 값이 L에 가까워질 때 수열 an 은 극한값 L 을 갖는다고 하고 lim an L n 이라고 쓴다 수열의 극한값 n이 커짐에 따라 an 의 값이 L에 가까워지면 수열 {an }은 극한값 L을 갖는다고 하고 lim an L n 이라고 쓴다 극한값 limn an 이 존재하면 수열 {an }은 수렴한다(converge)고 한다 수렴하지 않는 수열은 발산한다(diverge)고 한다 수렴하지 않는 수열 {an }에 대하여 n이 커짐에 따라 an 이 한없이 커질 때 수열 {an }은 양의 무한대로 발산한다(diverges to )고 말하고 lim an n 로 나타낸다 마찬가지로 수렴하지 않는 수열 {an }에 대하여 n이 커짐에 따라 an 이 한없이 작아질 때 수열 {an }은 음의 무한대로 발산한다(diverges to )고 하고 lim an n 로 나타낸다 고등학교에서 배운 수렴하는 수열의 성질을 정리하면 다음과 같다 n이 아주 크면 an 과 L의 차이 an L 가 아주 작다는 것을 의미한다 엄밀하게 쓰자면, 임의의 양수 ε > 에 대하여 유한개의 an 을 제외하고는 모두 an L < ε을 만족한다는 뜻이다 즉, n n 이면 an L < ε인 자연수 n 이 존재한다 엄밀하게 말하자면 임의의 양수 M 에 대하여 n N 이면 a M 이 되는 자연수 N 이 존재하면 수열 {a }은 n n 무한대로 발산한다고 한다 성질 5는 함수의 연속성과 관련(고등학교 과정 외)

7 수열 수렴하는 수열의 연산 수렴하는 두 수열 {an }, {bn }에 대하여 lim an α, lim bn β n n 라고 하자 상수 c에 대하여 다음 관계식이 성립한다 lim (an ± bn ) α ± β β 6 이면 lim n n an α bn β lim (can ) c α n 5 실수 k 에 대하여 lim akn αk lim (an bn ) α β n 기초다지기 n lim an, lim bn 일 때 다음 극한값을 구하여라5 n n (a) lim (an + bn ) (c) lim (an + an ) (b) lim (an bn ) (d) lim n n an + n bn + n a n + bn n an bn p an + bn (f) lim (e) lim n 수렴하는 두 수열의 대소 관계를 이용하여 두 수열의 극한값의 대소 관계를 알 수 있다 다시 말해서 lim an α, lim bn β이고 모든 n에 대하여 n n an bn 이면 α β이다 이 결과를 이용하면 다음 샌드위치 정리를 얻는다 샌드위치 정리 자연수 n에 대하여 an bn cn 이고 lim an lim cn L n n 이면 lim bn L이다 n lim an 이면 Note n an an an 이므로 샌드위치 정리에 의하여 lim an 이다 n 5 (a) (b) 6 (c) (d) (e) 6 (f)

8 CHAPTER 함수와 극한 보기 lim n Solution sin n 을 구하여라 n sin n 이므로 sin n 이고 n n n lim n 이므로 샌드위치 정리에 의하여 lim n lim ( ) n n n sin n 이다 n 등비수열과 복리 등비수열 {rn }에 대하여 r < 이면 limn r n 이다 반면, r > 이면 limn r n 이다 r 인 경우, lim n lim n n 이므로 수렴한다 그러나 r 일 때, 수열은,,,,, 이 되고 이 수열은 발산한다 따라서 등비수열 {rn }이 수렴하는 r의 범위는 < r 이다 일반적인 등비수열에 대하여 다음이 성립한다 등비수열의 극한 초항이 a(6 )이고 공비가 r(6 )인 등비수열 {an }은 < r 이면 수렴하고 다른 경우는 발산한다 (, < r < lim an n a, r 이율과 실효이율 은행 연이율이 6%라고 하자 이자는 년에 한 번 복리(compound interest)로 지급된다 올해 A원을 저금하였다면 년 후에 지급되는 이자는 6A이므로 자산은 A + 6A 6A 가 된다 마찬가지로 n년 후 자산의 가치는 (6)n A 가 된다 이자를 년에 한 번이 아니라 한 달에 한 번 지급한다면 어떻게 될까? 한 달에 한 번 지급하면 이율은 월 6%/ 6/이므로 k개월 후 자산의 가치는 6 k A +

9 수열 5 가 된다 보기 연이율 9%로 억원의 주택자금 대출을 받았다 상환을 전혀 하지 않았을 때 다음의 경우 개월 후 갚아야 할 금액은 얼마인가? (a) 사분기별 복리(년에 네 번) (b) 월복리 Solution (a) 사분기별 이율이 9/ 5이고 개월은 분기에 해당한다 ( + 5) ; 9 이므로 개월 후 갚아야 할 금액은 약 억 9만원이다 (b) 월간 이율이 9/ 75이므로 ( + 75) ; 5 이고 갚아야 할 금액은 약 억 5 만원이다 월별로 이자를 지급할 때 사분기별로 이자를 지급해야 하는 경우보다 약 만원을 더 내야 한다 보기 에서 살펴본 바와 같이 동일한 이율 9%에 대하여도 이자를 어떤 식으로 지급하느냐에 따라 이자의 액수가 달라진다 이때 이율 9%를 명목이율(nominal rate of interest)이라고 하고 지급방법에 따라 달라지는 이율을 실효이율(effective rate of interest)이라고 한다 혼동의 위험이 없는 한 명목이율은 단순히 이율이라고 부르기로 한다 보기 명목연이율이 9%라고 하자 이자를 다음과 같은 방법으로 지급할 때 실효연이율을 구하여라 (a) 사분기별 복리 Solution (b) 월별 복리 (a) 년 후 자산의 가치는 ( + 5) ; 9 배가 된다 따라서 실효연이율은 9%이다 (b) 같은 방법으로 ( + 75) ; 98 이므로 실효연이율은 98%이다 Note 일반적으로 명목연이율이 r일 때 이자를 년에 m번 지급하는 경우 실효연이율 re 는 r m + re + m

10 CHAPTER 함수와 극한 6 을 풀면 된다 즉 r m re + m 이다 현재가치와 미래가치 미래에 얻게 될 자산의 가치는 자산을 얻게 되는 시점의 가치와는 달라진다 현재의 자산 은 시간이 지나면 이자가 붙어 자산의 크기가 커지기 때문이다 예를 들어, 연이율이 5%일 때 (이자를 년에 한 번만 지급한다면) 현재의 만원은 년 후 5 만원이 된다 이런 경우, 현재 만원의 년 후의 미래가치(future value)는 5만원이고 년 후 5만원의 현재가 치(present value)는 만원이라고 한다 n년 후 자산의 가치가 F 이면 r(r%)의 연복리로 이자를 지급할 때 이 자산의 현재가치 P 는 F P ( + r)n 이므로 P F F ( + r) n ( + r)n 이다 보기 오크 통에 담근 포도주를 7년 후에 병으로 옮겨 담아 시장에 판매한다 7년 후 만 원에 판매할 수 있는 포도주를 담근 후 바로 도매상에 넘긴다면 가격은 얼마로 책정해야 하는지 만원 단위까지 반올림하여 구하여라 단, 연이율은 %이고 다른 모든 부대비용은 무시하기로 한다 Solution 도매상에게 넘기는 가격을 A라고 할 때 A의 7년 후의 미래가치가 만원이 되 어야 한다 다시 말해서 A( + )7 이어야 하므로 A 7 ; 76 이다 즉, 76만원에 판매하면 된다 등비수열의 합 초항이 a a이고 공비가 r(r 6 )인 등비수열 {an }의 n항 까지의 합을 sn 이라고 하자 sn rsn a+ ar + ar + + arn ar + ar + + arn + arn 에서 sn rsn a arn sn a( rn ) r

11 수열 7 이다 등비수열의 합 등비수열 {arn }에 대하여 sn n X ak 이라고 하면 다음 식이 성립한다 k n a( r ), r 6 sn r na, r 기초다지기 (a) 다음 등비수열의 합을 구하여라6 k n X (c) k n X (d) k k (b) n k X k n k X k n k X k (e) (f) n k X k 적금과 연금 매년 초 A원씩을 적립하는 정기적금을 생각해 보자 연이율이 r(%), n번째 A원을 적립한 직후 자산의 크기를 Wn 이라고 하면 W A W A( + r) + A Wn A( + r)n + A( + r)n + + A( + r) + A 가 된다 Wn 의 합에서 순서를 거꾸로 생각하면 Wn 은 초항이 A이고 공비가 (+r)인 등비수열의 합이다 즉, Wn A A ( + r)n (( + r)n ) ( + r) r 반대로 년 후부터 매년 A원 씩을 n년 동안 받을 수 있는 연금을 지금 일시불로 받는다고 하자 A A 년 후 받을 A원의 현재가치는, 년 후 받을 A원의 현재가치는,, n년 후 +r ( + r) ( n+ () ) 6 (a) ( ( )n ) (b) ( ( )n+ ) (c) ( )n (d) ( )n+ (e) ( ( )n ) (f)

12 CHAPTER 함수와 극한 8 받을 A원의 현재가치는 Pn A 이므로 이 연금의 현재가치를 Pn 이라고 하면 ( + r)n A A A r ( + r) ( + r)n A [( + r)n + ( + r)n + + ] ( + r)n 이다 계산을 간단히 하면 Pn A ( + r)n A [ ( + r) n ] n ( + r) ( + r) r 이다 보기 5 일 년에 한 번 만원이 지급되는 연금을 퇴직 시점부터 년 단위로 번 지급한다 고 한다 퇴직 시 일시불로 지급한다면 얼마를 지급하여야 하는가? 연이율은 복리 5%로 일년에 한번 지급한다 Solution k (k,,,, 9)년 후 지급하는 만원의 현재가치는 ( + 5) k 만원이다 따라서 이 연금의 현재가치 P 는 등비수열의 합이다 P (5) + (5) + + (5) 9 초항이, 공비가 5 이므로 P (5) 5 ( (5) ) ; 85 이므로 이 연금의 현재가치는 약 억 85만원이다 선형점화식 등차수열과 등비수열은 모든 n에 대하여 각각 다음과 같은 관계식을 만족한다 n+ n + d, n+ rn 이와 같이 어떤 규칙에 의하여 정의되는 수열은 앞, 뒤 항 사이의 관계식으로 표현할 수 있다 앞의 항을 이용하여 뒤의 항이 정의되는 식을 점화식(recursive equation)이라고 하고 n+ an + b 로 주어지는 점화식을 선형점화식(linear recursive equation) 이라고 한다 a + b이고 a + b a(a + b) + b a + (a + )b a + b a(a + (a + )b) + b a + (a + a + )b

13 수열 9 이다 일반적으로 다음 식이 성립한다 n an + (an + an + + a + )b 두번째 항은 공비가 a 6 인 등비수열의 합이므로 an b b n n a + b an + a a a 이다 따라서 a < 이면 lim n n b a 이다 a 이면 수열은 발산한다 선형점화식 선형점화식 n+ an + b (a 6, b 6 )의 일반항은 n a n + b an a 이다 a < 이면 lim n n b a 이고 a 이면 수열은 발산한다 Note 선형점화식 n+ an + b에서 a > 이면 수열 {n }은 단조증가, 또는 단조감소하지 b 만 a < 이면 수열 {n }은 a 의 위, 아래로 진동하면서 움직인다7 기초다지기 다음 선형점화식으로 정의되는 수열 {n }의 수렴, 발산을 판단하고 수렴하는 경우 극한값을 구하여라8 (a) n+ n +, (c) n+ n +, (b) n+ n 5, (d) n+ n +, 보기 6 자동차 보험 시장을 A, B 두 회사가 양분하고 있다고 하자 올해 A 보험을 들었던 사람이 내년에 B 보험으로 옮길 확률은 a, 올해 B 보험을 들었던 사람이 내년에 A 보험으로 옮길 확률은 b라고 한다 올해 A, B 의 점유율이 각각 5%일 때 n년 후 A 보험의 시장 점 유율을 구하여라 또, 시간이 충분히 지났을 때 두 회사의 점유율을 근사적으로 구하여라 단, 7 증명은 8 (a) 연습문제 참고할 것 수렴, 9/ (b) 수렴, 5 (c) 발산 (d) 수렴, 8/7

14 CHAPTER 함수와 극한 a >, b > 이다 Solution n년 후 보험 A, B 의 점유율을 각각 pn, qn 이라고 하자 보험 A 가입자의 a배가 다음 해에 보험 A 에 가입하고 보험 B 가입자의 b배가 다음 해에 보험 A 에 가입하므로 pn 은 다음 점화식을 만족한다 pn+ ( a)pn + bqn qn pn 이므로 다음과 같은 일계선형점화식을 얻는다 pn+ ( a b)pn + b 올해(n )의 가입률이 p q pn 이므로 b ( a b)n + ( ( a b)n ) a+b 이고 lim pn n b a+b 이다 보기 7 배추와 같은 농작물의 경우 전년도의 가격 pn 을 기준으로 올해의 생산량(공급) qns 을 결정한다 올해의 판매량(수요) qnd 은 올해의 가격 pn 에 의하여 결정된다고 하자 수요함수와 생산함수가 모두 선형이라면 수요함수와 생산함수는 각각 다음과 같이 쓸 수 있다 qnd a bpn, qns c + dpn (b, d > ) 항상 수요와 공급이 같다고 할 때 가격을 구하여라 시간이 흐름에 따라 가격의 움직임을 설명 하여라 Solution 수요와 공급이 같으므로 qnd qns 에서 {pn }은 다음 관계식을 만족한다 pn a c d pn b b 따라서 pn p a c b+d n d a c + b b+d 이다 d < b이면 lim pn n 이고 d > b이면 {pn }은 진동하면서 발산한다 a c b+d

15 수열 연습문제 일반항이 다음처럼 주어진 수열이 수렴하면 극한값 lim an 을 구하여라 n (a) an (b) an (c) an (d) an (e) an (f) an (g) an n n 5 ( 9)n n n + n + ( )n n n + n ( )n n n + n + n n + (h) an n + n r n (i) an n+ (j) an (k) an (l) an (m) an n n + n+ n n + n n n nn 연이율 6%에 대하여 이자지급방법이 다음과 같을 때 실효연이율을 각각 구하여라 소수점 이하 두자리까지 구하여라 (a) 반기별 복리 (b) 사분기별 복리 (c) 월별 복리 년 만기의 만원짜리 국고채권의 현재가치를 문제 의 이자지급방법에 대하여 백단 위까지 구하여라 매년 임대료가 5만원인 상가를 5년간 임대하면서 임대료를 일시불로 내려고 한다 이 자지급방법이 다음과 같을 때 그 금액을 백단위까지 구하여라 단, 임대료는 매년 초에 낸다고 한다 (a) 연복리 (b) 반기별 복리 (c) 사분기별 복리 5 분양가가 P 인 아파트를 분양받은 후 아파트대금을 년 동안 6개월에 한번씩 같은 금액 P/5를 5번 불입한다고 한다 처음에 일시불로 불입하는 경우와 마지막에 일시불로 불입하 는 경우 총 금액은 분양가의 몇 %가 되는지 소수점 이하 두자리까지 구하여라 연이율은 6%이고 이자는 반기별로 지급된다고 가정한다 6 다음은 시어핀스키 삼각형이라고 불리는 도형을 만드는 과정이다 넓이가 인 삼각형 S 에 대하여 아래 그림과 같이 각 변의 중점을 연결하여 만들어진 네 개의 작은 삼각형 중에서 가운데 삼각형을 제거한다 남은 삼각형에 대하여 같은 과정을 반복한다 n단계가 지난 후의 도형을 Sn 이라고 할 때 S limn Sn 을 시어핀스키 삼각형(Sierpinski Gasket)이라고 한다

16 CHAPTER 함수와 극한 S S S (a) n단계에서 제거된 도형의 넓이를 n, Sn 의 넓이를 sn 이라고 할 때, n 과 sn 을 구하 여라 이것을 이용하여 시어핀스키 삼각형의 넓이를 구하여라 (b) 처음 삼각형 S 의 둘레의 길이가 L 일 때, n단계에서 만들어진 도형 Sn 의 둘레의 길이 Ln 을 구하여라 또, limn Ln 은 얼마인가? 7 다음은 코흐의 곡선을 만드는 과정이다 길이가 인 변을 삼등분하여 가운데 구간을 제거하고 같은 길이의 선분을 두 개 그린다 단계가 끝나고 나면 길이가 / 인 네 개의 선분이 생긴다 네 개의 선분에 대하여 같은 작업을 반복한다 각 단계에서 생긴 선분에 대하여 같은 작업을 반복한다 이 작업을 무한히 반복하여 생긴 곡선을 코흐 곡선(Koch curve)이라 한다 즉, n단계가 지난 후의 곡선을 Kn 이라고 했을 때, K limn Kn 을 코흐곡선이라고 한다 (a) Kn 과 K의 길이를 구하여라 (b) 정삼각형의 각 변에 위 과정을 반복한 결과를 코흐의 눈송이 (Koch s snowflake) 라고 한다 처음 정삼각형의 넓이를 A라고 하였을 때, 코흐 눈송이의 내부의 넓이를 구하여라9 8 지금 억원을 은행에 예치하고 년 후부터 년에 한 번 만원씩을 출금하려고 한다 이율은 연복리로 5%이다 9 코흐의 눈송이는 넓이는 유한하고 둘레의 길이는 무한대인 도형이다

17 수열 (a) n 을 n번 출금 후 은행 잔고(단위: 만원)라고 할 때 n 에 대한 점화식을 구하여라 (b) n 을 구하여라 (c) 몇 년 동안 출금이 가능한가? 9 선형점화식 n+ an + b 에 대하여 (a) 이면 모든 n에 대하여 n 임을 보여라 (b) a > 이고 > ( < )이면 모든 n에 대하여 n+ > n (n+ < n )이 성립함을 보여라 (c) a < 이고 6 보여라 b a 이면 수열 {n }은 e b a 의 위, 아래로 진동하면서 움직임을 자동차의 수요 qnd 와 공급 qns 은 각각 다음 조건을 만족한다 올해의 수요와 공급은 올해의 가격에 의하여 결정되며 선형관계이다 내년의 자동차 가격은 올해의 재고 (qns qnd )에 비례하여 감소한다 이 조건들을 식으로 나타내면 다음과 같다 qnd a bpn, qns c + dpn, pn+ pn α(qns qnd ) (b, d, α 은 모두 양의 실수) (a) {pn }이 만족하는 점화식을 구하고 일반항 pn 을 구하여라 (b) α의 값이 다음과 같을 때 pn 의 움직임을 설명하여라 i < α < 이러한 b+d ii <α< b+d b+d iii α > b+d 성질을 갖는 수열을 상수수열이라고 한다 이러한 성질을 가지면 순증가한다(순감소한다(strictly monotone increasing (strictly monotone decreasing)고 한다 b b Hint 모든 n에 대하여 ( n+ a )(n a ) < 임을 보이면 된다 수열이

18 CHAPTER 함수와 극한 무한급수 무한급수 수열 {an }의 각 항을 모두 더한 것을 a + a + a + 로 나타내고 무한급수(infinite series), 또는 단순히 급수(series)라고 부른다 급수를 간단히 다음과 같이 나타내기도 한다 X X an, 또는 an n 그렇다면 무한히 많은 항을 더한다는 것은 무엇을 의미하는가? 다음과 같이 정의된 수열 {sn }을 무한급수 한다 s a s a + a P an 의 부분합(partial sum)의 수열이라고 sn a + a + + an n X ak k 부분합의 수열 {sn }이 극한값 s로 수렴하면 무한급수는 수렴한다고 하고 s를 무한급수의 합(sum of series)이라고 부른다 즉, s lim sn lim n n n X ak k X an n a + a + + an + {sn }이 발산하면 무한급수도 발산한다고 한다 보기 무한급수 Solution X 는 수렴함을 보이고 그 합을 구하여라 n(n + ) n 부분분수 sn 수열이 k(k+) k k+ 를 이용하면 부분합은 다음과 같다 n X n X k(k + ) k k+ k k n n+ n+ {an } n 로 정의되면 sn Pn k ak 로 정의한다

19 무한급수 5 그러므로 n n n+ 이다 따라서 주어진 급수는 수렴하고 그 합은 이다 X n(n + ) n lim sn lim 기초다지기 여라 (a) 다음 무한급수가 수렴하는지 발산하는지 판단하고 수렴하는 경우 그 합을 구하 X n(n + ) n (b) X n +n n (c) X n+ n n 기하급수 무한급수 중 가장 중요한 보기는 등비급수, 또는 기하급수(geometric series)이다 초항이 a(6 )이고 공비가 r(6 )인 등비급수는 다음과 같이 정의된다 a + ar + ar + + arn + X arn n r 이면 sn na이므로 급수는 발산한다 r 6 일 때 sn n X ark k < r < 이면 a( rn ) n n r a a a lim rn r r n r 이다 r > 이거나 r 이면 rn 은 발산하므로 등비급수도 발산한다 lim sn lim 등비급수의 합 초항이 a (a 6 )이고 공비가 r인 등비급수 X arn a + ar + ar + + arn + n 는 r < 이면 수렴하고 합은 X arn n 이다 r 이면 등비급수는 발산한다 (a) 수렴, (b) 수렴, (c) 발산 a r a( rn ) 이므로 r

20 CHAPTER 함수와 극한 6 다음 급수의 합을 구하여라5 기초다지기 X ( )n (b) n n X ( )n (a) n n (c) n X n (d) n X n 수렴하는 급수의 성질 X 무한급수 an 이 s로 수렴한다고 하자 그러면 an sn sn 이므로 n lim an n lim sn lim sn n n s s 이다 따라서 lim an 이 존재하지 않거나 lim an 6 이면 n n X an 은 발산한다 이 결과를 일반 n 항판정법이라고 한다 일반항판정법 무한급수 X an 이 수렴하면 lim an 이다 n n lim an 이 존재하지 않거나 lim an 6 이면 n n X an 은 발산한다(대우) n X Note lim an 이면 an 이 수렴하는지 발산하는지 일반항판정법으로는 알 수 없다 두 n n P P 급수 n, n 은 모두 lim an 이지만 첫번째 급수는 수렴하고 두번째 급수는 발산한다 n 보기 (a) 다음 급수들이 수렴하는지 발산하는지 판단하여라 p X n + n n (b) n Solution (a) p n +n n X an (단, < b < a) an + bn n n + n n n + n + n n 이므로 n +n+n n +n+n p n + n n lim q 6 n n + n + lim 5 (a) (b) (c) (d)

21 무한급수 7 이다 일반항 판정법에 의하여 급수는 발산한다 (b) 분자, 분모를 an 으로 나누면 an an n 6 + bn + ab 이므로 주어진 급수는 발산한다 무한급수의 합은 부분합 수열의 극한으로 정의되므로 수열에 대하여 성립한 많은 성질들이 무한급수에 대하여도 성립한다 수렴하는 급수의 기본 성질 급수 P an 과 X P can c n 보기 bn 이 수렴하면 상수 c에 대하여 다음 등식이 성립한다 X an n X (an ± bn ) n X n an ± X bn n r < 일 때 다음 급수의 합을 구하여라 S + r + r + r + r + r5 + Solution an rn 이라고 하자 ( bn 으로 정의하면 주어진 급수는 S, n이 짝수 n r, n이 홀수 X (an + bn ) n 이다 P an 과 P bn 은 각각 공비가 r, r, 초항이, r인 등비급수이므로 X an n, r X bn n r r 이다 따라서 S X (an + bn ) n 이다 X an + n X bn n r + r + r r r

22 CHAPTER 함수와 극한 8 수렴판정법 이 절에서는 급수가 수렴하는지 발산하는지를 판단할 수 있게 하는 몇 가지 수렴판정법을 다루 기로 한다 nx X X an an + an n nn n 에서 유한개의 항을 더한 것은 항상 유한하므로 X an X 과 an nn n P 는 동시에 수렴하거나 동시에 발산한다 따라서 n an 의 수렴, 발산은 임의의 자연수 n 에 P 대하여 n an 의 수렴, 발산에 의하여 결정된다 비교판정법 모든 n 에 대하여 an bn 이면 부분합의 수열 sn n X an, tn n X bn k k 는 모두 증가수열이고 sn tn, n 이다 그러므로 수열 {sn }이 수렴하지 않으면, 즉, 발산하면 lim sn n 이고 lim tn lim sn n P an 이 발산하면 bn 도 발산한다 대우를 생각하면 n 이다 다시 말해서 P P bn 이 수렴하면 P an 도 수렴한다 이 결과를 비교판정법 (comparison test)이라고 한다 비교판정법 어떤 자연수 n 에 대하여 n n 일 때 an bn 이라고 하자 P P bn 이 수렴하면, 급수 an 도 수렴한다 P P an 이 발산하면, 급수 bn 도 발산한다

23 무한급수 보기 (a) 9 다음 급수가 수렴하는지 판단하여라 X n n Solution (b) P n n(n+) (a) X n + n n < 이고 n n(n + ) 이므로 비교판정법에 의하여 주어진 급수는 수렴한다 (b) P n n 은 발산하고6 모든 n 에 대하여 n n + n +n n n 이므로 비교판정법에 의하여 주어진 급수는 발산한다 비율판정법 기하급수 P arn 에서 연속한 두 항의 비율은 다음과 같다 arn an+ n r an ar 이때 r 이 보다 작으면 기하급수는 수렴하고 보다 크거나 같으면 기하급수는 발산한다 비율판정법은 이 결과를 일반화한 것으로 연속한 두 항의 비율의 극한값에 따라 급수의 수렴, 발산을 판단할 수 있다 비율판정법 급수 P X an+ an 에 대하여 lim ρ라고 하자 급수 an 은 n an n ρ < 이면 수렴한다 Note ρ 이면 급수 P n ρ > 이면 발산한다 an 은 수렴할 수도 발산할 수도 있다 예를 들어, 두 급수 X, n X n 은 모두 ρ 이지만 첫번째 급수는 발산, 두번째 급수는 수렴한다 6 연습문제

24 CHAPTER 함수와 극한 보기 5 비율판정법을 이용하여 다음 급수들의 수렴, 발산을 판단하여라 X n (a) (b) n Solution (a) an+ an n+ /(n+) n /n (b) an+ an n+ /(n+) n /n (c) an+ an (n+) n+ /(n+) n n /n X n n+ (c) n n (n+) X ( )n n n n > 이므로 급수는 발산한다 < 이므로 급수는 수렴한다 (n+) n < 이므로 급수는 수렴한다 거듭제곱급수 X 기하급수 n 은 < 일 때 수렴하고 나머지 경우는 발산한다 이처럼 의 거듭제곱 형태로 쓰여진 급수를 거듭제곱급수라고 한다 다시 말해서, 다음과 같은 형태로 쓰여진 급수를 a 에서의 거듭제곱급수(power series)라고 한다 X cn ( a)n c + c ( a) + c ( a) + c ( a) + n 여기서 는 변수이고 cn 은 ( a)n 의 계수(coefficient)라고 한다 거듭제곱급수에서 에 실수값을 대입하면 실수의 급수를 얻는다 따라서 거듭제곱급수는 어떤 에 대하여는 수렴하고 어떤 에 대하여는 발산한다 급수가 수렴하는 의 집합을 E라고 하면 f () c + c ( a) + c ( a) + c ( a) + 는 E에서 정의된 함수이다 무한급수 수의 표현은 유일하다 다시 말해서, X P n n cn ( a) n cn ( a) n X 가 a < r 에서 수렴한다면 무한급 bn ( a)n, a < r n 이면 모든 n,,, 에 대하여 c n bn 이다 기하급수 기하급수 X n n + 는 모든 계수가 인 거듭제곱급수이다 n 기하급수는 < 일 때 나타내기로 한다 로 수렴하고 이면 발산하는데 이 사실을 다음과 같이 n +, < ()

25 무한급수 여기서 거듭제곱급수의 부분합은 다항식 Pn () n 이 되므로 왼쪽 식 은 < 일 때 다항식 Pn ()의 극한을 의미한다 다음 그림은 n, 일 때와(왼쪽 그림), n, 일 때(오른쪽 그림) Pn ()의 그래프를 y 의 그래프(실선)와 같이 그린 것이다 y y n n n n - - 보기 6 다음 거듭제곱급수는 어떤 함수로 수렴하는지 구하고 거듭제곱급수가 이 함수로 수렴하는 의 범위를 구하여라 + ( + ) + ( + ) + + n ( + )n + Solution 이 거듭제곱급수는 초항이 이고 공비가 이면, 즉, < < 이면 + 인 등비급수이다 따라서 + < + ( + ) + ( + ) + + n ( + )n + + 이다 보기 7 다음 함수들을 에서의 거듭제곱급수로 나타내고 거듭제곱급수가 이 함수로 수렴하는 의 범위를 구하여라 (a) + Solution (b) (c) 세 경우 모두 기하급수 를 이용하여 나타낼 수 있다

26 CHAPTER 함수와 극한 (a) 기하급수에서 를 으로 바꾸어 쓰면 + X ( )n ( ) n X ( )n n n 이다 이 급수는 < 일 때, 즉, < 일 때 수렴하고 이면 발산한다 (b) 기하급수 꼴로 나타내면 i h / X n n+ n 이 된다 / < 이면, 즉, < 이면 급수는 수렴하고 이면 급수는 발산한다 (c) + + ( ) + ( ) + 이므로 ( + + ( ) + ( ) + ) 이다 < 일 때 수렴하고 이면 발산한다 5 거듭제곱급수의 수렴반경 거듭제곱급수 X cn ( a)n 에서 극한값 ρ lim n cn+ [, ] cn 이 존재한다고 하자 lim n cn+ ( a)n+ cn ( a)n cn+ a cn ρ a lim n 이므로 비율판정법으로부터 ρ a < 이면 수렴하고 ρ a > 이면 발산한다 는 것을 알 수 있다 따라서 P cn ( a)n 는 ρ 이면 모든 에 대하여 수렴한다 ρ 이면 a에서만 수렴한다 < /ρ < 일 때 a < /ρ이면 수렴하고 a > /ρ이면 발산한다 여기서 R /ρ를 수렴반경(radius of convergence)이라고 한다

27 무한급수 표기의 편의상 /, / 으로 나타내면 다음 결과가 성립한다 수렴반경 거듭제곱급수 P cn ( a)n 에서 극한값 cn+ [, ] cn P 이 존재한다고 하자 이때 거듭제곱급수 cn ( a)n 의 수렴반경은 R /ρ이다 ρ lim n 수렴반경이 R일 때, a R이면 거듭제곱급수는 수렴할 수도 발산할 수도 있다 Note 보기 8 (a) 다음 거듭제곱급수의 수렴반경을 구하여라 X n n n n Solution (b) (a) cn n n 이라고 X n n n (c) X n n n 하면 n cn+ cn n+ 이다 따라서 수렴반경은 이다 다시 말해서, < 이면 급수는 수렴하고 > 이면 급수는 발산한다 (b) cn n 이라고 하면 cn+ n cn (n + ) n+ 이므로 수렴반경은 이다 따라서 거듭제곱급수는 모든 에 대하여 수렴한다 n 은 존재하지 않는다 an n 에 대하여 비율 (c) 홀수차수 항의 계수가 이므로 limn c cn+ n 판정법을 직접 사용하면 (n+) /n+ n /n 이므로 < 이면 급수는 수렴하고 > 이면 급수는 발산한다 즉, 수렴반경은 ρ 이다

28 CHAPTER 함수와 극한 연습문제 다음 급수가 수렴하는지 발산하는지 판정하고 수렴하는 경우 그 합을 구하여라 (a) X sin n (c) n n + n n (d) n (b) X X n + n 5n n X + n n (e) (f) X cos nπ n n X n n 동전을 던져서 앞면이 먼저 나오는 사람이 이기는 게임을 한다고 하자 (a) 두 사람이 게임을 할 때 첫번째 던진 사람이 이길 확률을 구하여라 (b) 세 사람이 게임을 할 때 첫번째 던진 사람이 이길 확률은 세번째 던진 사람이 이길 확률의 몇 배인가? (c) n 사람이 게임을 할 때 첫번째 던진 사람이 이길 확률은 마지막 던진 사람이 이길 확률의 몇 배인가? 무한급수 X n 이 수렴함을 보이고 이 급수의 합을 구하여라7 (n + ) n X 는 발산함을 보여라( ) n n 조화급수(harmonic series) 5 다음 급수가 수렴하는지 발산하는지 판정하고 그 이유를 설명하여라 (a) (b) X +n+ n n (d) X n n (e) X n n X n X n + (c) n + n (g) n n n + X n (f) n n X (h) X n (i) n n+ n n n + X nn (n) n 6 다음 명제가 참이면 증명을, 거짓이면 반례를 들어라 P P P (a) 두 무한급수 an 과 bn 이 발산하면 (an + bn )은 발산한다 P P (b) 무한급수 an 이 수렴하면 무한급수 an 은 발산한다 P P P (c) 무한급수 an 은 수렴, bn 은 발산하면 무한급수 (an + bn )는 발산한다 7 다음 거듭제곱급수의 수렴반경을 구하여라 7 Hint n (n+) n+ (n+) n (n+)

29 무한급수 5 X n (a) nn n (b) (d) X n n + n (e) X ()n (c) n n (f) X ( ) X n+ (g) n + n n n X X ( )n (h) (n) n ( + )n n n X n( )n n 8 에서 다음 함수를 거듭제곱급수로 나타내고 수렴반경을 구하여라 (a) f () + (c) f () (e) f () + (b) f () (d) f () + (f) f () + 9 거듭제곱급수함수 f () P an n 가 우함수 즉, f ( ) f ()이면 모든 n 에 대하여 an+ 임을 보여라 또한 f 가 기함수, 즉, f ( ) f ()이면 모든 n 에 대하여 an 임을 보여라 다음 거듭제곱급수가 어떤 함수로 수렴하는지 구하고 수렴반경을 구하여라 (a) X ( )n n n (b) X ( )n n

30 CHAPTER 함수와 극한 6 함수의 극한과 연속성 함수 f ()에서 가 a와 다른 값을 가지면서 a에 한없이 가까워질 때 f ()의 값이 일정한 값 L 에 한없이 가까워지면 a일 때 f ()는 L에 수렴한다고 하고 lim f () L 또는 a 일 때 f () L a 로 나타낸다 함수가 a에서 정의되지 않아도 a에서 함수의 극한값은 정의될 수도 있다 예를 들어 f () 는 6 에서 정의되며 Note lim ( + ) lim 이다 Note 극한값의 정의로부터 a에서의 함수의 극한값은 함숫값 f (a)와는 상관 없다 예를 들어, 함수 f ()가 다음과 같이 정의된 경우에도 에서 fi () (i, )의 극한값은 모두 이다, 6, 6 (b) f () (a) f (),, y f () y f () y f () 좌극한과 우극한 가 a보다 크면서(또는, a의 오른쪽에서) a로 갈 때 a+ 로 나타낸다 a보다 작으면서(또는, a의 왼쪽에서) a로 갈 때는 a 로 나타내기로 한다 다음과 같이 정의된 헤비사이드8 함수(Heaviside function)9 의 t 에서 극한값을 생각해 보자 (, < H(), 8 Olive 9 시간 Heaviside(85 95), 영국의 전기공학자 에서 전원을 올렸을 때 전류의 흐름을 나타내는 함수이다

31 함수의 극한과 연속성 7 y y - 이 경우 + 일 때 H()는 항상 에 가까워지지만 t 이면 항상 에 가까워진다 그 러므로 가 에 가까워질 때 H()가 가까워지는 단 하나의 수는 존재하지 않는다 다시 말해서 lim H() 는 존재하지 않는다 그러나 + 이면 H() 이고 이면 H() 이다 lim H(), + lim H() 여기서 은 에서의 우극한값(right-hand limit), 은 에서의 좌극한값(left-hand limit) 이라고 한다 일반적인 경우의 정의는 다음과 같다 좌극한과 우극한 가 a의 오른쪽에서(또는 왼쪽에서) a에 가까워질 때 f ()의 값이 L에 한없이 가까워지면 L을 a에서 f ()의 우극한(또는 좌극한)이라고 하고 lim f () L (또는 lim f () L) a a+ 이라고 쓴다 우극한, 또는 좌극한은 존재할 때도 있고 존재하지 않을 때도 있다 또한 두 극한이 모두 존재하는 경우, 두 극한이 다를 수도 있고 같을 수도 있다 두 극한이 존재하면서 일치하면 극한값이 존재하며 이 경우 일치하는 값이 극한값이 된다 극한값의 존재 조건 a에서 f ()의 극한값이 L이 될 필요충분조건은 두 극한값 lim f ()와 lim f ()가 존 a+ 재하고 lim f () lim f () L a+ 인 것이다 실제로는 정확히 이고 a a

32 CHAPTER 함수와 극한 8 보기 함수 y f ()의 그래프가 다음과 같을 때 a에서 우극한값 lim a+ f (), 좌극 한값 lim a f ()을 구하여라 a에서 극한값이 존재하지 않으면 그 이유를 설명하여라 - (a) a (b) a (c) a (d) a Solution (a) lim+ f (), lim f () 으로 우극한의 값과 좌극한의 값이 다르다 따라서 에서의 극한값 lim f ()는 존재하지 않는다 (b) lim+ f () lim f () f ()이므로 에서의 극한값은 lim f () ( f ()) 이다 (c) lim+ f () lim f () 이므로 lim f () (6 f ())이다 (d) lim f (), + lim f () 이므로 에서의 극한값 lim f ()는 존재하지 않는다 열린 구간 (a, b)에서 정의된 함수 y f ()에 대하여 양 끝점 a, b에서의 극한값은 우극한값 과 좌극한값으로 각각 정의한다 다시 말해서 lim f () L, a+ lim f () L b 이면 f 는 a에서 극한값 L, b에서 극한값 L 를 갖는다고 한다 극한법칙 가 무한히 커질 때 ( ) f ()의 값이 일정한 값 L에 한없이 가까워지면 일 때 f ()는 L에 수렴한다고 하고 lim f () L 로 나타낸다 같은 방법으로 lim f () L 도 정의한다

33 함수의 극한과 연속성 9 함수의 극한은 사칙연산과 그 순서를 바꿀 수 있다 다시 말해서 다음과 같은 성질이 성립 하며 이 성질들을 이용하여 여러 가지 함수들의 극한값을 구할 수 있다 극한법칙 극한값 lim f (), lim g()( a ) 가 존재하면 다음이 성립한다 a a lim [f () ± g()] lim f () ± lim g() a a a lim [cf ()] c lim f (), c는 상수 a a lim [f () g()] lim f () lim g() a a a lim f () f () a a g() lim g() lim g() 6 이면 lim a a 기초다지기 다음 값을 이용하여 주어진 극한값을 구하여라 lim g() 8, lim f (), (a) lim [f () + g()] f () (c) lim g() + h() (b) lim [(f ()) g()] (d) lim 보기 유리함수 f () Solution f () + g() h() + lim h() p (e) lim [ g()] (f) lim f ()h() 7 + g() + 에 대하여 lim f ()와 lim f ()를 구하여라 분자 분모를 로 나누면 + lim f () lim, + lim f () lim 이다 y f () + 이므로 y f ()의 그래프는 y 만큼 평행이동하면 얻어진다 (a) (b) 8 (c) (d) 의 그래프를 축 양의 방향으로 만큼, y축 양의 방향으로 (e) (f)

34 CHAPTER 함수와 극한 5 y Note 분수함수(유리함수)의 무한대에서의 극한값은 분모의 최대차수로 분자, 분모를 나누면 구할 수 있다 보기 다음 극한값을 구하여라 (a) lim p Solution (a) + (b) lim p 를 곱해주고 나누어 주면 p + ( + )( + + ) 이다 따라서 극한값은 다음과 같다 lim p + lim ++ (b) t 로 치환하면 lim p p + + lim t + t t 가 되고 (a)의 결과로부터 주어진 극한값은 이다 샌드위치 정리 a 근방의 에 대하여 f () g()이고 a에서 f 와 g의 극한값이 존재한다고 하자 그러면 다음 부등식이 성립한다 lim f () lim g() a a 다음 결과는 주어진 함수가 다른 두 함수 사이의 값을 갖는 경우를 다루기 때문에 샌드위치 정리(Sandwich Theorem), 또는, 압축 정리(Squeeze Theorem) 라고 부른다

35 함수의 극한과 연속성 샌드위치 정리 a 근방의 에 대하여 f () g() h()이고 lim f () lim h() L a a 이라고 하자 그러면 극한값 lim g()가 존재하고 다음 식이 성립한다 a lim g() L a Note 만약 lim f () 이면 f () f () f () 이므로 샌드위치 정리에서 a lim f () a 임을 알 수 있다 점근선 보기 에서 f () + 의 그래프는 가 커지면 (, 또는 ) 직선 y 의 그래프와 아주 가까워진다 또한 y 의 값이 커지면 (y 또는 y ) 직선 의 그래프와 아주 가까워진다 이렇게, 또는 y 의 값이 무한대로 갈 때 어떤 직선에 무한히 가까워지면 그 직선을 주어진 함수의 점근선(asymptote)이라고 한다 특히, 점근선이 -축에 평행하면 (y a) 수평점근 선(horizontal asymptote), y-축에 평행하면 ( b) 수직점근선(vertical asymptote) 이라고 한다 보기 에서 은 수직점근선, y 는 수평점근선이다 일반적인 점근선의 정의는 다음과 같다 점근선 함수 y f ()에 대하여 y a + b가 점근선이면 다음 식이 성립한다 lim [f () (a + b)], 또는 lim [f () (a + b)] Note 일반적으로 수평점근선은 의 양의 무한대와 음의 무한대에서 극한값을 구하면 찾을 수 있고 수직점근선은 함수가 양의 무한대, 또는 음의 무한대로 발산하는 의 값을 구하면 찾을 수 있다

36 CHAPTER 함수와 극한 보기 (a) y a + b a d (c 6 )은 수평점근선 y 와 수직점근선 를 갖는다 c + d c c (b) 보기 의 결과로부터 lim ( 이므로 함수 y p p lim ( + + ) + ), + 는 두 개의 점근선 y 와 y 를 갖는다 y 연속함수 a에서 함수의 극한값과 함수값이 같으면, 다시 말해서 다음 성질을 만족하면 f 는 a 에서 연속(continuous)이라고 한다 lim f () f (a) f a Note lim a () 식 ()는 다음과 같이 쓸 수도 있다 lim f (a + h) f (a) h 보기 5 다음 함수가 에서 연속이 되도록 상수 c의 값을 정하여라 +, f () c, > + Solution 에서 우극한이 f ()이 되도록 c를 정해야 한다 lim+ f () lim+ c c + 이므로 c f () + 에서 c 이면 f 는 에서 연속이다

37 함수의 극한과 연속성 y 함수 f 가 a에서 연속이 아닐 때, f 는 a에서 불연속(discontinuous)이라고 하고 a를 f 의 불연속점(discontinuity)이라고 한다 함수가 정의역의 모든 점에서 연속이면 연속함수 (continuous function)라고 한다 연속함수 lim f () f (a) f a lim 이면 함수 f ()는 a에서 연속이라고 한다 a 함수가 정의역의 모든 점에서 연속이면 연속함수라고 한다 함수의 연산과 연속성 복잡한 함수의 연속성은 함수의 연산에 대한 극한법칙으로부터 확인할 수 있다 예를 들어, 두 함수 f 와 g가 a에서 연속이면 lim f () f (a), a lim g() g(a) a 이다 따라서 극한성질에 의하면 lim [f () + g()] a lim f () + lim g() a a f (a) + g(a) (f + g)(a) 이다 따라서 함수 f + g는 a에서 연속이다 같은 방법으로 다음 사실을 보일 수 있다 함수의 연산의 연속성 두 함수 f 와 g가 a에서 연속이면 다음 함수들도 a에서 연속이다 f ± g, c f (c 는 상수 ), f g, f /g (단, g(a) 6 인 경우)

38 CHAPTER 함수와 극한 Note 결과적으로 모든 다항함수와 유리함수는 정의역에서(즉, 분모가 이 아닌 점에서) 연 속이다 Note 보기 5처럼 함수가 두 개 이상의 구간에서 다르게 정의되면 조각적으로(piecewise) 정의되었다고 한다 조각적으로 정의된 함수가 각 구간에서 연속이면 조각적으로 연속 (piecewise continuous)이라고 한다 조각적으로 연속인 함수는 각 구간의 끝점에서 연속이면 연속 함수가 된다 보기 5에서 f 는 조각적으로 연속인 함수이고 c 이면 f 는 연속함수이다 합성함수는 주어진 함수들에서 새로운 함수를 만들어내는 또 하나의 방법이다 a 근방에서 정의된 함수 g와 b g(a)근방에서 정의된 함수 f 에 대하여 합성함수 f g는 다음과 같이 정의한다 (f g)() f (g()) 보기 6 f (), g() h() 이라고 하면, (f g)(), (f h)(), (g h)() p (h f )() 이다 g가 a에서 연속이고 f 가 b g(a)에서 연속이라고 하자 그러면 가 a로 갈 때 g()는 b에 가까워지고(g가 a에서 연속이므로) g()가 b로 가면 f (g())는 f (b) f (g(a))에 가까워진다(f 가 b에서 연속이므로) lim f (g()) f (g(a)) a 다시 말해서 f g는 a에서 연속이다 합성함수의 연속성 g가 a에서 연속이고 f 가 b g(a)에서 연속이면 f g는 a에서 연속이다 Note 그 이외에도 뒤에 나오는 삼각함수(사인, 코사인, 탄젠트 등), 지수함수, 로그함수 등은 연속함수이다 이러한 함수들의 연산, 또는 합성으로 만들어지는 함수들도 역시 연속함수이다 예를 들어, 다음 함수들은 모두 연속함수이다 p f (), f () tan, f () e, f () log( ) 5 중간값정리와 최대 최소 정리 연속함수 y f ()의 그래프를 그려 보면 끊어짐이나 구멍없이 연결되어 있다 이러한 관찰에 의하여 다음 사실을 알 수 있다

39 함수의 극한과 연속성 5 중간값 정리 함수 f 가 닫힌 구간 [a, b]에서 연속이고 f (a) 6 f (b)이면 f (a)와 f (b)사이의 임의의 값 r 에 대하여 f (c) r 인 점 c가 구간 (a, b)에 적어도 하나 존재한다 중간값 정리를 기하학적으로 보면 r이 f (a)와 f (b)사이의 값이라면 직선 y r과 y f () 의 그래프는 적어도 한 점에서 만나는 것을 의미한다 y y f HL f HbL r f HaL a c b 이러한 사실을 이용하면 중간값 정리를 이용하여 방정식의 해가 존재하는 구간을 구할 수 있다 : 만약 연속함수 f 에 대하여 f (a) f (b) < 이면 f (a)와 f (b)의 부호가 다르므로 f (c) 인 c 가 구간 (a, b)에 적어도 하나 존재한다 방정식의 해 연속함수 f 에 대하여 f (a) f (b) < 이면 방정식 f () 의 해가 a와 b 사이에 적어도 하나 존재한다 보기 7 다음 방정식에 대하여 구간 (, )에 적어도 하나의 해가 존재함을 보여라 Solution f () 이라고 하면 f 는 연속함수이고 f () >, f () < 이므로 중간값정리에 의하여 과 사이에 f (c)

40 CHAPTER 함수와 극한 6 이 되는 c가 존재한다 즉, 주어진 방정식의 해가 과 사이에 적어도 하나 존재한다 함수를 다루는 데에 있어 가장 중요한 주제 중의 하나는 최댓값과 최솟값을 구하는 것이다 정의역 D에서 정의된 함수 y f ()는 모든 D에 대하여 f () f (c) 이면 c에서 최솟값(minimum value)을 갖는다고 한다 마찬가지로 모든 D에 대하여 f () f (c) 이면 c에서 최댓값(maimum value)을 갖는다고 한다 최댓값, 최솟값을 구하는 기본적 인 방법은 미분계수를 사용하며 다음 장에서 다루기로 하고 연속함수에 대한 다음 결과로 이 절을 마무리하기로 한다 최대 최소 정리 함수 y f ()가 닫힌구간 [a, b]에서 연속이면 이 구간에서 반드시 최댓값과 최솟값을 갖는다 보기 8 최대 최소 정리는 닫힌구간이 아니거나 닫힌 구간에서 연속이 아닌 경우 성립하지 않을 수 있다 예를 들어, 구간 (, )에서 함수 y /는 연속이지만 최댓값, 최솟값 모두 존재하지 않는다 또한 (, 6 f (), 는 구간 [, ]에서 최댓값을 갖지만 최솟값은 갖지 않는다 y y - 일반적인 경우의 최댓값, 최솟값을 구하는 문제는 장에서 다루기로 한다

41 함수의 극한과 연속성 7 연습문제 다음 극한값을 구하여라 (a) lim + (b) lim p (c) lim ( )/ + 9 다음 극한값을 구하여라 극한값이 존재하지 않는 경우는 ND 로 나타내어라 (b) lim + + (c) lim + s s s s + (e) lim + 5 (f) lim z z z +z (a) lim (d) lim 다음 극한값이 존재하면 구하여라 양의 무한대로 발산하면, 음의 무한대로 발산하면 로 나타내어라 6 6 (e) lim (f) lim lim (g) lim lim lim (h) lim + + f () 의 수평점근선을 구하여라( ) (a) lim (b) (c) (d) 함수 5 다음 극한값을 구하여라 ( + ) (b) lim ( + ) + (d) lim + (a) lim+ (c) lim + 6 다음 극한값을 구하여라 p (a) lim [ + ] (b) 7 연습문제 6의 결과를 이용하여 곡선 y 8 다음 극한값을 구하여라 p lim [ + + ] + 의 점근선을 구하여라

42 CHAPTER 함수와 극한 8 p p (a) lim [ + ] 9 극한값 lim t (b) p p lim [ + ] + ct, c 6 을 구하여라( ) t 다음 등식을 만족하는 상수 a, b를 구하여라 + a + (b + ) + b a + b (b) lim a +b (c) lim f () ( )일 때, lim f ()을 구하여라 (a) lim 다음 함수가 연속함수가 되도록 상수 a를 정하여라 ( (a) f () ( +, a, < (b) f () a +, a, < 함수 f ()는 임의의 실수, y에 대하여 다음 등식을 만족한다고 한다 f ( + y) f () + f (y) (a) f ()을 구하여라 (b) f ()는 에서 연속이면 모든 점에서 연속이 됨을 보여라 중간값정리를 이용하여 다음 방정식의 해가 주어진 구간에 존재함을 보여라 (a) 5 +, [, ] (b) +, 5 닫힌 구간 [, ]에서 연속인 함수 y f ()에 대하여 < f () <, < f () < 이면 f (c) c인 c [, ]가 존재함을 보여라 Hint Hint limh f ( + h) f ()임을 보인다 g() f () 라고 하고 중간값정리를 적용한다 [, ]

43 지수함수 9 지수함수 지수함수의 정의와 성질 양의 실수 a에 대하여 실수 집합에서 정의된 함수 f () a 을 지수함수(eponential function)라고 한다 자연수 n에 대하여 an ( a a{z a} a를 n번 곱한 것) 으로 정의하고 a n an 으로 정의한다 a 로 놓는다 가 유리수이면 서로 소인 정수 p, q에 대하여 있고 p 로쓸수 q a ( q a)p 으로 정의한다 그렇다면 무리수 에 대하여 a 는 어떤 의미일까? 이 경우, a 의 정의는 유리 수의 경우처럼 직접적이지는 않다 로 수렴하는 임의의 유리수 수열 {n }에 대하여 a lim an n 으로 정의한다5 지수함수는 다음과 같이 지수의 성질을 만족한다, y가 유리수인 경우는 지수의 성질에서 바로 보일 수 있고 일반적인 실수의 경우는 수열의 극한의 성질을 이용하면 보일 수 있다 지수함수의 성질 양의 실수 a, b와 임의의 실수, y에 대하여 다음 등식이 성립한다 a ay a+y a a y ay 5 a a b b (a )y ay a b (ab) 기초다지기 q a는 다음 값을 모두 다른 소수의 지수를 이용하여 나타내어라6 q 제곱하였을 때 a가 되는 유일한 양수를 의미한다 수가 잘 정의되는지, 즉, 로 수렴하는 모든 유리수 수열에 대하여 같은 극한값으로 수렴하는지에 대한 논의는 여기서는 다루지 않는다 이러한 논의는 해석학이나 고급미적분학 교재를 참고하기 바란다 6 (a) 8 (b) (c) 7 (d) 5 (e) (f) 5 (g) (h) 이

44 CHAPTER 함수와 극한 75 7 (e) (a) 5 (c) (b) ( )5 (d) (f) (g) (h) 지수함수 y a 는 a > 이면 증가함수이고 lim a, lim a 이다 또한, < a < 이면 감소함수이고 lim a, lim a 이다 y a 의 그래프와 y < a < b일 때의 그래프이다 a a 의 그래프는 y 축에 대하여 대칭이다 다음은 y y b y b- y a- y a 인구성장 자산의 가치나 인구성장, 방사성물질의 붕괴 등 사회나 자연에서의 여러 가지 현상은 지수 함수를 이용하여 수학적으로 표현된다 실험실에서 박테리아를 키운다고 하자 함수 P P (t)를 시간 t에서 박테리아의 개수라 하자 공간과 영양을 충분하게 제공하고 관찰한 결과 박테리아의 개수는 한 시간에 두 배로 늘어 났다 같은 방식으로 박테리아의 개수가 늘어난다면 처음 박테리아의 개수가 P 일 때 P () P () P P () P () P P () P () P 이다 일반적으로 자연수 t에 대하여 P (t) t P 이고 박테리아의 수는 연속적으로7 증가한다고 가정하면 임의의 실수 t에 대하여 박테리아의 개수함수는 P t P 로 가정한다 이런 방식으로 인구가 증가할 때 인구는 지수적으로 성장 7 수학에서의 연속적인 것과는 차이가 있다 여기서는 59분 까지 변화가 없다가 6분에 두 배가 되는 것이 아니라 그 사이에 계속 증가하여 6분에는 두 배가 된다는 뜻이다

45 지수함수 (eponential growth)한다고 한다 보기 관찰을 시작할 때 마리의 박테리아가 있었다고 하자 박테리아의 수가 분 만에 두 배로 늘어난다면 두 시간 후 박테리아의 수는 몇 마리인가? 또, 5분 후 박테리아의 수는 몇 마리인가? Solution 관찰을 시작한 지 t시간 후 박테리아의 수를 P (t)라고 하면 P (), P (5) P () 이고 일반적으로 P (5t) t, 또는 P (t) t 로 쓸 수 있다 따라서 두 시간 후의 박테리아의 수는 P () 6 이다 5분 후면 5시간 후이므로 P (5) 5 ; 이다 자연상수 이율이 %(r )이고 이자를 일년에 m번 지급한다고 하자 A원을 예금하였을 때 년 후 자산의 가치를 A()이라고 하면 m A() A + m 이다 이자를 아주 자주 지급한다면, 다시 말해서, m이 커지면 년 후의 자산가치 A()은 어떻게 m 의 값을 구한 것이다 될까? 다음 표는 m,,,, 일 때 + m m m + m m 은 증가하며 점점 어떤 수로 수렴한다8 이때 그 표에서 볼 수 있듯이 m이 커지면 + m 어떤 수를 자연상수라고 부르고 e로 나타낸다9 m e lim + ; 78 m m 8 증명은 실수의 완비성을 이용하면 보일 수 있으나 이 교재의 범위를 넘으므로 생략한다 e는 원주율 π와 마찬가지로 무리수이다 자연상수 e는 68년 스코틀랜드의 수학자 네이피어(John Napier, 55 67)가 출판한 책의 로그표에 처음 자연로그의 밑수로 등장한다 지수, 로그함수의 성질을 체계적 으로 확립한 오일러가 처음 이 수를 나타내기 위하여 e를 기호로 사용하였다 9 자연상수

46 CHAPTER 함수와 극한 보기 lim + n n (a) lim n Solution n n e를 이용하여 다음 극한값을 구하여라 n (a) (b) lim n n n n n n n lim n n n lim n a n (a 6 ) n 이고 (n ) 이므로 n ) (+ n lim + n + + n n + lim n n n + n e e e 이다 (b) a > 이면 n a 이므로 lim n a n + n " + n/a + n/a lim n " 이다 a < 이면 b a라고 하자 lim n a n + n n b n/a #a ea 이므로 (a)로부터 다음 결과를 얻는다 lim n n/a #a n b n " lim lim n " n n/b lim n n/b #b n/b n/b #b b e b ea e n + e 이므로 보기 (b)의 결과를 정리하면 다음과 같다 n n lim 자연상수 e n + e n n lim 실수 R에 대하여 lim n + n e n

47 지수함수 Note 좀 더 일반적으로 자연상수를 다음과 같이 정의하기도 한다 lim + 연속복리 현실적으로는 불가능하지만 m 일 때 이자를 연속복리로 지급한다(compounded continuously)고 표현한다 따라서 연이율 %의 이자를 연속복리로 지급하면 년 후 자산의 가치는 A() Ae ; 78A 다시 말해서 년 후의 미래가치는 처음 자산의 약 78배이다 t년 후의 미래가치는 mt A(t) A lim + Aet m m 가 된다 일반적으로 연이율이 r(r%)일 때 이자를 연속복리로 지급하면 t년 후의 자산가치는 A(t) A lim m + r mt m 이고 + r mt m + h mt m/r irt + m/r m/r 에서 m 일 때 m/r 이므로 A(t) Aert 가 된다 연속복리 연이율이 r(r%)일 때 이자를 연속복리로 지급하면 현재 A의 t년 후 자산가치(미래가 치)는 A(t) Aert 이다 Note t년 후 가격이 V 인 자산의 현재가치는 V e rt 이다 현대의 금융상품 중에는 한시간, 심지어는 일 분에 따라 그 가치가 변하는 것들이 많다 이자도 하루, 또는 하 루보다도 짧은 시간에 발생하는 경우가 있는데 이런 경우 자산의 가치를 구할 때 연속복리를 이용하여 근사적으로 구한다

48 CHAPTER 함수와 극한 보기 다음 표는 연이율이 6%일 때 만원을 저금한 경우 이자지급 방법을 달리하였을 때 자산의 미래가치를 구한 것이다 년 후 년 후 년 후 연 복리 월 복리 일 복리 연속복리 일복리로 계산하였을 때 실효연이율은 68 ; 68 이므로 연 68%이다 이는 연복리로 계산한 실효연이율 6%보다 약간 높다 일복리 실효연이 율이 연복리 실효연이율보다 높은 것은 매일 발생하는 이자에 이자가 붙기 때문이다 일복리로 계산하는 것과 연속복리로 계산하는 것은 오차가 아주 작아서 일복리 대신 연속복리를 근삿값 으로 사용하기도 한다 제한된 성장 798년 영국의 경제학자 맬더스(Malthus)는 세계 인구증가에 대한 논문을 발표하였다 이 논 문에서 그는 세계 인구가 인구의 현재 크기에 비례하여 증가한다는 가정을 제시하고 이 가정 하에서 인구는 지수함수를 따라 증가한다는 것을 보였다 지수함수 y Aa (a > ) 는 일 때 아주 빠른 속도로 커진다 그러나 실제 인구를 살펴 보면, 인구가 적을 때는 빠르게 증가하다가 인구가 많아지면 증가율이 둔화하는 경향을 볼 수 있다 비슷한 경우로, 따뜻한 방에 놓인 차가운 물체는 처음에는 온도가 빠르게 상승하다가 상승속도가 점점 느려져 방 온도와 비슷한 온도에 이르면 거의 상승하지 않는다 이런 현상을 어떤 함수들로 나타낼 수 있는지 살펴 보기로 한다 보기 내부온도가 C인 냉장고에서 맥주를 꺼내고 t분이 지난 후 맥주의 온도를 f (t)라고 하면 다음과 같이 나타내어진다 f (t) e t (a) 분 후 맥주의 온도는 얼마인가? 시간 후 맥주의 온도는 얼마인가? (b) 시간이 많이 지나면 맥주의 온도는 어떻게 되는가? 이 식은 물체의 온도는 물체와 방의 온도 차이에 비례하여 변한다 는 뉴턴의 냉각법칙(cooling law of Newton)에 의하여 유도된다

49 지수함수 5 단, 방의 온도에는 변화가 없다고 가정한다 Solution y f (t)의 그래프는 다음과 같다 y 6 (a) f () e ; 599, f (6) e ; 77이다 (b) 시간이 많이 지나면(t ) lim f (t) t 이므로 맥주의 온도(f (t))는 C로 수렴한다 로지스틱함수 998년 처음 시작된 초고속 인터넷 서비스는 년 가입자 수 5만에서 서비스 시작 불 과 년 만인 년에 가입자 수가 만을 넘어섰다 년에는 가입자 수가 786만으로 대부분의 가구가 초고속 인터넷을 사용하고 있는 것으로 나타났다 인터넷 가입자 수는 초반에 약 %의 증가율을 보이며 증가하다가 어느 정도 인터넷이 보급된 이후는 증가율이 둔화되어 최근에는 거의 정지된 상태를 보이고 있다 이는 전자제품이나 인터넷과 같은 새로운 상품이 개발되었을 때 나타나는 전형적인 현상이다 이러한 현상은 로지스틱함수(logistic function)를 이용하여 나타내는 것이 보편적이다 로지스틱함수는 인구성장이나 전염병의 전파 등을 표현하는 데에도 적합한 함수임이 알려져 있 다 또한, 신기술상품의 보급율이나 수요함수 등을 나타내는 데에도 유용하다 로지스틱함수는 다음과 같이 정의된다 f () 자료 N, + Ae r 출처 : e-나라지표 r, N, A >

50 CHAPTER 함수와 극한 6 population N N A+ time 로지스틱함수의 그래프는 누운 S-자 모양으로 증가하며 lim f () N, lim f () 이므로 두 개의 수평점근선 y, yn 을 갖는다 로지스틱함수는 단순증가하므로 그래프는 항상 두 직선 y 과 y N 사이에 위치한다 N <N < + Ae r 로지스틱함수의 분자 분모에 er 를 곱하면 f () N N er + Ae r er + A 을 얻는다 가 에 가까우면 er 는 에 가까우므로 f () N er N r e +A +A 을 얻는다 다시 말해서 근방에서 f ()는 지수함수와 비슷하게 움직인다 반면 의 값이 아주 커지면 Ae r 이므로 f () N 이다 여기서 N 을 수용용량(carrying capacity)이라고 한다

51 5 로그함수 5 7 로그함수 a > 이고 a 6 일 때 지수함수 f () a 는 순증가하거나(a > 일 때), 또는, 순감소하는( < a < 일 때) 일대일대응이다 따라서 역 함수가 존재하는데 이를 밑수(base)가 a인 로그함수(logarithmic function)라고 하고 loga 로 나타낸다 다시 말해서 y loga ay 로 나타낸다 예를 들어 8이므로 log 8 이고 (5) 8이므로 log5 8 이다 5 로그함수의 성질 로그함수와 지수함수는 서로 역함수이므로 다음 식이 성립한다 지수함수와 로그함수 aloga, loga a 다음 로그함수의 성질은 지수함수의 성질로부터 쉽게 보일 수 있다 곱, 몫, 지수에 대한 로그함수의 성질 양의 실수, y, 임의의 실수 r에 대하여 다음 식이 성립한다 (곱의 법칙) loga (y) loga + loga y loga loga loga y y loga r r loga (몫의 법칙) (지수의 법칙) 기초다지기 (a) log 7 다음을 간단히 하여라 (d) log 5 (b) log (e) log/ 7 (c) log5 (f) log8 앞으로 (a) (g) log e (h) log (i) e log 특별한 언급이 없어도 로그함수의 밑 a는 항상 a >, a 6 인 것으로 간주한다 (b) 5 (c) (d) (e) (f) (g) (h) (i)

52 CHAPTER 함수와 극한 8 지수함수의 성질로부터 다음과 같은 로그함수의 성질을 알 수 있다 지수함수의 정의역은 실수의 집합 R이고 치역은 R+ 이므로 로그함수의 정의역은 R+, 치역은 R이다 a > 이면 지수함수 y a 는 순증가하므로 역함수 y loga 도 순증가한다 마찬가지로 < a < 이면 지수함수 y a 는 순감소하므로 역함수 y loga 도 순감소한다 a, a a이므로 모든 a에 대하여 loga, loga a 이다 다시 말해서 y loga 의 그래프는 두 점 (, )과 (a, )을 지난다 로그함수의 그래프 로그함수의 그래프에 대하여는 다음 사실들을 알 수 있다 로그함수 y loga 는 지수함수 y a 의 역함수이므로 두 함수의 그래프는 직선 y 에 대하여 대칭이다 지수함수 y a 는 a > 일 때 아주 빠르게 증가하는 함수이다 따라서 로그함수 y loga 는 아주 느리게 증가하는 함수이다 b > a > 인 경우 지수함수와 로그함수의 그래프를 같이 그려보면 각각 다음과 같다 y y b y a y loga HL y logb HL log a loga 이므로 y log a 의 그래프와 y loga 의 그래프는 축에 대하여 대칭이다

53 5 로그함수 9 y y loga HL y log HL a 그래프로부터 로그함수 f () loga ()에 대하여 다음 사실들을 관찰할 수 있다 a > 이면 a가 클수록 일 때, 함수는 천천히 증가한다 a > 이면 lim loga, lim+ loga 이다 < a < 이면 lim loga, lim+ loga 이다 5 자연로그 밑수가 자연상수 e이면 밑수를 생략하고 다음과 같이 나타내기로 한다 loge log 이때 log 를 자연로그함수(natural logarithm)라고 부른다5 일반적인 밑수 a(a >, a 6 )를 갖는 로그함수는 자연로그함수를 이용하여 나타낼 수 있다 y loga 이면 ay 이고 양변에 자연로그를 취해 주면 log log ay y log a 이므로 y loga log log a 가 성립한다 이를 밑의 변환공식이라고 한다 밑의 변환공식 loga 보기 5 고등학교 log log a loga b c일 때, 다음 값을 c를 이용하여 나타내어라 교재에서는 밑이 인 상용로그의 경우 밑을 생략하여 쓰고 자연로그의 경우 ln을 사용한다

54 CHAPTER 함수와 극한 5 (a) logb a log b 이다 log a 밑의 변환공식에 의하면 c Solution (c) loga b (b) log a b (a) 다시 밑의 변환공식에 의하면 다음 식이 성립한다 logb a (b) 밑의 변환공식과 log a log a log b c log a라는 사실을 이용하면 log b log b log a log a log b c log a log a b 이다 (c) loga b log b log b c 이다 log(a ) log a 기초다지기 다음 근삿값을 사용하여 주어진 값을 근사적으로 구하여라6 log ; 5, log y ;, (a) log y (c) log (b) log z (d) log 8 log z ; 9 yz (e) logyz (f) log5 y 지수방정식 5 지수함수는 일대일대응이므로 a ay y 이다 밑이 다를 때에는 양변에 자연로그를 취하여 풀어 준다 예를 들어, 6 일 때 양변에 자연로그를 취하면 log log log log 이므로 log ; 6 log 이다 기초다지기 6 (a) 6 7 (a) log log 5 다음 방정식을 풀어라7 (b) (c) 5 (d) (e) 56 (f) 7 6 (b) log (c) log (d),/ (e) log log (f) log log log

55 5 로그함수 5 (a) 5 (c) e (b) 6 (d) (e) (f) e e 보기 연이율 6%의 연속복리로 이자를 지급한다고 한다 만원을 저금하였을 때 자산가 치가 만원이 되는 것은 언제인가? Solution t년 후 지급되는 액수를 A(t)라고 하면 A(t) e6t 이다 t년 후 자산이 만원이라면 e6t e6t 6t log 이다 양변에 자연로그를 취해 주면 log(e6t ) log 가 되므로 t log 69 ; 이다 다시 말해서 자산가치가 만원이 되는 것은 년 6개월 후이다 보기 맥주가 가장 맛있는 온도는 6 C라고 한다 내부 온도가 C인 냉장고에서 꺼낸 맥주의 t분 후의 온도는 다음과 같다 f (t) e t 맥주를 가장 맛있게 마시려면 맥주를 마시기 몇 분 전에 맥주를 냉장고에서 꺼내 놓아야 하는가? Solution 맥주 마시기 t분 전에 꺼내 놓는다고 하면 e t 6 이 성립하는 t를 찾으면 된다 정리하여 양 변에 로그를 취하면 e t 8 t log 8 ; 이므로 t; 이다 다시 말해서 마시기 분 전에 꺼내 놓으면 된다

56 CHAPTER 함수와 극한 5 5 로그선형 로그함수의 성질에 의하면 곱은 합의 형태로 변환된다 이 성질을 이용하면 어떤 종류의 식은 선형함수 꼴로 바꾸어 쉽게 다룰 수 있다 예를 들어, 가격이 P 일 때 수요 Q가 다음과 같이 주어진다고 하자 Q ap k, (a >, k < ) 양변에 자연로그를 취하고 p log P, q log Q로 놓으면 q log a + kp 이 성립한다 Q q P Q ap k p q log a + kp 이렇게 양변에 로그를 취하여 선형식이 되는 경우 로그선형(log-linear)이라고 한다 보기 상품의 생산성(productivity)은 노동력(labor)과 비용(budget)에 의하여 결정 된다 일반적으로 상품의 생산성은 만들어진 상품의 수로, 노동력은 노동자의 수로 간주한다 비용은 전기세, 자동화 설비 유지비 등 인건비 이외의 다양한 경비를 의미한다 생산성분석에서 널리 사용되는 모형 중의 하나인 콥 더글라스 모형에서는 생산함수를 다음과 같이 정의한다 P Aa y a 여기서 P 는 생산되는 상품의 수, 는 노동자의 수, y는 단위시간의 비용이다 A와 a는 생산되는 상품에 의하여 결정되는 수로 A는 양의 실수, a는 보다 작은 양의 실수이다8 양변에 로그를 취하면 log P log A + a log + ( a) log y 가 된다 P 가 고정된다면 log 와 log y가 선형관계를 갖게 된다 8 이렇게 정의된 생산함수를 콥-더글러스 생산함수(Cobb-Douglas production function)라고 한다

57 5 로그함수 5 연습문제 5 f () e 의 그래프는 다음과 같다 y (a) y me 의 그래프를 m 5, 일 때 그려라 (b) y e +C의 그래프를 C, 일 때 그려라 C가 어떤 역할을 하는지 설명하여라 (c) y e( h) 의 그래프를 h, 일 때 그려라 h가 어떤 역할을 하는지 설명하여라 f () 의 그래프는 다음과 같다 + e y (a) y f ()의 점근선을 모두 구하여라 C (b) g() 의 그래프를 C 5, 일 때 y 절편과 수평점근선을 포함하여 + e 그려라 (c) h() 의 그래프를 a 5, 일 때 y 절편과 수평점근선을 포함하여 + ae 그려라 다음 방정식을 풀어서 를 log를 이용하여 나타내고 근삿값을 소수이하 자리까지 구하 여라 (a) e (b) e e (c) e 생명체가 사망하면 생명체에 있는 방사성탄소 C-은 연속적으로 붕괴되어 안정적인 탄소 C-로 바뀐다 화석에 남아 있는 C-의 양이 처음 A였다면 t 후의 C-의 양은 다음과 같이 주어진다 C(t) A(99988)t

58 CHAPTER 함수와 극한 5 (a) 몇 년이 지나야 방사성 탄소 C-의 양이 절반이 되는지 자연수로 구하여라 (b) 최근 발견된 식물 화석이 함유한 C-의 양은 5g이라고 한다 이 식물이 살아 있을 때 함유하고 있는 C-의 양이 g이라면 이 화석은 얼마 전의 것이겠는가? 천의 단위에서 반올림하여 계산하여라 (c) 최근 발견된 조개 화석이 함유한 C-의 양은 g이라고 한다 이 식물이 만년 전 의 것이라면 살아 있는 조개가 함유하고 있는 C-의 양을 소수점 이하 두 자리까지 구하여라 5 세계 인구는 8년 억이었고 년에는 6억이었다 그렇다면 인구가 억인 해는 언제였겠는가? 인구가 지수적으로 증가한다고 가정하고 구하여라 6 배양기에 마리의 박테리아가 있다 박테리아는 연속적으로 증가하여 시간 후에 두 배가 된다 박테리아의 수가 지수적으로 성장한다고 할 때, t 시간 후 박테리아의 수 P (t) 의 식을 구하여라 이 함수를 이용하여 이틀 후 박테리아의 수를 구하여라 7 년 전 양어장에 마리의 물고기를 방류하였다 올해 물고기의 수를 측정하였더니 마리였다면 내년 물고기의 수는 얼마로 추정할 수 있는지 다음 경우에 각각 구하여라 단, 측정은 모두 같은 날에 한다고 한다 (a) 지수적으로 증가할 때 (b) 수용용량이 5인 로지스틱 모형을 따를 때 (c) 수용용량이 인 로지스틱 모형을 따를 때 8 아스피린을 섭취한 후 두 시간이 지나면 체내의 아스피린 양은 반으로 준다고 한다 그 양이 같은 방식으로 감소한다면 (a) mg의 아스피린을 섭취하고 5 시간 후 아스피린의 양은 얼마인가? (b) mg의 아스피린을 섭취하고 몇 시간이 지나면 아스피린의 양이 5 mg 이하가 되겠는가? 9 혈중알콜농도는 지수적으로 감소하는데 한 시간이 지나면 /이 감소한다고 한다 음주 단속에서 현재 혈중알콜농도가 5mg/L인 사람에 대하여 (a) 분 후 다시 측정한다면 혈중알콜농도는 얼마가 되겠는가? (b) 혈중 알콜 농도가 5mg/L 이하가 되려면 얼마나 지나야 하는가? 5 년만에 원금이 두 배가 되었다면 다음의 이자지급방법에 대하여 (명목)연이율을 각각 구하여라 (a) 분기별 복리 (b) 월별 복리 (c) 연속 복리 연이율은 8%라고 할 때 문제 의 이자지급방법에 대하여 원금이 두 배가 되는 기간을 각각 구하여라

59 5 로그함수 55 데시벨(decibel)은 소리의 크기를 측정하는 단위(dB)로 다음과 같이 주어진다 D log I I 여기서 I는 소리의 강도를 나타내며 평방미터 당 왓트(W/m )로 측정된다 I W/m 은 인간이 들을 수 있는 최소의 강도를 나타낸다 (a) 다음 소리들의 데시벨을 정수로 구하여라 TV (m 거리) : 5 W/m 헤드폰의 큰 음악 : W/m 비행기 (5m 거리) : W/m (b) 9dB이 넘는 소리는 인간의 청각에 손상을 줄 수 있는 것으로 알려져 있다 (a)의 소리 중에서 어떤 것이 인간의 청각에 손상을 줄 수 있는가? (c) D db의 소리와 D db의 소리가 갖는 강도 I 과 I 사이에는 다음과 같은 관계식이 성립함을 보여라 I (D D ) I (d) 데시벨이 커지면 소리의 강도는 몇 배가 되는가? 다음 극한값을 구하여라 (a) lim + (b) lim log ( + ) f () 하여라 일 때 다음 극한값이 존재하면 구하고 존재하지 않으면 그 이유를 설명 + / (a) lim+ f () (b) lim f () (c) lim f () 5 다음 함수에 대하여 에서 좌극한과 우극한을 각각 구하여라 연속이 되도록 f () 의 값을 정할 수 있으면 정하여라 (a) f () e / (b) f () e / 6 중간값정리를 이용하여 다음 방정식의 해가 주어진 구간에 존재함을 보여라 (a), [, ] (b) log ( + ), [, ]

60 CHAPTER 함수와 극한 56 삼각함수 6 각의 크기는, 5 처럼 일반적으로 6분법을 이용하여 나타낸다 그러나 이론적으로는 호도법을 이용하여 각의 크기를 실수로 나타낸다 호도법은 각의 크기를 단위원에서 대응하는 호의 길이를 이용하여 나타내는 방법이다 단위원에서 호의 길이가 일 때 대응하는 중심각을 rad 라고 정의하고 래디안(radian)이라고 읽는다 따라서 단위원에서 호의 길이가 θ일 때 중심각의 크기는 θrad이 된다 Θ rad Θ rad 반지름이 인 반원의 호의 길이는 π이므로 8 π (rad) 이다 따라서 π 8 (rad), π (rad) 이다 특별히 혼란의 위험이 없으면 호도법의 단위 rad은 생략하고 사용한다 6 삼각함수의 성질 좌표평면에서 원점을 중심으로 하는 단위원은 다음과 같이 방정식으로 나타낼 수 있다 + y 이때 축에서 양의 방향으로9 중심각 θ(rad)에 대응하는 단위원의 좌표를 (cos θ, sin θ) 로 정의한다 9 시계반대방향을 이 의미한다 정의에 의하면 θ π 에 대하여 직각삼각형을 이용한 정의와 일치한다 cos θ 밑변, 빗변 sin θ 높이 빗변

61 6 삼각함수 57 sin Θ Θ cos Θ 정의에 의하면 점 (cos θ, sin θ)는 모든 실수 θ에 대하여 단위원 위에 있으므로 다음 식을 만족 한다 cos θ + sin θ 단위원의 중심각은 π이므로 코사인함수 cos와 사인함수 sin은 모두 주기가 π인 주기함수이다 cos(θ + kπ) cos θ, sin(θ + kπ) sin θ (k 는 정수) 삼각함수의 성질 I cos θ + sin θ cos(θ + kπ) cos θ, sin(θ + kπ) sin θ (k : 는 정수) 또한 원의 대칭성에 의하여 다음 등식들이 성립함을 알 수 있다 삼각함수의 성질 II cos( θ) cos θ, sin( θ) cos(π θ) cos θ, sin(π θ) cos(π + θ) cos θ, sin(π + θ) sin θ sin θ sin θ sin Θ sin Θ sin Θ Π-Θ Θ -Θ cos Θ sinh-θl Θ+Π Θ coshπ - ΘL Θ cos Θ coshθ + ΠL cos Θ sinhθ + ΠL

62 CHAPTER 함수와 극한 58 기초다지기 다음 값을 구하여라 (a) sin π (c) cos 7π (e) sin 7π 6 (g) cos 5π (b) sin 5π (d) cos π 6 (f) sin 8π (h) cos 7π 6 단위원 위의 점 (, y)를 양의 방향으로 π/( 9 )만큼 회전하면 ( y, )가 되므로 cos(θ + π ) sin θ, sin(θ + π ) cos θ 가 성립한다 y sin Θ Θ cos Θ -y 식 ()과 삼각함수의 성질 II의 첫번째 식을 이용하면 π θ) π sin( θ) cos( sin( θ) sin θ cos( θ) cos θ 임을 알 수 있다 삼각함수의 성질 III π + θ sin θ, sin + θ cos θ π π cos θ sin θ, sin θ cos θ cos π 탄젠트함수 tan는 tan θ (a) (b) (c) (d) (e) (f) (g) sin θ cos θ (h) ()

63 6 삼각함수 59 π π 에서만 cos θ 이므로 탄젠트함수의 정의역은 θ 6 nπ + 인 실수이다 사인과 코사인의 대한 성질을 이용하면 다음 식이 성립함을 알 수 있다 로 정의한다 θ nπ + tan( θ) tan θ tan(π θ) tan θ tan(π + θ) tan θ 마지막 식에서 코사인함수, 사인함수와는 달리 탄젠트함수의 주기는 π임을 알 수 있다 함수의 그래프를 그리는 방법은 고등학교 교재를 참고하기 바란다 기초다지기 << π 이고 sin 일 때 다음을 구하여라 5 (a) cos (c) tan (b) cos( ) (d) sin 6 π (e) sin(π ) π (f) cos (g) cos(π + ) (h) tan π 삼각함수의 덧셈정리 단위원 위의 두 점 A(, )과 B(cos(α + β), sin(α + β)) 사이의 거리는 두 점 C(cos α, sin α)와 D(cos( β), sin( β)) (cos β, sin β)사이의 거리와 같다 C B Α Β Α A -Β D 따라서 ( cos(α + β)) + sin (α + β) (cos α cos β) + (sin α + sin β) 이 성립한다 항등식 cos θ + sin θ 을 이용하여 이 식을 정리하면 cos(α + β) (cos α cos β sin α sin β) 이 되므로 cos(α + β) cos α cos β sin α sin β 을 얻는다 이 식에 β 대신 β를 대입하면 cos(α β) cos α cos β + sin α sin β (a) 5 (b) 5 (c) (d) 5 (e) 5 (f) 5 (g) 5 (h)

64 CHAPTER 함수와 극한 6 π α를 대입하면 π π cos α β cos (α + β) sin(α + β) 이 성립한다 이 식에서 α 대신 이고 cos( π α β) π π α) cos β + sin( α) sin β sin α cos β + cos α sin β cos( 에서 sin(α + β) sin α cos β + cos α sin β 를 얻는다 마지막으로 이 식의 β 대신 β를 대입하면 sin(α β) sin α cos β cos α sin β 를 얻는다 또한 탄젠트함수의 정의에 의하여 tan(α + β) sin α cos β + cos α sin β sin(α + β) cos(α + β) cos α cos β sin α sin β 이 성립한다 분자와 분모를 모두 cos α cos β로 나누면 tan(α + β) tan α + tan β tan α tan β 를 얻는다 이 식에 β 대신 β를 대입하면 tan(α β) tan α tan β + tan α tan β 을 얻는다 위에 얻은 여러가지 등식을 삼각함수의 덧셈정리라고 한다 삼각함수의 덧셈정리 보기 cos(α ± β) cos α cos β sin α sin β sin(α ± β) tan(α ± β) sin α cos β ± cos α sin β tan α ± tan β tan α tan β 다음 값을 구하여라

65 6 삼각함수 6 (a) cos 5 cos Solution π (b) sin 75 (a) 5 5 (c) tan 75 π π 이므로 6 π π π π π π cos cos + sin sin cos ; 97 cos 5 이다 (b) sin 75 sin(9 5 ) cos 5 (c) ; 97 π π + 이므로 6 tan π + tan π6 π 6 tan π tan π ; 7 tan 75 tan π + 이다 기초다지기 다음 값을 구하여라 (a) sin 5 (b) tan 5 (c) cos 75 코사인함수의 덧셈정리에서 α β θ 이면 cos(θ) cos θ cos θ sin θ sin θ cos θ sin θ cos θ sin θ 을 얻는다 같은 경우 사인함수와 탄젠트함수에 대한 관계식도 정리하면 다음과 같다 이 등식을 배각공식이라고 한다 (a) 6 (b) (c) 6

66 CHAPTER 함수와 극한 6 배각공식 cos θ sin θ cos θ sin θ sin(θ) tan(θ) sin θ cos θ tan θ tan θ cos(θ) 기초다지기 << (a) cos π 이고 sin 일 때 다음을 구하여라 5 (b) sin (c) cos (d) cos sin 5, cos 5, tan 5 을 구하여라 보기 코사인의 배각공식으로부터 Solution cos 5 sin 5 이고 sin 5 > 이므로 s sin 5 이다 p ; 8 + cos 5 sin 5 이고 cos 5 > 이므로 s cos 5 p + + ; 9 이다 tan 5 는 정의로부터 sin 5 tan 5 cos 5 을 얻는다 (a) 5 (b) s ; + 5 (c) 7 5 (d)

67 6 삼각함수 6 6 삼각함수의 합성 실수 a, b에 대하여 y f () a cos + b sin 는 주기가 π인 주기함수이다 이제 이 함수의 최댓값을 구해보기로 한다 A 하면 ha i b a cos + b sin A cos + sin A A a + b 라고 이다 a A 이므로 점 + b A a b, 는 단위원 + y 위의 점이다 따라서 A A cos α a, A sin α b A 인 α에 대하여 a b cos + sin cos α cos + sin α sin A A cos( α) 이 성립한다 그러므로 f () a cos + b sin A cos( α) 으로 쓸 수 있고 최댓값은 A a + b, 최솟값은 A이다 y f ()의 그래프는 y A cos 의 그래프를 축 양의 방향으로 α 만큼 평행이동하면 얻어진다 y Α A -A 이렇게 a cos +b sin 꼴의 함수를 A cos( α), 또는 A sin(+β)꼴로 바꾸어 쓸 수 있는데 이를 삼각함수의 합성이라고 한다

68 CHAPTER 함수와 극한 6 삼각함수의 합성 A a + b, cos α a b, sin α 이면 다음 등식이 성립한다 A A a cos + b sin A cos( α) Note 삼각함수의 합성은 사인함수를 이용하여 나타낼 수도 있다5 보기 π θ π에서 f (θ) cos θ + sin θ의 최댓값을 구하고 f (θ) 이 되는 θ를 구하여라 Solution cos θ + sin θ h i cos θ + sin θ 이다 cos α, 가 되는 α를 구하면 α sin α π 이므로 cos θ + sin θ π cos θ 이다 y - Π Π 5Π 9Π Θ - - π 에서 이다 f (θ) 이 되는 θ는 π π cos θ, 또는 cos θ 따라서 f (θ)의 최댓값은 θ 를 만족하므로 θ 5 연습문제 참조 π π, 또는 θ π π

69 6 삼각함수 65 이다 따라서 θ, π 에서 f (θ) 이 된다 6 삼각함수의 극한값 삼각함수와 관련된 다음 두 극한값은 장에서 삼각함수의 도함수를 구하는 데에 중요한 역할을 한다 보기 (a) lim t 다음 식이 성립함을 보여라 sin t t (b) lim t cos t t Solution (a) < t < π 이면 호 AB의 길이는 t이고 BC 호 AB 의 길이 AT 이므로 < sin t t tan t sin t cos t 이다 T B t sinhtl t O C A 따라서 첫번째 부등식에서 sin t t 이다 또한 두번째 부등식에서 sin t t 이고 lim cos t 이므로 샌드위치 정리에 의하여 cos t t lim+ t sin t t tanhtl

70 CHAPTER 함수와 극한 66 이다 t < 이면 s t > 이라고 하자 그러면 t 일 때 s + 이고 사인함수는 기함수이므로6 sin t sin( s) sin s lim lim lim + + t s s t s s 을 얻는다 따라서 lim t sin t t 이 성립한다 - Π - Π -Π Π Π Π (b) 분자, 분모에 모두 ( + cos t)를 곱하면 sin t cos t + cos t t + cos t t + cos t 을 얻는다 sin t, lim t t t + cos t lim 이므로 cos t t t lim 이다 6 정의역의 sin t lim t t t + cos t lim 모든 에 대하여 f ( ) f ()이면 기함수(odd function), f ( ) f ()이면 우함수(even function)라고 한다

71 6 삼각함수 67 연습문제 6 <, y < π 5 이고 sin, cos y 일 때 다음을 구하여라 (a) cos (b) cos (c) sin( + y) (d) tan( + y) 다음 식의 값을 구하여라 (a) (cos 75 + sin 75 )(cos 75 sin 75 ) (b) sin 5 + cos 5 코시컨트함수(cosecant), 시컨트함수(secant), 코탄젠트함수(cotangent) 는 각각 다음 과 같이 정의된다 csc, sin sec, cos cot tan 다음 관계식이 성립함을 보여라 (a) + tan sec sin cos (b) + cot csc 일 때 다음 값을 구하여라 (b) sec + csc (a) tan + cot 5 < < π 이고 tan + cot 일 때 다음 값을 구하여라 (a) sin cos (b) sin + cos 6 다음 함수의 최댓값과 최솟값, 주기를 구하여라 (a) f () sin 5 cos (b) f () sin + cos 7 길이가 인 선분 AB를 지름으로 하는 반원이 있다 호 AB 위의 점 P 에 대하여 AP + BP 의 최댓값을 구하여라7 P A 7 Hint 각 AP B는 직각이다 B

72 CHAPTER 함수와 극한 68 8 삼각함수의 덧셈법칙을 이용하여 다음 식을 증명하여라 A+B A B cos A+B A B (b) sin A sin B cos sin A+B A B (c) cos A + cos B cos cos A+B A B (d) cos A cos B sin sin (a) sin A + sin B sin 9 다음 극한값을 구하여라 (a) lim sin[( + )π] (b) lim sin π sin 을 구하여라 sin 극한값 lim 다음 극한값을 구하여라 sin sin( ) (b) lim (sin ) (c) lim (a) lim sin sin sin (e) lim + sin(sin ) (f) lim (d) lim cos k cos k (h) lim cos (i) lim cos (g) lim 등식 lim sin 을 만족하는 상수 a, b 를 구하여라 + a + b lim sin 임을 보여라( ) 중간값정리를 이용하여 다음 방정식의 해가 주어진 구간에 존재함을 보여라 (a) + cos, [, ] (b) sin cos, [, π6 ]

73 Chapter 함수와 미분 미분계수와 접선 세상의 대부분의 것은 시간이 흐르면 변한다 이러한 변화는 변화율을 사용하여 수학적으로 나타내는데 변화율은 변화가 어느 방향으로 얼마나 빠르게 일어나는지를 나타낸다 변화가 시 간에 관계없이 일정하게 일어나면 직선의 형태로 나타낼 수 있는데 직선의 기울기는 변화율을 나타낸다 그러나 대부분의 경우 변화율은 시간에 따라 다르게 나타난다 이 절에서는 변화율에 관한 체계적이고 직접적인 방법을 살펴 보기로 한다 평균변화율과 순간변화율 함수 y f ()가 a를 포함하는 열린구간에서 정의되었다고 하자 변수 가 a에서 a + h로 변할 때, 의 변화량을 h 로 나타내고 그에 대응하는 종속변수 y의 변화량을 y f (a + h) f (a) 로 나타내자 여기에서 y의 변화량을 의 변화량으로 나눈 몫 f (a + h) f (a) y h 을 구간 [a, a + h]에서 에 대한 f 의 평균변화율(average rate of change)이라 한다 평균 변화율은 다음 그림에서 선분 P Q의 기울기이다 h > 일 필요는 없다 h < 인 경우 구간 [a, a + h]대신 구간 [a + h, a]로 생각한다 69

74 CHAPTER 함수와 미분 7 y QHa + h, f Ha + hll PHa, f HaLL a a+h 함수 y f ()에 대하여 h가 에 가까워짐에 따라 평균변화율 y f (a + h) f (a) h 가 어떤 값에 수렴하면 이 값을 a에서의 순간변화율, 또는 a에서의 미분계수 (derivative)라고 하고 f (a), 또는, y a 로 나타낸다 극한값 f (a)가 존재할 때 함수 f 는 a에서 미분가능하다 (differentiable)고 한다 a + h라고 하면 h 일 때 a이므로 f () f (a) f (a + h) f (a) lim a h h a f (a) lim 으로 나타내기도 한다 a에서 y f ()의 미분계수 f (a) lim h 보기 정의를 이용하여 에서 다음 함수의 미분계수를 구하여라 (a) g() Solution f (a + h) f (a) f () f (a) lim a h a (b) h() (a) 정의에 의하여 구하면 다음과 같다 +h ( + h )( + h + ) g () lim lim h h h h( + h + ) h lim lim h h( + h + ) h + h +

75 미분계수와 접선 7 (b) 다시 정의에 의하면 h () lim +h lim h lim h ( + h) h h ( + h) h( + h) 을 얻는다 기초다지기 미분계수의 정의를 이용하여 주어진 함수의 a에서의 미분계수를 구하여라 (a) f (), a (c) h() (b) g() +, a (d) k() +, a,a 미분가능성 f 가 a에서 미분가능하다고 하자 그러면 f () f (a) f () f (a) ( a) a 에서 다음 두 극한값이 존재한다 lim a f () f (a) f (a), a lim ( a) a 따라서 lim (f () f (a)) f (a) a 또는 lim f () f (a) a 이 성립하고 f 는 a에서 연속이다 대우를 생각하면 함수 f 가 a에서 연속이 아니면 미분가능하지 않다 는 것을 알 수 있다 그러나 그 역은 참이 아니다 다시 말해서 f 가 a에서 연속이라고 해서 항상 미분가능한 것은 아니다 미분가능성과 연속성 함수 f 가 a에서 미분가능하면 a에서 연속이다 함수 f 가 a에서 연속이 아니면 a에서 미분불가능하다 함수 f 가 a에서 연속이라고 해서 a에서 항상 미분가능한 것은 아니다 (a) 반례는 (b) (c) (d) 보기 를 참조할 것

76 CHAPTER 함수와 미분 7 h > 이면서 으로 갈 때 우극한 lim+ h f (a + h) f (a) h (a)로 f+ 나타내고 a에서 f 의 우미분계수(right-hand derivative) 가 존재하면 그 값을 라고 한다 마찬가지로, h < 이면서 으로 갈 때 좌극한 lim h f (a + h) f (a) f (a h) f (a) lim+ h h h 가 존재하면 그 값을 f (a)로 나타내고 a에서 f 의 좌미분계수(left-hand derivative)라 고 한다 극한의 성질로부터 우미분계수 f+ (a)와 좌미분계수 f (a)가 존재하고 두 값이 같으면 f 는 a에서 미분가능하고 이때, f (a) f+ (a) f (a) 이다 미분가능성 다음 두 명제는 동치이다 f 는 a에서 미분가능하다 (a)이다 (a)이다 이때 f (a) f+ (a) f (a)가 존재하고 f+ (a)와 f f+ 보기 Solution 에서 함수 y f () 의 미분가능성을 조사하여라 구간 [, h]에서 평균변화율을 구하면 h, h f ( + h) f () h h, h h> h< 이므로 f+ (), f () 이다 따라서 f 는 에서 미분가능하지 않다 보기 다음 함수가 에서 미분가능하도록 b, c의 값을 구하여라 ( f () h, b + c, < 이면 구간 [, h]는 [h, ]으로 정의하기로 한다 >

77 미분계수와 접선 Solution 7 에서 미분가능하려면 연속이어야 하므로 lim (b + c) b + c f () + 이다 에서 좌미분계수와 우미분계수가 같아야 하므로 f () f+ () ( + h) h h b( + h) + c b c lim+ b h h lim 에서 b, c 이다 y 8 - y - 접선의 방정식 유클리드에 따르면 원의 접선(tangent line)은 원과 한 점에서 만나는 직선이다5 그렇다면 일반적인 그래프에 대하여 접선은 어떻게 정의할까? 원과 접선을 접점 근방에서 확 대해 보면 원과 접선이 아주 비슷해 보인다 일반적으로 f (a)가 존재할 때 곡선 y f () 의 그래프를 점 P (a, f (a))근방에서 확대하면 할수록 곡선은 직선처럼 보인다 이런 경우 곡선 y f ()는 a에서 국소적으로 선형(locally linear)이라 한다 5 포물선, 타원, 쌍곡선과 같은 이차곡선에 대하여도 마찬가지이다

78 CHAPTER 함수와 미분 7 A B A C B C 그래프를 확대하면 할수록 곡선 y f ()위의 한 점 P (a, f (a))근방에서의 평균변화율 f (a + h) f (a) h 는 미분계수 f (a)로 수렴한다 따라서 미분계수 f (a)를 그 점에서 곡선의 기울기(slope of curve)로 정의하고 점 P (a, f (a))에서 곡선의 기울기와 같은 기울기를 갖는 직선을 접선(tangent line)으로 정의한다 접선의 방정식 곡선 y f ()가 a에서 미분가능하다고 하자 이때 곡선 y f ()위의 점 P (a, f (a)) 에서 접선의 기울기는 f (a + h) f (a) m f (a) lim h h 이다 따라서 점 P (a, f (a))에서 접선의 방정식은 다음과 같다 y f (a)( a) + f (a) 보기 Solution 보기 의 함수에 대하여 에서 접선의 방정식을 구하여라 (a) g () 이므로 곡선 위의 점 P (, )에서 접선의 방정식은

79 미분계수와 접선 75 y ( ), y ( + ) 이다 (b) h () 이므로 곡선 위의 점 Q(, )에서 접선의 방정식은 y ( ) y + 이다 y y M PI, P, 기초다지기 주어진 함수의 a에서 접선의 방정식을 구하여라6 (a) f (), a (c) h() (b) g() +, a (d) k() +, a,a 선형근사식 미분가능한 한 점에서 곡선의 그래프를 확대하면 할수록 접선과 비슷해진다는 사실은 함수의 근삿값을 구하는 데에 유용하게 활용할 수 있다 함수 y f ()가 a에서 미분가능하다고 하자 점 (a, f (a))에서 접선의 방정식 y L() f (a) + f (a)( a) 은 가 a에 충분히 가까울 때 곡선 y f ()의 근사식으로 쓸 수 있다 근사식 f () f (a) + f (a)( a) 을 함수 f 의 a에서의 선형근사식(linear approimation), 또는 일차근사식(first order approimation)이라고 한다 a, 6 (a) y (b) y (c) y + y f () f (a) 5 (d) y +

80 CHAPTER 함수와 미분 76 라고 하면 선형근사식은 다음과 같이 표현하기도 한다 y f (a + ) f (a) f (a) f (a)( a) 선형근사식 또는 일차근사식 함수 y f ()가 a에서 미분가능이면 a 근방의 에 대하여 f () L() f (a) + f (a)( a) 이다 여기서 L()를 a에서 f 의 선형근사식 또는 일차근사식이라 한다 보기 5 에서 함수 f () + 의 선형근사식을 구하여라 이 결과를 이용하여 99,,, 5, 의 근삿값을 구하여라 에서의 미분계수는 +h + h lim f () lim h h( + h + h h + ) 이고 f () 이므로 에서 + 의 선형근사식은 Solution y 이다 즉,, 또는 y + + L() + 이므로 다음과 같이 근삿값을 구할 수 있다 99 5 p + ( ) ; + ( ) ; + () 5 + ; + () ; + (5) 5 + ; + () 5 다음 그래프는 곡선 y + 와 에서 선형근사식을 그린 것이다

81 미분계수와 접선 77 y y + 5 y 접선이 곡선 위에 있으므로 근방에서의 근삿값은 함수의 참값보다 크다 그 차이는 가 커질수록 증가함을 볼 수 있다 다음 표는 선형근사에 의한 근삿값과 함숫값을 비교한 것이다 그래프에서 볼 수 있듯이 가 큰 경우, 오차가 커지므로 좋은 근삿값이라고 할 수 없다 99 5 근삿값 L() 함수값 f () 오차

82 CHAPTER 함수와 미분 78 연습문제 함수 f () 6에 대하여 (a) 곡선 위의 점 (, 5)에서 미분계수를 구하여라 (b) 곡선 위의 점 (, 5)에서 접선의 방정식을 구하여라 (c) (b)에서 구한 접선과 y f ()의 교점을 모두 구하여라 정의를 이용하여 다음 함수의 주어진 점에서 미분계수를 구하고 접선의 방정식을 구하여 라, (d) y +, 5, (b) y, (a) y (c) y f () 5일 때 다음 극한값을 구하여라 lim f ( + ) f ( ) f (), f () 5 일 때 다음 극한값을 구하여라 lim f ( + ) + f ( ) 5 다음 함수는 에서 미분가능한지 판단하여라 미분가능한 경우 f ()을 구하여라 sin, 6 (a) f (), sin, 6 (b) f (), 6 정의를 이용하여 다음 미분계수를 구하여라 (a) G() (b) K(), + +, G (), G () K (), K ( ) 7 함수 f 가 에서 미분가능하면 f ()을 구하고 미분가능하지 않으면 그 이유를 설명 하여라 p (a) f () p (b) f ()

83 미분계수와 접선 ( (c) f () 79 csc, 6, 8 다음 함수가 a에서 미분가능하도록 상수 b, c를 정하여라 (, (a) f () a b + c, >, (b) f () a b + c, > 9 f (), f () 인 함수 f ()에 대하여 다음 값을 구하여라 f ( + h ) h h f ( + h) f ( + h) (d) lim h h f () f ( ) (b) lim (a) lim (c) lim 선형근사식 ( + )k + k, (k는 실수)을 이용하여 다음 값들의 근삿값을 구하여라 (a) (8) (b) (c) 5 (d) 에서 f () sin 의 선형근사식을 구하고 이를 이용하여 sin 의 근삿값을 구하 여라 정육면체의 한 변의 길이가 반올림하여 cm 이다 이 정육면체의 부피의 범위를 선형근 사를 이용하여 구하여라

84 CHAPTER 함수와 미분 8 도함수와 미분법칙 도함수 함수 y f ()가 미분가능한 의 집합을 I라고 하면 f () lim h f ( + h) f () h 로 정의된 함수 f : I R을 f 의 도함수(derivative)라고 부른다 도함수의 정의 함수 f ()의 도함수는 f ( + h) f () h h f () lim 이다 Note y f ()의 도함수는 다음과 같이 여러 방법으로 나타낸다 y, Note f (), dy, d df () d 극한의 성질로부터 도함수의 다음 성질을 쉽게 보일 수 있다 (f + g) () f () + g (), 보기 Solution (cf ) () cf () y n 의 도함수를 구하여라 단, n은 자연수이다 f () n 이라고 하면 공식 an bn (a b)(an + an b + + abn + bn ) 으로부터 ( + h)n n h ( + h )[( + h)n + ( + h)n + + ( + h)n + n ] h n n ( + h) + ( + h) + + ( + h)n + n 을 얻는다 i,, n에 대하여 lim ( + h)n i i n h

85 도함수와 미분법칙 8 이므로 (n ) ( + h)n n h h lim ( + h)n + ( + h)n + + ( + h)n + n lim h n + n + + n (모두 n개) nn 을 얻는다 Note 상수함수 f () 의 도함수는 임의의 실수 에 대하여 f () lim h f ( + h) f () lim h h h 이다 이는 f () 을 f () 으로 나타내면 위 보기의 결과와 일치한다 기초다지기 주어진 함수의 도함수를 구하여라7 (a) f () (d) f () (g) f () + + (b) f () π (e) f () + (h) g() 8 85 (c) f () (f) f (t) 보기 Solution > 에 대하여 y ( + t ) (i) h() + 의 도함수를 구하여라 > 에 대하여 dy +h +h +h+ lim lim h h d h h +h+ h lim h h( + h + ) 이다 보기 Solution f () sin 와 g() cos 의 도함수를 구하여라 사인함수의 덧셈법칙에서 sin( + h) sin cos h + cos sin h f () (b) f () (c) f () (d) f () 8 (e) f () + + (h) f () 7 (i) f () (a) f () (f) f (t) t (g)

86 CHAPTER 함수와 미분 8 이므로 sin( + h) sin sin (cos h ) + cos sin h 이 된다 따라서 (sin ) sin( + h) sin h h cos h sin h i lim sin + cos h h h cos h sin h sin lim + cos lim h h h h sin + cos lim h cos 이다 같은 방법으로 코사인함수의 덧셈법칙에서 다음 결과를 보일 수 있다8 (cos ) sin 함수의 연산과 미분법칙 두 함수의 합의 도함수는 각 도함수를 더한 것과 같다 그렇다면 두 함수의 곱의 도함수는 각 도함수를 곱한 것과 같을까? 다시 말해서 (f ()g()) f ()g () 가 성립할까? f () g() 인 경우를 생각해 보면 그렇지 않음을 간단히 알 수 있다 즉, (f ()g()) ( ) 이지만 f ()g () 이므로 (f ()g()) 6 f ()g () 이다 이제 곱의 도함수를 구해 보기로 한다 f, g가 미분가능한 함수라고 하자 (f ()g()) lim h f ( + h)g( + h) f ()g() h 의 분자에서 f ()g( + h)를 빼고 더하면 f ( + h)g( + h) f ()g() f ( + h)g( + h) f ()g( + h) + f ()g( + h) f ()g() [f ( + h) f ()]g( + h) + f ()[g( + h) g()] 로 쓸 수 있다 이로부터 f ( + h)g( + h) f ()g() h 8 연습문제 f ( + h) f () g( + h) g() g( + h) + f () h h

87 도함수와 미분법칙 8 을 얻는다 lim g( + h) g()이므로 양변에 극한을 취하면 h (f ()g()) f ()g() + f ()g () 을 얻는다 g() 6 인 에 대하여 의 도함수를 구하면 g() g() g( + h) g( + h) g() lim lim h h hg( + h)g() h g( + h) g() lim lim h h g( + h)g() h g () [g()] g() 이다 곱의 미분법칙과 식 ()을 같이 이용하면 f () f () 의 도함수를 다음과 같이 g() g() 구할 수 있다 f () g() + f () g() g() f () g () f () g() [g()] f ()g() f ()g () [g()] f () 이 결과를 몫의 미분법칙이라고 한다 지금까지 다룬 미분법칙을 정리하면 다음과 같다 함수의 연산에 대한 미분법칙 미분가능한 두 함수 f (), g()에 대하여 다음이 성립한다 (f () ± g()) f () ± g () (cf ()) cf () (f ()g()) f ()g() + f ()g () f () f ()g() f ()g () g() 6 이면, g() [g()] 기초다지기 9 (a) 7 (b) f (a), f (a), g(a), g (a) 5일 때 다음 값을 구하여라9 (c) 7 (d) 7 9 ()

88 CHAPTER 함수와 미분 8 (a) (f + g) (a) (c) (f /g) (a) (b) (f g) (a) (d) (g/f ) (a) 보기 다음 함수의 도함수를 구하여라 (a) y n Solution n (b) y (n은 자연수) (a) (n ) nn 이므로 (절 보기 6) nn (n ) n n n (n ) (n ) 이다 (b) ( ) 이므로 ( ) y ( ) 이다 기초다지기 다음 함수의 도함수를 구하여라 + (a) y + (d) y (b) y t t (e) y + (c) y s + s + s (f) y + 보기 5 곡선 y (g) y t t u u + u + u + p (i) y p+ (h) y 에 대하여 + (a) 곡선 위의 점 (, )에서 접선의 방정식을 구하여라 (b) 축과 평행한 접선이 있으면 모두 구하여라 이 결과와 절의 보기 6을 합하면 모든 정수 n에 대하여 다음 식이 성립한다 (n ) nn (a) (f) y y (+) ( +) (b) y t + (g) y t t ( t) t (c) y (h) y u (u +u+) s (i) s y (d) y p( p+) (e) y (+)

89 도함수와 미분법칙 85 (c) 곡선 위의 임의의 점에서 접선의 기울기는 보다 커질 수 없음을 설명하여라 Solution (a) y ( + ) () 이므로 에서 접선의 기울기는 ( + ) ( + ) m ( + ) 5 이다 따라서 접선의 방정식은 y ( ) 5 5 y 이다 (b) 기울기가 이 되는 를 구하면 ( + ) ± 이므로 축과 평행한 두 개의 접선 y ± 이 존재한다 (c) + 이므로 임의의 에 대하여 접선의 기울기는 y ( + ) + 이다 따라서 접선의 기울기는 항상 절댓값이 보다 작거나 같다 5 - y tan 보기 6 Solution sin 이므로 탄젠트함수의 도함수는 몫의 미분법으로 구할 수 있다 cos f () tan 의 도함수를 구하여라 몫의 미분법으로부터 다음 결과를 얻는다 (tan ) (sin ) cos sin (cos ) cos cos cos + sin sec cos cos sin

90 CHAPTER 함수와 미분 86 삼각함수의 도함수에 대한 결과를 정리하면 다음과 같다 삼각함수의 도함수 (sin ) cos, 기초다지기 (tan ) sec 다음 함수의 도함수를 구하여라 (a) f () sin (cos ) sin, (b) f () cos (c) f () tan 연쇄법칙 : 합성함수의 미분법칙 y f (u), u g()가 미분가능한 함수이고 F () (f g)() f (g()) 라고 하자 에 대응하는 g()의 변화량을 u, u에 대응하는 f 의 변화량을 y라고 하자 u g( + ) g() y f (u + u) f (u) 그러면 F ( + ) F () f (g( + )) f (g()) f (u + u) f (u) y 이므로 에 대응하는 F ()의 변화량은 y가 된다 따라서 에 대한 F 의 평균변화율은 y y u u 로 쓸 수 있다 함수 g()는 연속이므로 이면 u 이다 따라서 양변에 일 때의 극한값을 취해 주면 h y u i dy y lim lim u d y u dy du lim lim u u du d 을 얻는다 다시 말해서 F () [f (g())] f (u)g () f (g())g () (a) f () sin + cos (b) f () cos sin (c) f () tan + sec

91 도함수와 미분법칙 87 이 성립한다 합성함수에 대한 이러한 미분법칙을 연쇄법칙(chain rule)이라고 한다 특히, f (u) un 이면 f (g()) [g()]n 이고 f (u) nun 이므로 dy f (g())g () n[g()]n g () d 이다 연쇄법칙 함수 f (u)가 u g()에서 미분가능하고 함수 g()가 에서 미분가능하다고 하자 그러면 합성함수 F () f (g())는 에서 미분가능하고 다음 식이 성립한다 F () ([g()]n ) 보기 7 f (g())g () n[g()]n g () 다음 함수의 도함수를 구하여라 (a) y ( + ) ( ) (c) y + (b) y ( + ) Solution (a) 우선 곱의 미분법칙에 의하여 h i h i dy ( + ) ( ) + ( + ) ( ) d 이다 f (u) u, g() + 로 놓으면 ( + ) f (g())이므로 첫번째 미분은 [f (g())] Note (g()) g () ( + ) () 6( + ) 연쇄법칙은 세 개 이상의 함수가 합성되었을 때에도 같은 방법으로 주어진다 예를 들어 h(t), y g(), z f (y) 라고 하면 z f (g()) f (g(h(t)))에 대하여 dz dz d dz dy d dt d dt dy d dt 이다 따라서 F (t) (f g h)(t) f (g(h(t)))에 대하여 다음 식이 성립한다 F (t) dz dz dy d f (y)g ()h (t) dt dy d dt f (g(h(t)))g (h(t))h (t)

92 CHAPTER 함수와 미분 88 이다 같은 방법으로 h i ( ) ( ) (6) ( ) 을 얻는다 따라서 dy d 6( + ) ( ) + ( + ) ( ) 6( + ) ( ) (7 + ) 이 된다 (b) y ( + ) 이므로 y ( + ) () ( + ) 이다 (c) y ( + ) 이므로 y ( + ) ( + ) ( + ) + 이다 기초다지기 다음 함수의 도함수를 구하여라 (e) y + (f) y + (a) y ( )7 (b) y ( + ) (c) y ( + )5 (d) y ( + + ) (g) y + (h) y (i) y (j) y ( + ) ( + ) 연쇄법칙의 응용 가격이 p일 때 판매량(수요량)을 f (p), 판매량이 f (p)일 때 생기는 총수입과 총비용을 각각 R(), C()라고 하자5 총이윤(P )은 총수입(R)에서 총비용(C)을 제하면 얻어진다 P () R() C() 가격이 p일 때의 총이윤을 F (p)라고 하면 F (p) P () P (f (p))이고 df dp d (p) dp d dp (a) y 7( )6 ( + ) (f) y (j) y 6( + ) y 5 생산량과 (b) y 6( + ) (c) y ( + ) (d) y ( + )( + + ) (e) + (g) y ( + ) (h) y (i) y 8( + ) 판매량이 일치한다고 가정한다

93 도함수와 미분법칙 89 d dp 은 가격의 변화에 따른 판매량의 변화율을 의미하고, 은 판매량의 변화에 dp d 따른 총이윤의 변화율을 나타낸다 이다 여기서 보기 8 설탕을 (kg) 생산하는데 비용이 다음과 같이 주어진다 C() + + 설탕의 가격이 kg 당 p(단위 천원) 이면 수요량은 다음과 같다 f (p) p 설탕 가격이 kg 당 6원일 때 가격을 원 올리거나 내리면 총이윤은 어떻게 변하는지 근사 적으로 구하여라 단, 재고는 생기지 않는다고 가정한다 Solution 가격이 p인 설탕을 kg 팔면 총수입은 p이고 p + 이므로 R() + + 이다 따라서 총이윤은 P () R() C() + ( + + ) 9 + 이다 총이윤을 가격의 함수로 나타내면 F (p) P (f (p))이고 df dp d ( ) dp d dp (p ) 이다 p 6일 때 5이므로 df (6) ( 5) ( 65) 5 dp (6 ) 이다 따라서 가격을 원( p )을 올리면 F (6) F (6) ; F (6) 5 F (6) F (6) ; 5 이다 즉, 가격을 원 올리면 총이윤은 약,5원 감소한다 반면 가격을 원 인하하면 총이윤은 약,5원 증가한다

94 CHAPTER 함수와 미분 9 5 고계도함수 y f ()가 미분가능한 함수라면 f 의 도함수 f 도 함수이다 따라서 f 이 미분가능하면 f 의 도함수를 생각할 수 있다 여기서 f 의 도함수를 f 로 나타내고 f 의 이계도함수(second derivative)라고 부른다 d f () (f ()) [f ()] d 이계도함수는 다음과 같은 기호로 나타낸다 f (), d y, d d dy, d d y 같은 방법으로 f 의 삼계도함수는 f 로 나타내고 f 의 도함수로 정의한다 일반적으로 자연수 n에 대하여 n계도함수는 f (n) ()로 나타내고 다음과 같이 귀납적으로 정의한다 f (n) () [f (n ) ()] d (n ) [f ()] d 다음 함수의 k-계도함수를 구하고 f (k) ()을 구하여라 단, k n이다 보기 8 f () a + a + + an n n X ai i i Solution (i ) ii 이므로 k i이면 (i )(k) i(i ) (i k + )i k i i k (i k) 이다 특히, i k이면 (i )(k) k이고 i < k이면 (i )(k) 이다 f (k) () n X ik ai (i )(k) n X ik ai i i k (i k) (k + ) n kak + ak+ + + an n k (n k) 이므로 다음 결과가 성립한다 f (k) () kak Note 보기 8에서 k > n이면 f (k) () 이다 기초다지기 k,,, 에 대하여 k-계 도함수 f (k) ()를 구하고 f (k) ()을 구하여라6 (a) f () (b) f () sin f () + +, f (), f () + 6, f (), f () () 6, f () () 6, f () () (b) f () cos, f (), f () sin, f (), f () () cos, f () () (), f () sin, f () () 6 (a), f () ()

95 도함수와 미분법칙 9 연습문제 다음 함수의 도함수를 구하여라 (a) g() ( + )( + ) (y + y ) (b) h(y) y y (c) f () u + u + (d) f (u) u (e) f () + (f) g() a + a (g) f () + f () g()이라고 한다 (a) g() 5, g () 7일 때 f ()의 값을 구하여라 (b) f (), f () 일 때 g ()의 값을 구하여라 주어진 함수의 접선을 주어진 점에서 구하여라 (a) f (), + (b) f () (, 5) +, (, ) f () sin cos 일 때 f () cos 임을 보여라 5 다음 식이 성립함을 보여라 (sec ) tan sec, (csc ) cot csc, (cot ) csc 6 다음 함수의 도함수와 이계도함수를 구하여라 (a) y + (b) r θ + θ + θ 7 이차다항식 f ()가 다음 조건을 만족할 때 f ()를 구하여라 (a) f (), f (), f () (b) f (), f (), f () 8 어느 도시의 일인당 일일 전기 소비량은 5단위라고 한다 에너지 절약운동으로 전기 소비는 한달(일 기준)에 일인당 단위씩 감소하고 있다고 한다 현재 이 도시의 인구 는 5만명이고 일년에 인구가 7명씩 증가한다면 에너지 절약운동을 시작하고 일 후의 일일 총전기 소비량의 변화율은 어떻게 되는가? 시간의 단위에 주의하여 구하여라 9 에서 미분가능한 함수가 모든 실수, y에 대하여 f ( + y) f () + f (y) + y, 일 때 f (), f ()을 구하여라 lim h f (h) h

96 CHAPTER 함수와 미분 9 임의의 실수, y에 대하여 함수 f ()는 다음 등식을 만족한다고 한다 f ( + y) f () + f (y) f () 일 때 f ()과 f ()을 구하여라 항상 양의 값을 갖는 함수 f ()는 임의의 실수, y에 대하여 다음 등식을 만족한다고 한다 f ( + y) f ()f (y) f () 일 때 f ()을 구하고 f ()을 f ()를 이용하여 나타내어라 f (a), f (a), g(a), g (a) 5라고 하자 a에서 다음 함수의 미분계수 를 구하여라 (a) p f () (c) p (d) + [g()] + [f ()] (b) [f ()] [f ()] + [g()] 다음과 같이 주어진 y f (u), u g()에 대하여 y f (g())를 구하고 dy 를 구하여라 d (a) f (u) u +, g() + (b) f (u) u, g() tan (c) f (u) u, g() + (d) f (u) u,, g() + 다음 함수의 도함수를 구하여라 (a) y ( )5 (b) y ( + ) (c) f () + (d) y + (e) y ( + + ) (f) h(s) s s (g) g(t) sin t (j) f () sin (k) g() (h) r(u) sin(u ) (i) f () (sin ) (l) y g(), g( ), f (), f ( ) 7, g () g ( ) 이다 a에서 다음 함수의 미분계수를 구하여라 (a) f ()g(), a (b) /f (), a (c) f (g()), a (d) g(f ()), a, a f (g()) p (f) f () + g(), a (e) (g) f ( + g()), a cos + 5 ±에서 미분가능한 함수 f, g에 대하여 f (), f ( ),

97 평균값 정리와 극값 9 평균값 정리와 극값 최대 최소 정리에 의하면 닫힌구간 [a, b]에서 정의된 연속함수는 항상 최댓값 M 과 최솟값 m을 갖는다 평균값정리 함수 f ()가 닫힌구간 [a, b]에서 연속이고 열린구간 (a, b)에서 미분가능하다고 하자 f (a) f (b)라고 하자 M m이면 f ()는 닫힌구간 [a, b]에서 상수함수이다 따라서 a < c < b인 모든 c에 대하여 f (c) 이 성립한다 M > m이면 최댓값, 또는 최솟값을 갖는 점에서 도함수의 값은 이다 다시 말해서, 열린구간 (a, b)의 한 점 c에 대하여 또는 f (c) m f (c) M 이면 f (c) 이다 이 결과를 롤의 정리(Rolle s theorem)라고 한다7 y y M M m m a b a b 롤의 정리 함수 f ()가 닫힌구간 [a, b]에서 연속이고 열린구간 (a, b)에서 미분가능하다고 하자 f (a) f (b)이면 f (c) 을 만족하는 c가 구간 (a, b)안에 적어도 하나 존재한다 Note 함수 f ()가 열린구간 (a, b)에서 미분가능하다는 조건이 성립하지 않으면 롤의 정리는 성립하지 않을 수도 있다 예를 들어, f () 는 닫힌구간 [, ]에서 연속이고 f ( ) f () 이지만, 열린구간 (, )에서 f (c) 인 c 는 존재하지 않는다 7 증명은 연습문제 참고할 것

98 CHAPTER 함수와 미분 9 y - 롤의 정리를 이용하면 다음 평균값정리(mean value theorem)를 얻는다 평균값 정리 함수 f ()가 닫힌구간 [a, b]에서 연속이고 열린구간 (a, b)에서 미분가능하다고 하자 그러 면 다음 등식을 만족하는 c가 구간 (a, b)안에 적어도 하나 존재한다 f (c) f (b) f (a) b a Note 평균값정리는 어떤 구간에서 정의된 미분가능한 함수에 대하여 평균변화율과 순간변 화율이 같아지는 점이 존재한다는 것을 말해 준다 y y f HL a c b 평균값 정리의 증명 : 롤의 정리로부터 평균값 정리를 증명하기로 한다 함수 g()를 다음과 같이 정의하자 f (b) f (a) g() f () f (a) ( a) b a g()는 구간 [a, b]에서 연속이고 구간 (a, b)에서는 미분가능하다 또한 g(a) g(b) 이므로 롤의 정리에 의하여 g (c) f (c) 을 만족하는 c가 열린구간 (a, b) 안에 존재한다 f (b) f (a) b a

99 평균값 정리와 극값 보기 95 임의의 실수, y에 대하여 다음 부등식이 성립함을 보여라 sin sin y y Solution y이면 등식이 성립한다, y가 서로 다른 실수라고 하자 (sin ) cos 이므로 평균값정리에 의하여 sin sin y cos c y 인 c가 와 y 사이에 존재한다 따라서 sin sin y cos c y sin sin y y 이 성립한다 도함수와 함수의 증감 구간 [a, b]의 모든 에 대하여 f () 이라고 하자 평균값정리에 의하면 [a, b]에 대하여 f () f (a) f (c) a 인 c (a, b)가 존재한다 따라서 모든 [a, b]에 대하여 f () f (a) 인 상수함수가 된다 이제 구간 I에서 정의된 함수 y f ()에 대하여 항상 f () > 이면 어떤 일이 일어나는지 살펴보기로 하자 구간 내의 임의의 두 점, y ( < y)에 대하여 f (y) f () f (c) y 인 점 c가 [, y]에 존재한다 따라서 < y이면 f (y) f () f (c)(y ) > 이 성립한다 다시 말해서 구간 I에서 함수 y f ()는 순증가함수(strictly increasing function)이다8 마찬가지로 구간의 모든 점에서 f () < 이면 f 는 그 구간에서 순감소 함수(strictly decreasing function)이다 8 구간 I의 임의의 두 점, y ( < y)에 대하여 f () f (y)( 또는 f () < f (y)) 이면 f 를 증가함수(또는, 순증가함수)라고 한다

100 CHAPTER 함수와 미분 96 도함수와 함수의 증감 함수 y f ()가 구간 I의 모든 에 대하여 f () 이면 f ()는 상수함수이다 f () > 이면 f ()는 순증가함수이다 f () < 이면 f ()는 순감소함수이다 Note f () 이면 f ()는 증가함수이고, f () 이면 f ()는 감소함수이다 보기 구간 [, π]에서 다음 함수가 증가하는 구간과 감소하는 구간을 구하여라 f () sin Solution f () 인 구간과 f () 인 구간을 구하면 된다 f () cos cos 이므로 π/, 5π/에서 f () 이다 그 이외의 구간에서 f ()의 부호를 구해보면 다음과 같다 << f () f () & π π << + % 5π 5π < < π & π 5π 따라서 함수 y f ()는 구간 [ π, 5π ]에서는 증가하고 나머지 구간 [, ], [, π]에서는 감소 한다 기초다지기 다음 함수가 증가하는 구간과 감소하는 구간을 구하여라9 (a) f () (b) f () + (c) + 위의 결과는 부등식을 증명하는 데에도 유용하다 다시 말해서, f (a) b일 때, > a에 대하여 다음 결과가 성립한다 f () > 이면 f () > b (f () 이면 f () b) f () < 이면 f () < b (f () 이면 f () b) 9 (a) 증가 (, ], [, ), 감소 [, ] (, ], [, ) (b) 증가 [, ), 감소 (, ] (c) 증가 [, ], 감소

101 평균값 정리와 극값 보기 97 f (), g() sin 에 대하여 이면 항상 f () g()임을 보여라 Solution h() f () g() sin 라고 하자 그러면 h() 이고 h () cos 이므로 h()는 증가함수이다 따라서 이면 h() h() 이므로 f () g() 가 성립한다 미분가능한 함수 y f ()에 대하여 f (a)f (b) < 이면 중간값정리에 의하여 구간 (a, b)에 적어도 하나의 근이 존재한다 이 구간에서 f () >, 또는, f () < 이면 y f ()는 순증가, 또는 순감소하므로 유일한 근이 존재한다 보기 f () cos 는 구간 (, π )에서 유일한 근을 가짐을 보여라 Solution f 는 연속함수이고 f () <, f ( π ) (, π ) 에서 적어도 하나의 근을 갖는다 π > 이므로 중간값정리에 의하여 구간 f () + sin > 이므로 f 는 구간 (, π )에서 순증가함수이고 따라서 유일한 근을 갖는다 최대와 최소 : 전체에서와 부분에서 함수의 정의역에서 가장 큰 함숫값을 최댓값, 가장 작은 함숫값을 최솟값이라고 한다 수학적 으로 엄밀하게 쓰면 다음과 같다 최댓값, 최솟값 D에서 정의된 함수 f 와 점 D에 대하여 D의 모든 점 에 대하여 f () f ( )이면 을 f 의 최대점이라 하고 f ( )을 f 의 최댓값(absolute maimum)이라고 한다 D의 모든 점 에 대하여 f () f ( )이면 을 f 의 최소점이라 하고 f ( )을 f 의 최솟값(absolute minimum)이라고 한다

102 CHAPTER 함수와 미분 98 보기 의 함수 f () sin 는 lim f (), lim f () 이므로 실수 집합에서 최댓값과 최솟값이 존재하지 않는다 그러나 정의역을 구간 (, π)로 5π 제한하면 f ()는 π/에서 최솟값 f ( π ) π, 5π/에서 최댓값 f ( 5π ) + 을 갖는다 y + 5Π Π y - sinhl y Π Π Π 5Π Π 이렇게 a를 포함하는 열린구간에서 함수 f ()가 a에서 최댓값을 가지면 f ()는 a 에서 극대라고 하고 a를 극대점, f (a)를 극댓값(local maimum)이라고 한다 또 a 를 포함하는 열린구간에서 함수 f ()가 a 에서 최솟값을 가지면 f ()는 a에서 극소 라고 하고 a를 극소점, f (a)를 극솟값(local minimum)이라고 한다 극대점과 극소점을 통틀어 극점(etremal points), 극댓값과 극솟값을 통틀어 극값(etremum)이라고 한다 극댓값과 극솟값 함수 y f ()가 a를 포함하는 어떤 열린구간의 모든 에 대하여 f () f (a) ( 또는 f () f (a)) 이면 f 는 a에서 극댓값 (또는 극솟값) f (a)를 갖는다고 한다 Note 위의 정의는 다음과 같이 쓸 수도 있다 : 어떤 양수 h가 존재하여 a < h인 에 대하여 f () f (a)이면(f () f (a)이면) f 는 a에서 극댓값(극솟값) f (a)를 갖는다고 한다 Note 다음 그림에서 볼 수 있듯이 극댓값이 항상 극솟값보다 큰 것은 아니다 y y

103 평균값 정리와 극값 99 보기 의 함수 f () sin 의 그래프를 살펴 보면 극대점과 극소점에서 미분계수는 이다 이러한 사실은 미분가능한 모든 함수에 대하여 성립한다 보기 5 가지면 열린구간 (a, b)에서 미분가능한 함수 y f ()가 c (a < c < b)에서 극댓값을 f (c) 임을 보여라 Solution y f ()가 c (a < c < b)에서 극댓값을 가지면 어떤 양수 h에 대하여 구간 (c h, c + h)의 모든 에 대하여 f () f (c) 이다 따라서 (c, c + h)이면 f () f (c) c 이므로 f+ (c) lim+ c f () f (c) c 이다 마찬가지로 (c h, c)이면 c < 이므로 f () f (c) c 이 성립하고 f (c) lim c f () f (c) c 을 얻는다 f 는 c에서 미분가능하므로 f (c) f+ (c) 이고 f (c) 임을 알 수 있다 같은 방법으로 c에서 y f ()가 극소이면 f (c) 임을 보일 수 있다 극점에서의 미분계수 c에서 미분가능한 함수 y f ()가 c에서 극값을 가지면 f (c) 이다 Note 이 결과의 역은 일반적으로 참이 아니다 다시 말해서 미분가능한 함수 y f ()에 대하여 f (c) 이라고 해서 y f ()가 c에서 항상 극값을 갖는 것은 아니다 예를 들어, f () 에서 f () y

104 CHAPTER 함수와 미분 이므로 f () 이다 하지만 f 는 순증가하는 함수이므로 f 는 에서 극값을 갖지 않는다 Note 연속함수 f 가 c를 기준으로 f 의 부호가 바뀌면 f 는 c에서 극값을 갖는다 다시 말해서 f 의 부호가 +에서 로 바뀌면 극댓값을, 에서 +로 바뀌면 극솟값을 갖는다 다음 그림들에서 볼 수 있는 것처럼 함수 y f ()가 c에서 미분가능하지 않더라도 극값을 가질 수 있다 y y 임계점 함수 y f ()가 c에서 극값을 갖는다면 f (c)가 존재하지 않거나 또는 f (c) 이다 이러한 성질을 갖는 점 c를 임계점(critical point)이라고 한다 임계점 f (c)가 존재하지 않거나 f (c) 인 점 c를 임계점이라고 한다 기초다지기 다음 함수의 임계점을 모두 구하여라 (a) y + (b) y + Note 위의 논의들을 요약하면 c에서 y f ()가 극값을 가지면 c는 임계점이다 다시 말하면 (대우) c가 임계점이 아니면 y f ()는 c에서 극값을 갖지 않는다 따라서 극값을 구하기 위해서는 임계점만 살펴 보면 된다 연속인 함수 y f ()가 유일한 임계점을 갖는다고 하자 임계점 왼쪽에서는 증가하고 임계 점 오른쪽에서는 감소하면 임계점에서 최댓값을 갖는다 반대로 임계점 왼쪽에서는 감소하고 교재에 (a) 따라서는 정의역이 닫힌구간일 때 구간의 양 끝값을 임계점에 포함시키는 경우도 있다 ± (b) ±

105 평균값 정리와 극값 임계점 오른쪽에서는 증가하면 임계점에서 최솟값을 갖는다 함수 y f ()가 미분가능하다면 이 결과는 다음과 같이 쓸 수 있다 유일한 임계점 미분가능한 함수 y f ()가 유일한 임계점 c를 갖는다고 하자 < c일 때 f () > 이고 > c일 때 f () < 이면 y f ()는 c에서 최댓값 f (c)를 갖는다 < c일 때 f () < 이고 > c일 때 f () > 이면 y f ()는 c에서 최솟값 f (c)를 갖는다 최대 최소 정리에 의하여 연속인 함수 y f ()는 닫힌구간 [a, b]에서 최댓값, 최솟값을 갖는다 부분에서 최대(또는 최소)가 아닌 값, 즉, 극대가 아닌 값은 전체에서 최대(또는 최소) 가 될 수 없다 따라서 최댓값, 최솟값은 임계점, 또는 양 끝점에서 얻어진다 따라서 닫힌구간 [a, b]에서 연속함수 y f ()의 최대, 최소는 다음과 같이 구할 수 있다 최대, 최소 구하기 닫힌구간 [a, b]에서 연속함수 y f ()에 대하여 y f ()의 임계점에서 f 의 값을 모두 구한다 구간 [a, b]의 양 끝점에서의 값 f (a)와 f (b)를 구한다, 에서 구한 값 중에서 가장 큰 값이 최댓값, 가장 작은 값이 최솟값이 된다 Note 닫힌구간 [a, b]에서 함수 y f ()가 미분가능하면 은 다음과 같이 바꾸어 쓸 수 있다 a : f (c) 이 되는 c에 대하여 f (c)를 모두 구한다 보기 6 주어진 구간에서 보기 의 함수 f () sin 의 최댓값과 최솟값을 구하여라 (a) [, π] Solution (b) [, π] (a) 임계점은 f (c) π π, π 5π, 이고 π π, f 5π 5π +

106 CHAPTER 함수와 미분 이다 구간의 양 끝점, π에서 f (), 이므로 π 일 때 최솟값 f (b) 임계점은 π π π 이고 f f (π) π, π 5π 일 때 최댓값 f 5π 5π + 를 갖는다 π 이다 구간의 양 끝점, π에서 f (), 이므로 π 일 때 최솟값 f π π f (π) π, π일 때 최댓값 f (π) π를 갖는다 (c) 구간 내에 임계점은 존재하지 않는다 구간의 양 끝점 π, π 에 대하여 π π π π f, f + π π 이므로 π 에서 최솟값 f π π, π + 를 갖는다 에서 최댓값 f 기초다지기 구하여라 (a) [, ] 보기 7 함수 f () + 6에 대하여 다음 구간에서 최댓값과 최솟값을 (b) [, ] (c) [, ] (d) [, ] 다음 함수의 최댓값과 최솟값을 구하여라 (b) f () + (a) f (), ++ Solution (a) f 의 도함수를 구하면 f () ( )( + + ) ( + )( + ) ( + + ) ( ) ( + + ) 이고 분모는 항상 양수이므로 f () (a) ± 에서 최댓값 8, 에서 최솟값 9 (b) 에서 최댓값 8, 에서 최솟값 (c) 에서 최댓값, 에서 최솟값 9 (d) 에서 최댓값 6, 에서 최솟값 7

107 평균값 정리와 극값 이다 그러나 이므로 유일한 임계점 을 갖는다 임계점과 양 끝점에서 함수의 값을 구하면 f (), f (), f () 을 갖는다 이다 따라서 에서 최댓값, 에서 최솟값 y (b) 정의역이 언급되어 있지 않은 경우 정의역은 그 함수가 의미를 갖는 최대 집합을 의미한다 따라서 f 의 정의역은 이다 f 의 도함수를 구하면 p + ( ) 이므로 정의역 안에 두 개의 임계점 ± 를 갖는다 임계점에서의 함숫값은 f () f ( ), f ( ) 이고 양 끝점에서는 이므로 f () f ( ) 에서 최댓값 를, 에서 최솟값 를 갖는다 y - - -

108 CHAPTER 함수와 미분 연습문제 다음 함수의 임계점을 구하고 극값을 모두 구하여라 (a) f () + 6 (b) f () 9 (c) f () + (d) f () 삼차함수 f () a + b + c + d가 다음 조건을 만족할 때 상수 a, b, c, d를 구하여라 (a) 에서 극솟값 를 갖고 에서 극댓값 를 갖는다 (b) 에서 극댓값 를 갖고 에서 극솟값 를 갖는다 미분가능한 함수 f, g가 모두 증가함수이다 참이면 증명하고 거짓이면 반례를 들어라 (a) f + g는 증가함수이다 (b) f g는 증가함수이다 미분가능한 두 함수 f, g가 모든 에 대하여 f () 6, g () 6 이라고 하자 (a) f, g 모두 순증가함수이면 f g는 순증가함수임을 보여라 (b) f 는 순증가함수이고 g는 순감소함수이면 f g는 순감소함수임을 보여라 (c) f, g 모두 순감소함수이면 f g는 순증가함수임을 보여라 5 다음 부등식이 성립함을 보여라 ( ), R (b) sin, 6 (a) cos 6 다음 방정식의 근은 모두 몇 개 존재하는지 구하여라 (a) ( ) (b) 9 + (c) (d) 방정식 7 + k 의 근이 다음과 같을 때 k의 범위를 구하여라

109 평균값 정리와 극값 (a) 하나의 근 5 (b) 두 개의 근 (c) 세 개의 근 8 롤의 정리를 증명하여라( ) 9 미분가능한 두 함수 f, g가 모든 에 대하여 f () g ()이면 다음 식이 성립함을 보여라 f () g() + C (단, C는 상수) 모든 에 대하여 f () 6 이면 f (c) c인 c는 많아야 하나 존재함을 보여라( ) 두 함수 f ()와 g()가 닫힌 구간 [a, b]에서 연속이고 열린 구간 (a, b)에서 미분가능하다고 하자 그러면 f (c)(g(b) g(a)) g (c)(f (b) f (a)) 을 만족하는 점 c가 열린 구간 (a, b)에 존재함을 보여라 ( )5 미분가능한 함수 f : [, ] R 에 대하여 부등식 f () 이 성립한다고 한다 f ( ), f () 이면 f () 임을 보여라6 Hint 이 성질을 갖는 값이 두 개 있다면 모순이 생김을 평균값정리를 이용하여 보여라 결과를 코시의 평균값정리라고 한다 5 Hint h() (f (b) f (a))g() (g(b) g(a))f ()에 대하여 롤의 정리를 적용한다 6 Hint f () > 이거나 f () < 인 점이 존재하면 모순이 생김을 평균값정리를 이용하여 보인다 이

110 CHAPTER 함수와 미분 6 미분의 응용 최적화 문제 자연현상이나 경제활동에서의 최적화 문제는 비용을 최소화하거나 효율을 최대화하는 문제 등 을 포함한다 대부분의 문제에는 변수가 둘 이상이 주어지며 변수 사이의 관계식이 주어진다 이런 경우 최소화, 또는 최대화하고 싶은 함수를 하나의 변수로 나타내는 것이 우선되어야 한다 또한 주어진 관계식에서 변수가 취할 수 있는 범위도 구하여야 한다 보기 벽에 직사각형 모양의 울타리를 쳐서 임시 창고를 만들려고 한다 울타리의 길이가 5m일 때 창고의 넓이를 최대로 하려면 창고의 가로, 세로의 길이를 어떻게 해야 하는가? y y Solution 만족한다 가로의 길이를, 세로의 길이를 y라고 하자 그러면, y는 다음과 같은 조건을 + y 5,, y () 창고의 넓이는 A y 이므로 문제는 조건 ()를 만족하는, y 중에서 A y를 최대화하는, y를 구하는 것이 된다 조건 ()에서 5 y 이므로 y가 취할 수 있는 범위는 y 5 이고 A (5 y)y 5y y 이다 A의 임계점은 A 5 y y 5 이다 유일한 임계점에서 A 의 부호가 에서 +로 바뀌므로 y 에서 A는 최댓값 (m ) 8 을 갖는다 일반적으로 최적화 문제는 다음과 같은 과정을 거쳐서 해결한다

111 미분의 응용 7 최적화 문제의 해결 조건에 있는 미지수들을 이용하여 최적화하고 싶은 함수(목적함수)를 구한다 조건에 있는 미지수에 대한 조건을 구하고 이 미지수들 사이의 관계식을 구한다 주어진 조건식을 이용하여 목적함수를 일변수함수로 바꾸어 쓴다 목적함수의 임계점을 모두 구한다 5 임계점과 조건의 끝점에서 목적함수의 값을 모두 구한다 6 위에서 구한 값 중에서 가장 큰 값이 최댓값, 가장 작은 값이 최솟값이 된다 보기 점 (,)과 원 + y 사이의 거리를 구하여라7 Solution 원 위의 임의의 점 (, y)와 점 (, ) 사이의 거리는 p ( ) + (y ) 이다 두 점 사이의 거리의 제곱이 최소일 때 거리도 최소이므로 d ( ) + (y ) 의 최솟값을 구하기로 한다 여기서 (, y)는 단위원 위의 점이므로 다음과 같은 조건을 만족한 다 + y 또한, 점 (,)이 사분면의 점이므로, y 이고 p y 으로 쓸 수 있다 이 사실을 이용하여 d 을 의 함수로 나타내면 d ( ) + (y ) + + y y + p 6 이다 따라서 목적함수를 p f () 6 라고 하자 f ()의 임계점을 구하면 f () 5 이다 함수 f 의 증감을 살펴 보면 다음과 같다 7 일반적으로 한 점과 도형 사이의 거리는 한 점과 도형 위의 점 사이의 거리의 최솟값으로 정의한다

112 CHAPTER 함수와 미분 8 << & f f 극솟값 << + % 따라서 유일한 극솟값은 최솟값이 되고 이때 최솟값은 f 이다 따라서 점 (,) 과 원 + y 사이의 거리는 q 이다 한계함수와 평균함수 물량 개를 만드는데 드는 총비용함수가 C()로 주어졌을 때 에서의 한계비용은 C( + ) C() 로 정의한다 한계비용은 다음과 같은 근사식으로 계산할 수도 있다 C( + h) C() C( + ) C() C( + ) C() h h + C () lim 여기서 C ()를 한계비용함수(marginal cost function)라고 부른다8 한계비용함수는 물량 수 에서 하나를 더 만드는데 필요한 비용을 근사적으로 구하여 준다 보기 USB 를 개 만드는 비용함수가 다음과 같이 주어진다고 한다 (단위 : 천원) C() + 번째 USB 를 만드는데 드는 한계비용을 한계비용함수를 이용하여 구하여라 Solution 한계비용함수를 구하면 C () 이므로 C() C() ; C () 9 이다 따라서 번째 USB 를 만드는데 드는 한계비용은 약 9천원이다 Note 비용함수로부터 번째 USB 를 만드는데 드는 한계비용을 직접 계산해보면 C() C() ( + () ) ( + () ) 한계비용함수를 M C()로 나타내기도 한다

113 미분의 응용 9 으로 한계비용함수를 이용하여 구한 근사값이 상당히 정확함을 알 수 있다 한계수입함수, 한계이윤함수도 같은 방법으로 정의한다 즉, R(), P ()를 각각 물품 개를 판매하였을 때 총수입함수, 총이윤함수라고 하면 에서 한계수입(marginal cost)과 한계이 윤(marginal profit)은 각각 R( + ) R(), P ( + ) P () 으로 정의하고 한계수입함수(marginal revenue function)와 한계이윤함수(marginal profit function)는 각각 R (), P () 로 정의한다 에서의 한계수입은 한계수입함수를 이용하여 다음과 같이 근사적으로 구할 수 있다 R( + ) R() ; R () 보기 공연의 관람료를 p(단위 천원)로 했을 때 관람객의 수 는 다음과 같이 주어진다 5 5p (a) 한계수입함수를 구하여라 (b) 고정비용이 만원이고, 관람객 한 명에게 나누어주는 기념품의 비용이 5원이라고 할 때 한계이윤함수를 구하여라 (c) 총이윤이 최대가 되려면 관람료를 얼마로 책정해야 하는가? Solution (a) 관람료가 p일 때 총수입은 R p 이다 한계수입함수는 관람객의 수 에 대한 도함수이므로 R을 의 함수로 나타내기로 하자 p 5 이므로 R() 5 R () 5 이다 따라서 한계수입함수는 이다 (b) 총비용은 고정비용에 가변비용을 더한 것이므로 C() + 5

114 CHAPTER 함수와 미분 이다 따라서 한계이윤함수는 다음과 같다 P () R () C () (c) 한계이윤함수 P ()는 < 이면 P ( + ) P () ; P () > 이고 > 75이면 P () < 이다 따라서 총이윤 P ()는 < 75일 때 가 커지면 증가하는 반면 > 75일 때는 가 커지면 감소한다 즉, 75일 때 총이윤이 최대가 된다 p 5 이므로 총이윤이 최대가 되는 가격은 p 즉, 5만 5원이다9 평균비용함수 한계비용함수 C ()는 곡선 위의 점 (, C())에서 접선의 기울기이다 이 값은 개를 생산 한 후 하나를 더 생산할 때 드는 추가비용으로 해석된다 비용함수를 상품의 수 로 나누면 상품 하나를 생산하는데 필요한 평균비용이 나온다 이를 평균비용함수(average cost function) 라고 한다 C() AC() 개를 생산할 때의 평균비용은 원점과 곡선 위의 점 (, C())를 잇는 직선의 기울기가 된다 보기 5 휴대전화를 생산하는데 비용함수가 C() 이다(단위 : 만원) 평균비용이 최소가 되는 를 구하여라 Solution 평균비용함수 AC() AC () +5+ 를 + 미분하면 이고 > 이므로 이 유일한 임계점이다 < AC () AC() & 9 이 문제는 이차함수의 꼭짓점을 구하여 풀 수도 있다 > + 최소 %

115 미분의 응용 y y ACHL 7 y +5 5 따라서 일 때 평균비용은 최소가 되고 이때 평균비용은 AC() (만원) 이다 비용함수 C()가 미분가능하다고 하자 AC () C () C() 이므로 AC () C () C() AC() 이다 평균비용함수 AC()의 임계점에서 AC()(한계비용) C() (평균비용) 이다 일반적으로 한계비용이 평균비용보다 작으면 생산을 늘림으로써 평균비용을 낮출 수 있다 반대로 한계비용이 평균비용보다 크면 생산을 줄임으로써 평균비용을 낮출 수 있다 평 균비용은 한계비용함수 y C ()와 평균비용함수 y AC()가 만나는 교점에서 최솟값을 갖는다 y y ACHL y C HL

116 CHAPTER 함수와 미분 탄력성 가격의 변화에 수요가 어떻게 반응하는지는 경제학, 또는 경영학에서 아주 중요한 문제이다 버스요금이 원 오른 것과 택시비(기본요금)가 원 오른 것은 액수는 같지만 소비자의 입장에서는 큰 차이가 있다 현재 버스요금이 원, 택시 기본요금이 5원이라고 하면 버 스요금과 택시기본요금은 각각 %, % 5 오른 것이다 이렇게 변화량 p p p 을 초기값 p 으로 나눈 것을 상대변화율, 또는 백분 율 변화율(percent rate of change)이라고 한다 백분율 변화율은 어떤 단위를 사용하여도 단위가 상쇄된다 지하철요금이 % 오르면 승객의 수는 5% 감소하고 반면 택시요금이 % 오르면 승객은 % 감소한다고 하자 이때 지하철의 경우 요금을 인상하여도 총수입함수는 감소하지 않는다 요금 인상분이 승객 감소를 상쇄하고도 남기 때문이다 택시의 경우 요금 인상은 수요에 급 격한 감소를 야기하며 총수입도 크게 줄어든다 이런 경우 택시의 수요가 지하철의 수요보다 가격에 대하여 더 탄력적(elastic)이라고 말한다 일반적인 경우 가격에 대한 수요의 탄력성을 어떻게 정의하는지 살펴 보기로 하자 수요 의 탄력성(elasticity of demand)은 E로 나타내며 가격의 상대증가율 p/p에 대한 수요의 상대감소율 q/q의 비율로 정의한다 E q/q q p p/p p q 여기서 p는 가격을, q는 가격이 p일 때의 수요를 나타낸다 q가 p의 감소함수이면 E는 양수가 된다 여기서 가격의 변화 p는 가능한 한 작은 값으로 잡아 준다 따라서 근사식 q dq p dp 를 이용하여 다음과 같이 정의한다 수요의 탄력성 가격에 대한 수요의 탄력성 E는 다음과 같이 정의한다 E dq p dp q 현재의 지하철 요금을 p, 승객의 수를 라고 하면 요금 인상 후의 요금은 p, 승객의 수는 95 가 된다 요금 인상 전 총수입을 R p 라고 하면 요금 인상 후 총수입은 (p )(95 ) 5 p 5R 으로 오히려 5% 증가한다 이 경우 총수입은 % 줄어 든다 교재에 따라 탄력성을 부호 없이 정의하기도 한다

117 미분의 응용 Note 있다 수요함수를 q f (p)라고 하면 f (p) E dq dp 이므로 수요의 탄력성은 다음과 같이 쓸 수 p dq f (p)p q dp f (p) Note E > 이면 가격의 상대변화율보다 수요의 상대변화율이 더 큼을 의미하므로 수요는 탄력적(elastic)이라고 한다 E < 이면 수요는 비탄력적(inelastic)이라고 한다 보기 6 음료수의 수요함수가 q p로 주어져 있다 p, 5, 6일 때 수요탄 력성을 구하여라 Solution dq 이므로 수요의 탄력성은 dp E p dq p dp q p 이다 따라서 p, 5, 6일 때 수요탄력성은 각각 E, E, E 이다 이제 총수입이 최대가 되는 경우의 수요탄력성을 구하여 보기로 하자 가격이 p이고 수요가 q인 경우의 총수입은 R pq 이다 q가 p의 미분가능한 함수라면 dr dq p dq q+p q + q( E) dp dp q dp 이므로 dr dp E dr dr > 이므로 가격이 오르면 총수입이 증가한다 E > 이면 < 이므로 dp dp 가격이 오르면 총수입은 감소한다 따라서 E 일 때 총수입은 극대가 된다 E 이 되는 p 의 값이 유일하다면 E 일 때 총수입은 최대가 된다 이다 E < 이면 보기 7 게임기의 수요함수는 q 75 p (p의 단위 : 천원)로 주어진다 (a) p, 6일 때 가격에 대한 수요의 탄력성을 구하고 수요가 탄력적인지 판단하여라 (b) 수요가 탄력적인 가격의 범위를 구하여라 (c) 총수입이 최대가 되는 가격을 구하고 이때 총수입을 구하여라 Solution

118 CHAPTER 함수와 미분 (a) 가격에 대한 수요의 탄력성은 E dq p p p p dp q 75 p 75 p 이므로 p, 6일 때 수요의 탄력성은 각각, 이다 따라서 p 일 때는 비탄력적이고 p 6일 때는 탄력적이다 (b) E > 인 p의 범위를 구하면 된다 E p > 75 p p > 5 이므로 p > 5이다 (c) p 5일 때만 E 이므로 p 5일 때 총수입은 최대가 되고 총수입은 R 5(75 5 ) 5 즉, 억 5만원이다 일반적으로 변수 가 % 변할 때 y가 몇 % 변하는가를 나타내는 수치를 에 대한 y의 탄력성이라고 한다 가격의 변화에 따른 공급의 변화를 가격에 대한 공급의 탄력성이라고 한다 또한 소득에 대한 탄력성도 많이 다루는데 국민소득이 % 증가할 때 고용된 노동자 수의 변화를 소득에 대한 고용의 탄력성, 수입량의 변화를 소득에 대한 수입의 탄력성이라고 부른다 이러한 탄력성에 대한 문제는 연습문제에서 다루기로 한다

119 미분의 응용 5 연습문제 밀의 수확량 y는 토양에 있는 질소의 농도 N 에 의하여 다음과 같이 결정된다고 한다 y kn + N 밀의 수확량이 최대가 되는 질소의 농도를 구하여라 단, k > 은 상수이다 (a) 합이 인 두 수 중에서 곱이 최대가 되는 두 수를 구하여라 (b) 차가 인 두 수 중에서 곱이 최소가 되는 두 수를 구하여라 (c) 곱이 인 두 수 중에서 합이 최소가 되는 두 양수를 구하여라 (a) 같은 둘레를 갖는 직사각형 중에서 넓이가 최대가 되는 것은 정사각형임을 보여라 (b) 같은 넓이를 갖는 직사각형 중에서 둘레의 길이가 최소가 되는 것은 정사각형임을 보여라 주어진 점과 곡선 사이의 거리를 구하여라 (a) + y, (, ) (b) y, (c) y, (, ) (, ) 5 타원 모양의 체육관에 직사각형 모양의 운동장을 만들려고 한다 운동장의 넓이가 최대가 되는 가로와 세로의 길이를 구하여라 단 타원의 방정식은 y +, a b a>b> 이라고 한다 6 반지름이 인 원의 일부를 잘라내어 원뿔형의 일회용 컵을 만들었다 이 컵의 용량을 최대가 되게 하려면 얼마를 잘라내면 되는가? 이 때 최대부피를 구하여라 A C Θ B 7 5cm 들이 음료수용 깡통을 만들려고 한다 재료비가 가장 적게 들도록 하려면 어떻게 해야 하는가? 다시 말해서 깡통의 겉넓이가 가장 작아지게 하려면 어떻게 해야 하는가?( ) 곡선과 점의 거리는 곡선 위의 점과 주어진 점 사이의 최소 거리를 의미한다

120 CHAPTER 함수와 미분 6 r h 8 총비용함수 C()가 다음과 같을 때 한계비용함수 C ()와 평균비용함수 AC() 를 구하 여라 (a) C() + + (b) C() (c) C() 평균비용함수 AC()가 다음과 같을 때 총비용함수 C()와 한계비용함수를 구하여라 (a) AC() (c) AC() (b) AC() 가격이 p일 때 수요 와의 관계를 나타내는 수요함수가 다음과 같다 수입함수 R() 와 한계수입함수를 구하고 총수입이 최대가 되는 가격 p를 구하여라 (a) p (b) p 어떤 상품을 개당 (단위 : 만원)에 판매하고 생산된 상품은 모두 판매된다고 가정하자 개 생산할 때 총비용은 C() 이라고 한다 (a) 총이윤함수 P ()와 한계이윤함수 P ()를 구하여라 (b) 5개에서 하나를 더 팔았을 때 생기는 이윤을 미분계수를 이용하여 근사적으로 구하여라 (c) 몇 개를 팔았을 때 이윤이 최대가 되는가? PMP의 가격 p(단위 : 만원)와 일주일 동안의 판매량 q(단위 : 백대)의 관계는 다음과 같다 p + q

121 미분의 응용 7 일주일동안 q개의 PMP를 생산하는데 드는 총비용은 C(q) + 5q q + q 라고 한다 총이윤이 최대가 되도록 가격을 정하여라 이때 판매량은 얼마인가? 어떤 품목의 가격 p와 수요 q는 p + 5q 8을 만족한다 가격이 다음과 같을 때 수요의 탄력성을 구하여라 (a) p (b) p 수요함수가 pq C (C는 상수)일 때 (a) 수요탄력성은 항상 임을 보여라 (b) 이때 가격을 올리면 총수입은 어떻게 변화하는가? (c) p

122 CHAPTER 함수와 미분 역함수정리 연쇄법칙의 응용 공모양의 풍선에 바람을 불어 넣으면 시간이 지남에 따라 풍선의 반지름도 늘어나고, 표면적과 부피도 늘어난다 반지름과 표면적, 부피는 서로 관련이 되어 있다 다시 말해서, 하나는 다른 하나의 함수로 나타낼 수 있다 많은 경우 부피의 변화율을 구하기가 반지름이나 표면적의 변 화율을 구하는 것보다 훨씬 간단하다 따라서 부피의 변화율을 이용하여 다른 변화율을 구할 수 있다면 훨씬 효율적이 될 것이다 이렇게 연관된 두 변수의 함수 관계와 한 변수의 변화율을 이용하여 다른 변수의 변화율을 구하여 보기로 한다 보기 구 모양의 대형풍선에 분당 L의 물을 넣고 있다 풍선은 표면적이 πm 이하 에서는 안전하지만 이보다 더 넓어지면 파열될 위험성이 있다고 한다 표면적이 πm 일 때 시간에 대한 반지름의 변화율과 표면적의 변화율을 구하여라 Solution 반지름이 r이면 표면적과 부피는 각각 S πr, V πr 이다 표면적이 π(m )일 때 반지름은 r (m)이고 부피의 시간에 따른 변화율은 dv (m /min) dt 이다 따라서 부피의 식에서 양변을 t에 대하여 미분하면 dv dr dr π r πr dt dt dt 이다 따라서 r 일 때 π dr dt dr ; dt π 이므로 반지름의 변화율은 dr ; (m/min) (cm/min) dt 이다 표면적의 식을 t에 대하여 미분하면 ds dr π r dt dt 이고 r 일 때 dr 임을 이용하면 표면적의 변화율은 dt ds 8π ; 8(m /min) dt 이다 m L

123 5 역함수정리 9 Note 위의 보기에서 볼 수 있듯이 어떤 단위를 사용하느냐에 따라 다른 답을 얻을 수 있다 그러나 단위를 환산하면 같은 답을 얻는다 이런 종류의 문제에서 사용하는 단위를 일관되게 사용하도록 주의하여야 한다 보기 다음과 같이 아이스크림콘 모양(원뿔)의 모래탱크에서 초당 L의 모래가 빠져 나가고 있다 모래의 높이가 5m일 때 높이의 변화율을 구하여라 또, 높이가 m일 때 높이의 변화율은 5m일 때의 몇 배가 되는가? 6 r h Solution 모래탱크에 저장된 모래의 양을 V 라고 하면 dv dt ( m /sec ) () 이다 높이가 h일 때 표면의 반지름을 r이라고 하면 직각삼각형의 닮음비에서 h : r : 6 r h 5 을 얻는다 따라서 높이가 h일 때 모래의 부피는 V 9 πr h π h h πh (m ) 5 5 이고 연쇄법칙에 의하여 부피의 변화율은 다음과 같다 dh 9 dh dv π h πh dt 5 dt 5 dt () h 5일 때 식 ()과 ()에서 9 dh dh π5 9π 5 dt dt dh ; 6 dt π 을 얻는다 여기서 부호는 높이가 낮아짐을 의미한다 또한, 높이가 m일 때 높이의 변화율은 9 dh dh π 9π 5 dt 5 dt 5 이 되므로 5m일 때 변화율의 65배가 된다 dh 5 dt π

124 CHAPTER 함수와 미분 5 음함수미분법 함수가 y, y log 와 같이 y f ()꼴로 쓰여졌을 때 양함수 꼴로 표현되었다(defined eplicitly)고 한다 그러나 + y 5, + y 6y (5) 와 같이 함수의 관계식이 명확히 주어지지 않는 경우 음함수 꼴로 표현되었다(defined implicitly)고 한다 함수가 음함수 꼴로 주어진 경우 표현된 식을 (국소적으로) y에 대하여 풀어 y f () 와 같이 양함수 꼴로 나타낼 수 있는 경우가 있다 예를 들어 + y 5을 y에 대하여 풀면 p y ± 5 이 된다 따라서 + y 5을 양함수 꼴로 바꾸어 쓰면 다음과 같이 두 개의 함수로 표현이 된다 p p f () 5, f () 5 y 5 y y fhl y fhl 일반적으로 음함수 꼴로 주어진 그래프는 하나의 함수의 그래프가 아니라 여러 함수의 그래프의 합집합이다 보기 원 + y 5 위의 점 (, )에서 접선의 기울기를 구하여라 Solution 접선의 기울기를 두 가지 방법으로 구해 보기로 하자 방법 점 (, )근방에서 양함수 꼴로 고쳐 쓰면 p y 5 (5 )/ 이다 f () /, g() 5 으로 쓰면 y f (g())이고 y f (g())g () 5

125 5 역함수정리 이 된다 따라서 점 (, )에서 접선의 기울기는 f () 이다 방법 합성함수의 미분법을 이용하여 + y 5의 양변을 에 대하여 미분하면 d d ( + y ) 5 d d + y dy d 이다 따라서 점 (, )에서 dy d y 이다 y 5 + y y 5 5 보기 의 두번째 풀이처럼 음함수를 양함수 꼴로 바꾸지 않고 미분계수를 구하는 것을 음 함수미분법(implicit differentiation)이라고 한다 이 방법을 이용하면 음함수 꼴로 주어진 함수를 양함수 꼴로 바꾸지 않아도 주어진 점에서 미분계수를 구할 수 있다 음함수미분법 음함수의 양변을 에 대하여 미분한다 이때 y는 의 함수로 간주한다 dy/d를 포함하는 항을 방정식의 한 변으로 정리하여 dy/d에 대하여 푼다 기초다지기 5 (a) 음함수 미분법으로 주어진 점에서 접선의 기울기를 구하여라5 (b) (c) (d)

126 CHAPTER 함수와 미분 (a) + y, (, ) (c) (b) y, (, ) (d) 유리수 r에 대하여 y r 이라고 하자 r m 제곱하면 + y, (9, ) +, (, ) y n 인 서로 소인 정수 m, n이 존재한다 양변을 m y m (r )m n 가 되고 양변을 에 대하여 연쇄법칙으로 미분하면 my m y nn 이다 y n n n m y rn ( m) m m 에서 의 지수를 계산하면 n + ( m) n n n + nr m m 이므로 y rr 이 성립한다6 y r 의 도함수 임의의 유리수 r에 대하여 다음 식이 성립한다 (r ) rr 보기 다음 함수의 도함수를 구하여라 (a) f () Solution (a) f () (b) f () + / 이므로 f () / 이다 6 실제로는 임의의 실수 r에 대하여 이 결과가 성립하다 증명은 5절 참조할 것

127 5 역함수정리 (b) f () ( + ) / 이므로 + f () ( + ) / ( + ) ( + ) / ( + ) + 이다 기초다지기 (a) y 다음 함수의 도함수를 구하여라7 (b) y (c) y ( + ) (d) y ( + ) 역함수 정리 5 기울기가 이 아닌 모든 일차함수 y f () a + b (a 6 ) 의 역함수는 y f () 이므로 f 의 기울기 a와 f 의 기울기 b a a 은 항상 역수이다 a f 와 g f 의 기울기에 대한 역수 관계는 선형함수가 아닌 경우에도 역시 성립한다8 그 러나 이 경우 어떤 점에서의 기울기를 고려해야 하는지를 신중히 판단하여야 한다 점 (a, f (a)) 에서 y f ()의 기울기가 f (a) 6 이라면 점 (f (a), a)에서 y g()의 기울기는 /f (a)이다 따라서 b f (a)로 놓으면 a g(b)이고 g (b) f (a) f (g(b)) 이다 y y f HL a y f -HL b b 7 (a) a 5 y (b) y (c) y ( + ) (d) y ( + ) 점에서 곡선의 기울기는 그 점에서 접선의 기울기로 정의됨을 기억하자 8 어느

128 CHAPTER 함수와 미분 역함수 정리 구간에서 정의된 미분가능한 함수 y f ()가 정의역의 모든 점에서 f () 6 이면, f 의 역함수 g f 가 존재하고 미분가능하다 b f (a)이면 다음 관계식이 성립한다 g (b) f (a) f (g(b)) (6) Note 역함수 정리의 결론은 라이프니츠의 표기법을 이용하면 다음과 같이 쓸 수 있다 d dy dy/d Note 역함수가 존재하고 미분가능하다는 것을 안다면 (6)은 음함수의 미분법으로 쉽게 보일 수 있다 즉, g(f ()) 이므로 양변을 에 대하여 미분하면 g (f ())f () 이다 b f (a)이면 a g(b)이므로 g (b) f (g(b)) 이다 보기 5 Solution 함수 f () + 의 역함수 g가 존재함을 설명하고, 미분계수 g ()를 구하여라 임의의 실수 에 대하여 f () + > 이므로 역함수 정리에 의하여 역함수 g가 존재한다 f (a) a + a 이면 a 이므로 g() 이다 따라서 g () 이다 f (g()) f () 6

129 5 역함수정리 5 y y f HL y f -HL 제품의 가격이 p일 때 수요함수 q f (p)가 순감소하는 함수이면 역함수 g f 가 존재한 다 p g(q) q f (p) 보기 6 (a) 수요의 탄력성 E를 수요함수 q f (p)의 역함수 p g(q)를 이용하여 q의 함수로 나타내어라 (b) 한계수입함수 R (q)를 탄력성 E와 가격 p를 이용하여 나타내어라 Solution dq (a) E pq dp 이고 역함수정리에 의하여 dq dp dp/dq g (q) 이므로 수요의 탄력성을 q의 함수로 나타내면 다음과 같다 E g(q) qg (q) (7) (b) R(q) pq qg(q)이므로9 R (q) g(q) + qg (q) 이고 식 (7)에서 qg (q) g(q) p E E 이므로 한계수입함수 R (q)와 탄력성 E에 대한 다음 관계식을 얻는다 p R (q) p p E E 9 따라서 R(q) g(q) q 이다 이런 이유로 수요함수의 역함수 g를 평균수입함수(average revenue function) 라고 부르기도 한다 이 관계식을 한계수입에 대한 Lerner의 공식이라고 한다

130 CHAPTER 함수와 미분 6 연습문제 5 공 모양의 풍선에 공기를 5cm /sec의 비율로 불어 넣고 있다 (a) 풍선의 반지름이 5cm일 때 반지름의 시간에 대한 증가율을 구하여라 (b) 풍선의 반지름이 5cm일 때 표면적의 시간에 대한 증가율을 구하여라 반지름이 5cm인 원기둥 모양의 물탱크에서 분당 L의 물이 바닥의 구멍을 통하여 빠져 나오고 있다 높이가 5cm일 때 높이의 변화율은 얼마인가? (단위에 주의할 것 : m L) 같은 지점에서 두 대의 차가 출발하였다 한 대는 북쪽으로 km/min의 속도로 움직이고 다른 한 대는 서쪽으로 75km/min의 속도로 움직인다면 5분 후 두 차가 멀어지는 속도는 얼마인가? 배가 섬 근처를 직선항로를 따라 운항하고 있다 이 섬 선착장에 배가 가장 가깝게 접근한 거리는 6m이고 이 배는 초당 m 의 속도로 동쪽으로 움직인다 (a) 배가 선착장에서 멀어지기 시작한 지 분 후 배가 선착장에서 멀어지는 속도를 구하 여라 (b) 배에서 선착장까지의 거리가 m가 되었을 때 배가 선착장에서 멀어지는 속도를 구하여라 5 바람없는 날 열기구가 수직으로 상승하고 있다 출발지점에서 m 떨어진 지점에서 열 π 기구를 올려다 본 각도가 일 때 각도의 증가속도는 rad/min이었다 이때 열기구의 상승속도는 얼마인가? 6 수요함수가 다음과 같이 주어져 있다 p(q + ) 9 가격이 (단위 만원)일 때 수요량의 가격에 대한 변화율 7 음함수 미분법을 이용하여 (a) + y dy d 를 dq dp 를 구하여라 구하여라 (b) y + y 8 (a) 음함수의 미분법을 이용하여 타원 a (c) y +y + yb 위의 점 (, y )에서 접선의 방정식은 y y + a b 임을 보여라 (b) 음함수의 미분법을 이용하여 쌍곡선 a yb 위의 점 (, y )에서 접선의 방정 식은 y y a b 임을 보여라

131 5 역함수정리 7 9 양의 실수 k에 대하여 다음과 같이 주어진 두 곡선이 직교함을 보여라 ( ) + y k, y 다음과 같이 정의되는 곡선을 아스트로이드(astroid)라고 부른다 / + y / (a) 곡선 위의 점 (a, b )에서 접선의 방정식을 구하여라 (b) 사분면에 있는 곡선 위의 점 (a, b )에서 접선을 그렸을 때, 사분면에 있는 접선 부분의 길이는 항상 이 됨을 보여라 y - - 함수 y f () 위의 점 (a, b)에서의 접선의 방정식이 y 이다 y f ()위의 점 (b, a) 에서 접선의 방정식을 구하여라 함수 f () + 5의 역함수 g f 가 존재함을 보이고 g ( )을 구하여라

132 CHAPTER 함수와 미분 8 지수, 로그함수의 도함수 6 6 자연상수 지수함수 f () a 의 도함수를 정의를 이용하여 계산하면 a+h a a (ah ) lim h h h h h a a lim h h f () lim 을 얻는다 따라서 f () a 가 에서 미분가능하면, 다시 말해서 ah La h h f () lim 가 존재하면 모든 에 대하여 f ()가 존재하고 (a ) La a 가 된다 극한값 La 는 그래프 f () a 의 에서의 기울기이다 a가 커지면 La 의 값도 커진다 y y 5 y - a, 5, 인 경우 f ()의 근삿값을 구해 보면 h ; 69, h h L lim 5h ; 9, h h L5 lim 에 가까운 h에 대하여 근삿값을 구하여 보면 다음과 같다 h h h h h h h h ; h h L lim

133 6 지수, 로그함수의 도함수 9 이다 이 식으로부터 5 < a < 인 값 중에서 La f () 이 되는 수가 존재함을 알 수 있는데 이 수를 문자 e로 나타내고 자연상수라고 부른다 이렇게 정의된 자연상수는 장 절에서 정의된 자연상수와 일치한다 자연상수 다음 식을 만족하는 실수를 자연상수 e라고 한다 eh h h lim 6 지수함수의 도함수 정의로부터 자연상수 e를 밑으로 갖는 지수함수 f () e 의 도함수는 자기 자신과 일치한다 f () (e ) e f () 따라서 합성함수의 미분법에 의하면 다음 식을 얻는다 d g() (e ) f (g())g () eg() g () d 특히 상수 k에 대하여 g() k이면 d k (e ) ek (k) kek d (8) 가 성립한다 바꾸어 말하면 h() ek 의 변화율을 나타내는 도함수 h 은 h의 k배이다 h () kh() 자연로그함수는 지수함수 y e 의 역함수이므로 다음 등식이 성립한다 log e, elog 여기서 등식 elog 는 일반적인 함수를 지수함수로 변형시켜주는 유용한 공식이다 예를 들어 밑이 a인 지수함수는 다음과 같이 나타낼 수 있다 y a (elog a ) e log a 따라서 식 (8)로부터 (a ) (e log a ) (log a)e log a a log a

134 CHAPTER 함수와 미분 을 얻는다 지수함수의 도함수 d k (e ) kek, d 보기 d (a ) a log a,, d d g() (e ) eg() g () d 다음 함수의 도함수를 구하여라 (a) y (b) y esin (c) y e (d) y e Solution (a) e log e( log ) ( 이므로 log ) ( log ) e log 이다 (b) (esin ) (sin ) esin (cos )esin (c) 함수의 곱에 대한 미분법칙을 적용하면 다음 결과를 얻는다 (e ) () e + (e ) e + e ( + )e (d) 함수의 몫에 대한 미분법칙을 적용하면 e (e ) e () ( )e 이다 기초다지기 다음 함수의 도함수를 구하여라 (b) y e (e) y ( e) (c) y e (f) y (a) y (a) (g) y e (h) y ( )e (i) y (h) y e y log (b) y log (c) y e (g) y e (d) y (i) y (d) y e (+)e (+) (e) y e + e (f) y log

135 6 지수, 로그함수의 도함수 보기 f () e 에 대하여 (a) 에서 접선의 방정식을 구하여라 (b) 이를 이용하여 e 과 e 의 근삿값을 구하여라 (c) 임의의 실수 에 대하여 e + 임을 보여라 Solution (a) f () e 이므로 f () 이다 따라서 에서 f () e 의 접선의 방정식은 y f () + f () + 이다 y - (b) 접선의 값을 함숫값의 근삿값으로 사용하면 e f () ; +, e f ( ) ; 9 이다 (c) g() e 라고 하자 > 이면 g () e 이므로 g는 증가함수이고 g() 이므로 g() g() 이다 < 이면 g () e < 이므로 g는 감소함수이다 따라서 < 이면 g() g() 이다 따라서 모든 에 대하여 e + 이다 보기 실제값을 함수 f () e 의 최댓값, 또는 최솟값이 존재하면 구하여라 소수점 이하 9자리까지 구하면 e 5998, e 9878이다

136 CHAPTER 함수와 미분 Solution 곱의 미분법을 적용하면 f () e + (e ) ( )e 이므로 은 유일한 임계점이다 < 이면 f () >, > 이면 f () < 이므로 에서 극댓값 f () e 를 가지며 이 값은 최댓값이 된다 lim f () 이므로 최솟값은 존재하지 않는다 최적보유기간 t년 후 가격이 V (t)( )가 되는 자산을 갖고 있다고 하자 이자는 연 r%의 연속복리라면 t년 후 이 자산의 현재가치는 F (t) V (t)e rt 이다 V (t)가 미분가능하면 F (t) (V (t)e rt ) V (t)e rt rv (t)e rt 이므로 F (t) V (t) r V (t) (9) 을 만족하는 t의 값에서 극값을 갖는다 식 (9)을 만족하는 t의 값이 t t 으로 유일하다고 하자 t < t 일 때 V (t) >r V (t) 이고 t > t 일 때 V (t) <r V (t) 이면 F 는 t t 에서 최댓값을 갖는다 이때, t t 을 최적보유기간 (optimal holding time) V (t) 이라고 한다 식 (9)에서 은 자산의 백분율변화율을, 오른쪽 값은 은행의 이율 r을 나 V (t) 타낸다 자산의 백분율변화율이 은행의 이율보다 높을 때에는 자산을 보유하는 것이, 은행이 이율이 더 높을 때는 자산을 처분하는 것이 더 높은 수익을 낸다 보기 t년 후 가치가 V (t) + 5t인 자산이 있다 연이율은 연속복리로 %일 때 최적 보유기간을 구하여라 Solution V (t) 5이므로 5 V (t) V (t) + 5t 을 풀면 + t 5 t5

137 6 지수, 로그함수의 도함수 이다 t < 5이면 V (t) 5 > V (t) + 5t 이고 t > 5이면 5 V (t) < V (t) + 5t 이므로 5년 후 자산을 처분하면 수익을 최대화할 수 있다 6 로그함수의 도함수 y log 라고 하면 ey 이다 양변을 에 대하여 미분하면 dy d y (e ) ey d d 을 얻고 양변을 ey 로 나누면 dy y d e 을 얻는다 또한, loga log 이므로 log a d loga d log a 이다 합성함수 y log g() f (g())에 대하여 연쇄법칙을 적용하면 f (u) 이므로 u dy g () f (g())g () d g() 이 성립한다 로그함수의 도함수 d (log ), d 보기 5 d (loga ), d log a d g () (log g()) d g() 다음 함수의 도함수를 구하여라 (a) y log( + ) (b) y log(log ) (c) y log Solution (a) 합성함수의 미분법을 적용하면 다음을 얻는다 y ( + ) + + (d) y log

138 CHAPTER 함수와 미분 (b) f () log 일 때 y log f ()이므로 (log f ()) f () / f () log log 이다 (c) 함수의 곱의 미분법칙을 적용하면 ( log ) () log + (log ) log + log + 이다 (d) 함수의 몫의 미분법칙을 적용하면 log (log ) log () (/) log log 이다 기초다지기 다음 함수의 도함수를 구하여라 (g) y log + (a) y log (d) y log( + ) (b) y log (e) y log( + ) (c) y log (f) y ( + ) log( + ) 보기 6 (h) y log( + ) + 함수 f () log( + )에 대하여 (a) 에서 접선의 방정식을 구하여라 (b) (a)의 결과를 이용하여 log 9, log 의 근삿값을 구하여라 (c) > 이면 다음 부등식이 성립함을 보여라 log( + ) Solution (a) f () 이므로 f () 이다 f () 이므로 접선의 방정식은 다음과 같다 + y f () + f () (a) log ( +) y (g) y (b) y log (c) y (h) y log(+) (+) log (d) y + (e) y + + (f) y log(+)+

139 6 지수, 로그함수의 도함수 5 (b) 접선의 값을 함수의 근삿값으로 사용하면 log 9 log( + ( )) ;, log log( + ) ; 이다5 (c) g() log( + )라고 하자 g () + 에서 > 이면 g > 이므로 g는 순증가, < < 이면 g < 이므로 g는 순감소함수 이다 따라서 > 이면 g() > g(), < < 이면 g() > g() 이다 Note 보기 6(c)의 방법으로 > 이면 log( + ) > y 임을 보일 수 있다 (연습문제) y y y loghl y loghl - - 보기 y - 함수 y log 의 최댓값, 또는 최솟값이 존재하면 구하여라 Solution 곱의 미분법을 이용하면 f () log + (log ) log + log + 이다 따라서 e 는 유일한 임계점이다 < e 이면 f () <, > e 이면 f () > 이므로 e 에서 극솟값을 가지며 이 값은 최솟값이 된다 lim log 이므로 최댓값은 존재하지 않는다 5 log 9 ; 5656, log ; 958

140 CHAPTER 함수와 미분 6 y log 는 6 인 실수에 대하여 정의된다 ( log, log log( ), > < 이므로 > 이면 log log 이고 < 이면 log log( ) 을 얻는다 이 결과는 따로 기억할 만한 가치가 있는 공식이다6 (log ) 6 로그미분법 곱, 몫, 또는 지수에 대하여 로그를 취하면 각각 합, 차, 곱으로 변환된다 이러한 로그함수의 성 질을 이용하면 복잡한 함수의 도함수도 간단하게 구할 수 있다 예를 들어, y 일 때 양변에 로그를 취하면 log y log 이고 양변을 에 대하여 미분하면 y log + (log ) log + y y (log + ) 이다 이렇게 양변에 로그를 취하여 도함수를 구하는 방법을 로그미분법이라고 한다 보기 8 r이 임의의 실수일 때 다음 식이 성립함을 보여라 (r ) rr Solution y r 에서 양변에 로그를 취하면 log y r log 이고 양변을 에 대하여 미분하면 y r y 을 얻는다 6 적분을 참고할 것 y ry rr

141 6 지수, 로그함수의 도함수 7 거듭제곱의 미분법칙 임의의 실수 r에 대하여 다음 식이 성립한다 (r ) rr, 보기 9 > 수요함수가 f (p) kp r (k, r > )로 주어진다고 하자 탄력성을 구하면 E f (p)p ( r)kp r p r f (p) kp r 으로 항상 상수이다 수요함수의 양변에 로그를 취하면 log log k r log p 이다 따라서 로그좌표로 나타내면 즉, 수평축을 log p, 수직축을 log 로 잡으면 수요함수는 기울기가 r인 선형함수로 나타낼 수 있다 loghl logh pl p 보기 휴대전화 생산공장의 콥-더글러스 생산함수는 P 5 75 y 로 주어진다 여기서 P 는 한 달 동안 만들어지는 휴대전화의 수, 는 노동자의 수, y는 (인건 dy 비를 제외한) 한 달 비용이다 한 달에 대의 휴대전화가 만들어진다고 할 때 를 구하여라 d Solution 생산함수는 양변에 로그를 취하면 선형함수 꼴이 되어 계산이 간단해진다 P 이므로 log log + 5 log + 75 log y 이고(C는 상수) 양변을 에 대하여 미분하면 dy + y d dy y d C log + log y

142 CHAPTER 함수와 미분 8 이다 Note 위의 보기에서 6이면 65 y 75 y ( )/ 8 이다 따라서 dy 8 ; 8 d 6 이다 이는 노동자의 수를 6명에서 한 명 더 늘리면 한 달 동안 대의 휴대전화를 만드는데 드는 한달 비용은 약 8원 만큼 감소함을 의미한다

143 6 지수, 로그함수의 도함수 9 연습문제 6 dy 를 구하여라 d (a) y e (b) y e (f) y log + (g) y (log( + e )) (h) y log( + ) (c) y ee e (d) y + e (e) y (log ) (i) y log sec (j) y log sec + tan 로그미분법을 이용하여 다음 함수의 도함수를 구하여라 (a) y / (b) y log (e) y ( + )/ (c) y (log ) p (d) y ( + ) 다음 함수의 도함수를 구하여라 (a) y (b) y (log )(log ) 음함수 미분법을 이용하여 (a) ey + y (c) y log ( + e ) dy 를 구하여라 d r (b) y log( + y ) (c) y 5 다음 함수의 극댓값 또는 극솟값을 모두 구하여라 (a) y e (e) y log (b) y e (c) y e (d) y e + e (f) y log (g) y log( + ) 6 에서 다음 부등식이 성립함을 보여라 (a) + + e ( ) 7 다음 부등식이 성립함을 보여라 (a), y 이면 e ey y (b), y 이면 log log y y (b) log( + ) +

144 CHAPTER 함수와 미분 8 방정식 log + a은 몇 개의 실근을 갖는지 판단하여라 9 도함수가 자기자신의 상수배가 되는 함수는 지수함수 뿐임을 보여라 즉, 함수 y f () 가 y ky을 만족하면 f () Cek 임을 보여라 온도가 y 인 물체를 온도가 A인 공간에 놓았을 때 시간 t에서의 물체의 온도 y f (t)는 뉴턴의 냉각법칙에 의하면 다음과 같이 주어진다 y f (t) A + (y A)e kt, k> y 을 y를 이용하여 나타내고 이 식이 의미하는 바를 설명하여라( ) 포도주의 가치는 시간이 지남에 따라 다음 함수에 따라 증가한다 (단위 : 만원) v(t) + e t 연이율 5%의 연속복리인 경우 최적보유기간을 구하여라 LTE 가입자가 현재 (단위 : 만명)이라고 한다 t개월 후 가입자의 수를 y(t)라고 할 때 y(t)는 다음 방정식을 만족한다7 dy y( y) dt (a) 가입자 수가 가장 빨리 증가할 때의 가입자 수는 얼마인가? (b) 함수 f (t) 는 주어진 식을 만족함을 보여라 + 9e t (c) 가입자 수가 5 만이 되는 것은 약 몇 개월 후인가? (d) y 을 y를 이용하여 나타내어라 7 로지스틱 방정식(logistic equation)이라고 한다

145 7 함수의 그래프 7 함수의 그래프 다음 두 그래프는 모두 증가하는 함수의 그래프이지만 다른 특징을 갖는다 7 함수의 볼록성 주어진 구간에서 첫번째 그래프는 위로 볼록(concave down)하다고 하고 두번째 그래프는 아래로 볼록(concave up)하다고 한다 곡선 위의 임의의 두 점을 연결하였을 때 위로 볼록 한 경우는 곡선의 그래프가 직선의 그래프보다 위에, 아래로 볼록한 경우는 곡선의 그래프가 직선의 그래프보다 아래에 있다 이 사실을 식으로 나타내면 다음과 같다 : 함수 f 가 구간 I의 임의의 두 점 a, b와 임의의 t [, ] 에 대하여 f (( t)a + tb) ( t)f (a) + tf (b) 이 성립하면 함수 f 는 구간 I에서 아래로 볼록하다 y f HbL H - tl f HaL + tf HbL f Ha H - tl + tbl f HaL a a H - tl + tb b f ()가 미분가능한 함수라면 첫번째 경우(위로 볼록)는 가 커지면 접선의 기울기가 점점 작아 진다 다시 말해서 f ()가 감소한다 두번째 경우(아래로 볼록)는 접선의 기울기 f ()가 점점 커진다 이러한 성질을 미분가능한 함수 y f ()의 볼록성을 정의하는 데에 사용한다

146 CHAPTER 함수와 미분 함수의 볼록성 미분가능한 함수 y f ()가 구간 (a, b)에서 f 이 증가하면 y f ()는 아래로 볼록이다 구간 (a, b)에서 f 이 감소하면 y f ()는 위로 볼록이다 만약 y f ()의 이계도함수가 존재하면 위의 결과는 다음과 같이 쓸 수 있다 이계도함수와 볼록성 두번 미분가능한 함수 y f ()가 구간 (a, b)에서 f > 이면 y f ()는 아래로 볼록이다 구간 (a, b)에서 f < 이면 y f ()는 위로 볼록이다 보기 함수 f () 9 + 5에 대하여 위로 볼록인 구간과 아래로 볼록인 구간을 구하여라 Solution f () ( )이므로 f () 6( ) 이다 따라서 > 이면 f () > 이고 아래로 볼록이다 < 이면 f () < 이고 위로 볼록이다 기초다지기 8 (a) 다음 함수에 대하여 보기 의 질문에 답하여라8 위로 볼록 <, 아래로 볼록 > (b) 위로 볼록 <, 아래로 볼록 > (c) 아래로 볼록 >

147 7 함수의 그래프 (a) f () 5 5 (b) f () e (c) f () log 임계점에서 함수가 위로 볼록하면 극댓값을 갖는다 또한 임계점에서 함수가 아래로 볼록 하면 극솟값을 갖는다 이 사실은 f 가 연속인 경우 다음과 같이 바꾸어 쓸 수 있다 이계도함수 판정법 f (c) 이고 f 가 연속일 때 f (c) > 이면 f 는 c에서 극솟값을 갖는다 f (c) < 이면 f 는 c에서 극댓값을 갖는다 보기 Solution 이계도함수 판정법으로 함수 f () f () 의 극값을 구하여라 + ( + ) () 이므로 두 개의 임계점, 을 갖는 ( + ) ( + ) 다 f () ( )() ( ) + ( ) ( + ) ( + ) ( + ) 이므로 f () <, f ( ) > 이고 에서는 극댓값 f () 을 에서는 극솟값 f ( ) 을 갖는다 y - - 기초다지기 다음 함수에 대하여 보기 의 질문에 답하여라9 (a) f () (a) (b) f () e (c) f () log 에서 극솟값, 에서 극댓값 (b) 에서 극댓값 e (c) 에서 e 극솟값 e

148 CHAPTER 함수와 미분 7 변곡점 보기 에서 의 왼쪽에서 곡선은 위로 볼록하다가 오른쪽에서는 아래로 볼록한 모양이 된다 이렇게 곡선의 볼록한 모양이 바뀌는 점을 변곡점(inflection point)이라고 한다 y - - 변곡점 함수 y f () 위의 점의 좌우에서 그래프의 모양이 아래로 볼록에서 위로 볼록으로 바뀌거 나, 반대로 위로 볼록에서 아래로 볼록으로 바뀌면 이 점을 함수 y f ()의 변곡점이라고 한다 기초다지기 다음 함수의 변곡점을 구하여라5 (b) f () e (a) f () 5 5 (c) f () log 보기 새로 개설된 인터넷 사이트의 가입자 수(단위 : 만명)는 시간 t(단위 : 개월) 에 대하여 다음과 같이 주어진다고 한다 t f (t), t 7 + t y f (t)의 변곡점을 구하고 변곡점 근처에서 어떤 현상이 일어나는지 설명하여라 Solution f (t) t 5 로 쓰면 7 + t 7 + t 8t f (t) > (7 + t ) 이므로 회원 수는 순증가한다 곱의 미분공식을 이용하면 f (t) 5 (a) (b) 8 t (9 t ) (7 + t ) (7 + t ) (7 + t ) (c) 변곡점 없음

149 7 함수의 그래프 5 이고 t > 이므로 t 에서 유일한 변곡점을 갖는다 t < 이면 f (t) > 이고 t > 이면 f (t) < 이므로 f (t)는 t 에서 최댓값을 갖는다 다시 말해서 인터넷 사이트를 개설하고 개월 후에 신규가입자수 f (t)는 최대가 된다 개월이 지나고 나면 총가입자 수 f (t)는 계속 늘지만 신규가입자 수 f (t)는 줄어들기 시작한다 y y f HtL 5 5 y f HtL t 그래프 그리기 지금까지 배운 내용들을 이용하여 함수의 그래프를 그려 보기로 하자 함수의 그래프를 그리기 위해서는 우선 의 범위를 결정하여야 한다 일반적으로 함수 y f ()의 정의역은 특별히 주어지지 않은 경우 f ()가 의미를 갖는 최대집합이다 정의역이 무한한 경우 유한한 평면에 정의역을 모두 나타내는 것은 불가능하다 따라서 의 범위를 제한하여야 하는데 y f ()에 대한 정보를 갖고 있는 모든 점을 포함하여야 한다일반적으로 함수의 그래프를 그릴 때 다음과 같은 것들이 존재하면 그래프에 표시한다 (a) -절편 : 그래프와 축이 만나는 점 방정식 f () 을 푼다 (b) y-절편 : 그래프와 y축이 만나는 점 y-절편 (, f ()) (c) 대칭성 y축 대칭 : f ( ) f ()5 원점 대칭 : f ( ) f ()5 (d) 점근선 lim f () L, 또는 lim f () L이면 y L은 수평점근선이다 lim f () ±, 또는, lim f () ± 이면 a는 수직점근선이다 a 5 정의역의 5 정의역의 a± 모든 점에 대하여 이런 성질을 갖는 함수를 우함수(even function)라고 한다 모든 점에 대하여 이런 성질을 갖는 함수를 기함수(odd function)라고 한다

150 CHAPTER 함수와 미분 6 (e) 임계점 f ()가 존재하지 않거나 f () 인 를 구한다 (f) 함수의 증가하는 구간과 감소하는 구간 임계점과 임계점 사이의 구간에서 f 의 부호를 판정한다 (g) 극댓값, 극솟값 임계점 근방에서 f 의 부호가 바뀌는지 살펴 본다 +에서 로 바뀌면 극댓값, 에서 + 로 바뀌면 극솟값을 갖는다 (h) 변곡점 방정식 f () 을 푼다 변곡점 근방에서 f ()의 부호를 확인하여 그래프가 위로 볼록한 구간과 아래로 볼록한 구간을 구한다 보기 Solution 함수 f () 의 그래프를 그려라 + 우선 f () 이므로 y-절편은 이다 f () 이므로 두 개의 -절편, 이 존재한다 ( ) f () + ( ) + f ( ) 이므로 y축에 대하여 대칭이다 f () + lim f (), 이므로 + lim f () 이고 y 은 유일한 수평점근선이다 분모가 항상 양수인 유리식이므로 수직점근선은 존재 하지 않는다 f () + + ( + ) 이므로 하나의 임계점 이 존재한다 f () ( ) ( + ) ( + ) 이므로 두 개의 변곡점 ±/ 이 존재한다 임계점과 변곡점을 기준으로 f 과 f 의 부호를 구하면 다음과 같다 f () f () + + / + + / +

151 7 함수의 그래프 7 지금까지 살펴 본 것을 토대로 하여 그래프를 그리면 다음과 같다 y 로피탈의 법칙 f () e 나 g() log 와 같이 지수함수 또는 로그함수를 포함한 그래프를 그리려면 lim e lim, e lim log + 와 같은 극한값을 구하여야 한다 첫번째 극한값은 분자, 분모가 모두 가 되는 경우이고 두번 째 경우는 과 의 곱의 형태이어서 극한의 곱, 또는 몫의 법칙을 적용할 수 없다 로피탈의 법칙은 이러한 극한값을 구하는데 유용한 방법이다 미분가능한 두 함수 f, g에 대하여 f (a) g(a) 이라고 하자 f, g 이 연속이고 g (a) 6 이면 f () a g() lim f () f (a) a a g() g(a) a (a) lim a f () f a lim a g() g(a) a lim f (a) g (a) 이다 이 결과는 로피탈의 법칙(L Hopital s rule)의 특수한 경우이다 로피탈의 법칙은 a ± 일 때도 성립한다 또한 a에서의 함숫값이 ± 인 경우에도 성립한다

152 CHAPTER 함수와 미분 8 로피탈의 법칙 f () 이 존재한다고 하자 a [, ]에 a g () 미분가능한 함수 f, g에 대하여 극한값 lim 대하여 lim f () lim g() 이거나 a a lim f () lim g() ± a a 이면 다음 등식이 성립한다 f () f () lim a g () a g() lim 로피탈의 정리는 /꼴이나 / 꼴의 부정형의 극한값을 구하는데 유용하다 예를 들어 log lim lim 은 /꼴의 부정형에 로피탈의 법칙을 적용한 것이고 다음은 / 꼴의 부정형의 계산에 로피 탈의 법칙을 적용한 것이다 lim lim e e 다음과 같이 로피탈의 법칙을 두 번 이상 적용하여 극한값을 구하기도 한다 보기 5 f () n (n은 자연수), g() e 에 대하여 a 에서 로피탈의 법칙을 반복하여 적용하면 n e lim nn e lim lim n e 을 얻는다 따라서 임의의 다항식 p()에 대하여 lim p() e 이다 위의 결과로부터 지수함수는 가 아주 커지면 어떤 다항식보다 빨리 커짐을 알 수 있다 Note 보기 6 다음 극한값을 구하여라 (a) lim log, (p > ) p Solution (b) lim log +

153 7 함수의 그래프 9 (a) / 꼴의 부정형이다 로피탈의 법칙을 적용하면 다음을 얻는다 log lim lim p p p pp lim (b) ( )꼴이지만 다음과 같이 / 꼴로 고쳐서 로피탈의 법칙을 적용할 수 있다 lim log + / log lim + / / lim ( ) lim + e (a) lim log( + ) (g) lim log (h) lim (log ) (i) lim (d) lim e (e) lim lim e (f) lim (b) lim 보기 7 다음 극한값이 존재하면 구하여라5 기초다지기 (c) + log( + ) log(e + ) 다음 함수의 그래프를 그려라 (a) f () e Solution (b) f () log c (a) f () 이므로 원점을 지난다 f ( ) e 이므로 대칭성은 없다 y ( )e 이므로 은 유일한 임계점이고 에서 최댓값 f () e 를 갖는다 로피탈 법칙을 적용하면 lim e lim lim e e 이고 lim e 이므로 y 은 유일한 수평점근선이 된다 y ( ) e + ( )(e ) ( )e 이므로 는 유일한 변곡점이다 < 이면 y < 이므로 위로 볼록이고 > 이면 y > 이므로 아래로 볼록이다 지금까지 살펴본 것을 토대로 그래프를 그리면 다음과 같다 5 (a) (b) (c) (d) (e) (f) (g) (h) (i)

154 CHAPTER 함수와 미분 5 y ã y ã- (b) 함수는 > 에서 정의된다 따라서 대칭성과 y 절편은 존재하지 않는다 y log + 이므로 e 는 유일한 임계점이고 이 점에서 최솟값 f ( e ) e 를 갖는다5 lim log 이므로 수평점근선은 존재하지 않는다 y (log + ) > 이므로 곡선은 정의역에서 아래로 볼록하다 보기 6(b)의 결과 lim log + 과 지금까지 살펴 본 사실들을 종합하여 그래프를 그리면 다음과 같다 y t - ã 함수 f 가 a에서 연속이라면 lim f () f (a)이다 특히, 지수함수는 모든 점에서 연속이므로 lim ef () elim a f () ef (a) a 5 5절 보기 7 의 결과

155 7 함수의 그래프 5 가 성립한다 보기 8 Solution lim 의 값을 구하고 y 의 그래프를 그려라 + log log 이고 lim log 이므로 + f () elog f () 라는 사실을 이용하면 lim lim e log e + + 이다 > 이고 f () (log + ) 이므로 유일한 임계점 e 에서 극솟값 f ( e ) e /e 을 갖는다 < f () f () e e 극솟값 & > e + % lim 이므로 수평점근선은 존재하지 않으며 f () ( ) (log + ) + (log + ) (log + ) + ((log + ) + ) > 이므로 곡선은 항상 아래로 볼록하다 지금까지 살펴 본 것과 lim+ 임을 토대로 그래프를 그리면 다음과 같다 y ã

156 CHAPTER 함수와 미분 5 연습문제 7 다음에 주어진 함수에 대하여 (i) 함수가 증가하는 구간과 감소하는 구간을 구하여라 (ii) 극댓값과 극솟값을 구하여라 (iii) 위로 볼록인 구간과 아래로 볼록인 구간을 구하여라 (a) f () + + (b) f () (c) f () + (e) f () 6 (d) f () (f) f () 다음 함수의 그래프를 그려라 (a) y e (b) y + e (c) y log( + ) log (d) y 삼차함수 f () a + b + c + d의 절편이 α, β, γ일 때 이 함수의 유일한 변곡점은 α+β+γ 임을 보여라 함수 y f ()의 도함수가 f () ( ) ( )이다 f 가 극값을 갖는 의 값을 구하여라 변곡점이 있으면 변곡점의 좌표를 구하여라 5 함수 f () a + 8가 극값을 하나만 가질 때 a의 범위를 구하여라 6 구간 I에서 정의된 두번 미분가능한 함수 f, g가 아래로 볼록이라고 한다 다음 명제가 성립함을 보여라 단, f, g의 도함수와 이계도함수는 이 되는 점이 없다고 가정한다 (a) f + g도 아래로 볼록이다 (b) f, g가 양의 값을 갖고 f, g가 증가함수이면 f g도 아래로 볼록이다 (c) f 가 양의 값을 갖는다면 [f ()] 도 아래로 볼록이다 (d) f 가 증가함수이면 f (g())도 아래로 볼록이다 7 다음 극한이 존재하는 경우 극한값을 구하고 존재하지 않는 경우는 ND 로 나타내어라 e (b) lim log (a) lim (c) lim log (d) lim /n log (n은 자연수) (e) lim e (f) lim (log( + ) log( )) 8 아래의 그림과 같이 빗변의 길이가 이고 밑변의 길이가, 높이가 y인 직각삼각형에서 다음 극한값을 구하여라

157 7 함수의 그래프 5 y Θ θ θ θ θ (a) lim θ θ (b) lim (c) lim 9 두번 미분가능한 함수 y f ()에 대하여 다음 극한값이 존재한다고 한다 lim f ( + ) + f ( ) () (a) f (), f ()의 값을 구하여라 (b) f () 일 때 극한값 ()을 구하여라 임의의 양수 c > 에 대하여 lim+ c log 임을 보여라 다음 식이 성립하는 상수 a, b를 구하여라 lim sin +a+ b 경제상황을 나타나는 지표를 함수 y f ()로 나타낼 수 있다고 하자 다음 조건이 성립할 때 에서 어떤 현상이 나타나는지 설명하여라 (a) f (), f () (b) f (), f () (c) f (), f () (d) f (), f () (e) f (), f () (f) f (), f ()

158 CHAPTER 함수와 미분 테일러급수 테일러다항식 미분가능한 함수 f 에 대하여 a에서 선형근사식은 f 의 그래프와 가장 유사한 직선으로 정의되었다 f () f (a) + f (a)( a) 가 a에 가까울수록 선형근사식에서 좋은 근삿값을 얻을 수 있다 이 선형근사식을 a에서 f 의 차 테일러다항식이라고 하고 T f ()로 나타내기로 한다 차 테일러다항식 a에서 미분가능한 함수 f 의 차 테일러다항식은 다음과 같다 f () T f () f (a) + f (a)( a) 다시 말해서, f 의 차 테일러다항식은 a에서 함수 f 와 함숫값과 기울기가 같은 차 다항 식이다 좀 더 정확한 근삿값은 n차 근사다항식을 구하면 얻을 수 있다 a에서 n번 미분가능한 함수 f 에 대하여 n차 다항식 Tn f () c + c ( a) + c ( a) + + cn ( a)n 이 다음 조건을 만족하면 n차 근사다항식, 또는 n차 테일러다항식이라고 한다 함수 f 와 Tn f 는 a에서 같은 함숫값을 갖는다 함수 f 와 Tn f 는 k,,, n에 대하여 a에서 같은 k계 미분계수를 갖는다 k n일 때, (Tn f )(k) () k(k ) ck + (k + )k ck+ ( a) + + n(n ) (n k + )(n k + )cn ( a)n k 이므로 두번째 조건으로부터 f (k) (a) (Tn f )(k) (a) k(k ) ck ck f (k) (a) k 가 성립한다 n차 테일러다항식 a에서 n번 미분가능한 함수 f 에 대하여 n차 테일러다항식은 다음과 같다 Tn f () f (a) + f (a)( a) + f (a) f (n) (a) ( a) + + ( a)n n

159 8 테일러급수 보기 55 에서 f () cos 의 차 테일러다항식과 8차 테일러다항식을 구하여라 Solution f () sin f () cos f () () sin f () () cos f () f () f () () f () () 이므로 자연수 k에 대하여 f (k+) () f (k+) (), f (k+) (), f (k) () 이다 따라서 차 테일러다항식과 8차 테일러다항식은 각각 다음과 같다 T f () T8 f () y n 8 n -Π Π - 에서 cos ; 559의 근삿값은 각각 다음과 같다 T f () , T8 f () 차 테일러다항식을 이용한 근삿값은 소수점 아래 6자리까지 일치하는 아주 좋은 근삿값을 구 하여 준다 보기 Solution 지수함수 f () e 의 에서 n차 테일러다항식을 구하여라 k 에 대하여 f (k) () e f (k) ()

160 CHAPTER 함수와 미분 56 이므로 f () e 의 n차 테일러다항식은 e Tn f () + + n n 이다 y n n n n 이제 거듭제곱급수로 나타낼 수 있는 함수의 테일러다항식을 살펴 보자 예를 들어, f () 대하여 n f (n) () ( )n+ 에 이므로 f (n) ()/n 이고 n차 테일러다항식은 Tn f () n 이다 다시 말해서, n차 테일러다항식은 거듭제곱급수의 n차 항까지와 일치하는데 이 결과는 거듭제곱급수로 표현되는 함수에 대하여 항상 성립한다 거듭제곱급수의 테일러다항식 f () P k ck k ( < R)일 때 f ()의 n차 테일러다항식은 다음과 같다 Tn f () n X ck k k 보기 에서 f () /( )의 n차 테일러다항식은 Tn f () n 이고 에서 g() /( )의 n차 테일러다항식은 Tn g() ()n 이다

161 8 테일러급수 8 57 테일러정리 Tn f ()를 a에서 f ()의 n차 테일러다항식이라고 할 때 Rn f () f () Tn f () 를 f ()의 n차 테일러나머지항(Taylor s remainder)이라고 한다 혼동의 위험이 없으면 Rn f ()를 간단히 Rn ()로 쓰기도 한다 만약 a < R일 때 lim Rn f () n 이 성립하면 lim Tn f () f () n 가 된다 이때 lim Tn f () n X cn ( a)n, a < R n 를 f 의 테일러급수(Taylor series), 또는 테일러전개(Taylor epansion)라고 정의한다 지금부터 a 일 때의 테일러급수로 한정하여 논의를 전개하기로 한다55 a 6 일 때의 P 결과도 유사하게 성립한다 테일러급수 n cn n 이 f ()로 수렴하는 의 범위를 구하기 위 해서는 n차 나머지 항의 크기를 구하는 것이 필요하다 n차 테일러나머지항에 대하여는 다음과 같은 결과가 알려져 있는데 이 결과를 테일러정 리(Taylor Theorem)라고 한다 테일러정리 구간 ( r, r)에서 (n + )번 미분가능한 함수 f 에 대하여 f () f () + f () + + f (n) () n f (n+) ( ) n+ + n (n + ) 이 성립하는 가 과 사이에 존재한다 테일러정리를 이용하면 n차 테일러다항식의 오차 범위를 다음과 같이 구할 수 있다 r 일 때 f (n+) () M 이면 n차 나머지항 Rn f ()는 다음 부등식을 만족한다 f () Tn f () 55 a Rn f () f (n+) ( ) n+ (n + ) M rn+ (n + ) 일 때 테일러급수를 Maclaulin 급수(Maclaulin series)라고 부르며 다음과 같이 쓸 수 있다 f () X f (n) () n f () f (n) () n f () + f () n n n

162 CHAPTER 함수와 미분 58 n차 테일러다항식의 오차한계 r일 때 f (n+) () M 이면 n차 나머지항 Rn f ()는 다음 부등식을 만족한다 Rn f () M rn+ (n + ) 보기 f () sin 의 차 테일러다항식을 구하고 이를 이용하여 sin 의 근삿값과 이때의 오차에 대하여 설명하여라 Solution (a) f () cos, f () sin 이므로 f (), f () 이고 T f () 이다 모든 에 대하여 f () cos f () 이므로 이면 6 이다 따라서 근삿값 sin ; 의 오차는 /6보다 작다 R f () 8 특수함수의 테일러급수 X n 와 같이 거듭제곱급수로 쓸 수 있는 함수의 테일러급수는 거듭제곱급 n 수의 표현이 유일하므로 거듭제곱급수와 일치한다 따라서 f () 와 g() 의 테일러급수는 각각 다음과 같다 f () X n X n, n, < < n 거듭제곱급수가 알려져 있지 않은 함수의 테일러급수는 테일러다항식을 구한 후 테일러다항식 의 극한으로 정의된다 이때 오차가 으로 수렴하는 의 범위를 구하여야 한다 보기 5 Solution f () cos 의 테일러급수를 구하고 테일러급수가 수렴하는 의 범위를 구하여라 f () cos 에 대하여 f (k ) (), f (k) () ( )k

163 8 테일러급수 59 이므로 코사인함수의 에서 테일러다항식은 다음과 같다 Tn f () Tn+ f () 6 n ( )n 6 (n) 임의의 자연수 k에 대하여 cos(k) () 이므로 나머지항은 Rn f () Rn+ f () n+ (n + ) 을 만족한다 임의의 실수 에 대하여 lim Rn f () lim Rn+ f () n n 이므로 테일러급수는 모든 실수 에 대하여 수렴한다 lim Tn f () lim Tn+ f () cos n n 다시 말해서 코사인함수의 테일러전개는 모든 실수 에 대하여 다음과 같다 cos X ( )n n n (n) 6 보기 6 지수함수 f () e 의 에서 테일러급수를 구하고 테일러급수가 수렴하는 의 범위를 구하여라 Solution f () e 의 n차 테일러다항식은 n n 이므로 테일러급수는 X n n n n n 이다 임의의 실수 에 대하여 Rn () M n+, (n + ) M ma(, e ) 이고 의 값이 고정된 상태에서 n 이면 lim Rn () n 이다 따라서 등식 e X n n n n n 은 모든 실수 에 대하여 성립한다

164 CHAPTER 함수와 미분 6 특히 일 때 다음 식이 성립한다 Note e 자연상수의 근삿값 장에서 자연상수 e를 다음과 같이 정의하였다 n e lim + n n 지수함수의 테일러급수로부터 e n X k k 을 근삿값으로 사용하면 오차는 (n + ) (n + ) (n + ) (n + ) n + (n + ) n+ (n + ) n + 보다 작아진다56 정의로부터 구하는 것보다 테일러급수를 이용한 근삿값이 훨씬 정확한 값을 빨리 구하여 준다57 8 도함수와 테일러급수 다음과 같은 관계식이 성립하는 세 함수를 생각해 보자 f () g(), h () f () () 이 관계식을 만족하는 함수들의 테일러급수는 f 의 테일러급수로부터 쉽게 구할 수 있다 f () 의 테일러급수가 X f () cn n c + c + c + n 56 (e )(n) 57 t n e 이므로 + n n 테일러정리로부터 오차한계를 구하면 다음과 같다 e (n + ) 이라고 하고 sn n X 라고 할 때 근사값 sn 이 근사값 tn 보다 훨씬 오차가 작다 처음 몇 k k 항의 값을 구하면 다음과 같다 n tn sn n 8 9 tn sn

165 8 테일러급수 6 이고 수렴반경이 R > 이라고 하자 g (n) () f (n+) () 이고 f (n) () n cn 이므로 g (n) () f (n+) () f (n+) () (n + ) (n + )cn+ n n (n + ) 이다 따라서 g()의 테일러급수는 g() X X g (n) () n X (n + )cn+ n ncn n n n n n c + c + c + + ncn n + 이다58 lim n 이므로 g() P (n + )cn+ cn+ lim n ncn cn R ncn n 의 수렴반경은 f ()의 수렴반경 R과 일치한다 h () f ()라면 h(n) () f (n ) () (n )이다 따라서 h(n) () f (n ) () f (n ) () cn n n (n ) n n 이고 h() h() + X X n h(n) () n h() + cn n n n n 을 얻는다 lim n cn+ /(n + ) cn+ lim n cn /n cn R 이므로 수렴반경은 f ()의 수렴반경 R과 일치한다 58 이 결과는 f () P n cn n 에서 X n P 과 미분의 순서를 바꿀 수 있음을 의미한다 즉 c n n X (cn n ) n X n ncn n

166 CHAPTER 함수와 미분 6 도함수와 테일러급수 f ()의 테일러급수가 f () X cn n c + c + c + n 이고 수렴반경이 R > 이라고 하자 g() f (), h () f () 이면 g()와 h()의 테일러급수는 각각 다음과 같다 g() X ncn n c + c + c + n h() X h() + cn n n h() + c + c + c + n 두 테일러급수의 수렴반경은 모두 R이다 보기 7 Solution g() sin 의 테일러급수와 수렴반경을 구하여라 ( cos ) sin 이므로 6 5 sin 이고 수렴반경은, 즉, 모든 실수 에 대하여 수렴한다 보기 8 Solution g() ( ) 와 ) ( h() log( )의 테일러급수와 수렴반경을 구하여라 ( ) 이고 P n n 이므로 X nn nn + ( ) n 이다 ( log( )) 이고 h() 이므로 h() log( ) 이다 두 함수 모두 수렴반경은 이다 X n n n

167 8 테일러급수 Note 6 보기 8의 g()에 대한 결과에 /를 대입하면 n g n + 이고 g ( ) 이므로 n n + 이다 한편, h()에 대한 결과로부터 h n + + n 이고 h log log 이므로 다음 결과를 얻는다 n + log n 지금까지 구한 중요함수의 테일러급수를 정리하면 다음과 같다 중요함수의 테일러급수 log( ) X n, < n + X n + +, n n < X n n n cos sin e 85 X 6 n + + ( )n 6 (n) n X 5 n + + ( )n 5 (n ) n 테일러급수 구하기 거듭제곱급수가 존재하는 경우, 테일러급수와 거듭제곱급수가 일치하므로 테일러급수를 쉽게 구할 수 있다 예를 들어 f () e 의 테일러급수를 구하여 보자 e 에서 대신 을 대입 하면 다음과 같은 테일러급수를 얻는다 e X 6 ( )n + + n n

168 CHAPTER 함수와 미분 6 이번에는 f () e +e 의 에서의 테일러급수를 구하여 보자 e X n, n n e X ( )n n n n 이고 두 급수 모두 임의의 실수에 대하여 수렴하므로 e + e X n X ( )n n + n n n n X n ( )n n + n n n 이다 k, ( ) + (k) n n, n n n n k 일 때 n k + 일 때 이므로 n e + e (n) 이다 보기 9 주어진 점에서 다음 함수들의 테일러급수를 구하여라 (a) cos, (b) e, Solution (a) cos 의 테일러급수에서 대신 를 치환하면 얻어진다 () () + X n ( )n n (n) n cos (b) e + + e + + 이므로 e 의 테일러급수는 다음과 같다 X n n n

169 8 테일러급수 65 연습문제 8 자연수가 아닌 실수 r에 대하여 f () ( + )r 이라고 할 때, 에서 테일러급수를 구하여라 단, 실수 r 과 자연수 n 에 대한 이항계수(binomial coefficient)를 다음과 같이 정의하기로 한다59 ( ) r r r (r ) (r n + ), n n 문제 의 결과를 이용하여 다음 함수의 에서 테일러급수의 (이 아닌) 처음 개 항을 구하여라( ) (a) + (b) + (c) + 에서 다음 함수의 차 테일러다항식을 구하여라 (a) + (c) log( + ) (b) sin (d) e 에서 함수 f 의 차 테일러다항식이 T f () 5 + 이다 f (), f (), f () 을 구하여라 5 f () sin( )일 때, n 9,, 에 대하여 f (n) ()을 구하여라 6 a에서 다음 함수의 n차 테일러다항식을 구하여라 (a) f () +, (b) f () + +, a, n a, n (c) f () +, a, n 7 차의 속도가 5m/sec일 때, 가속도가 m/sec 라면 초 동안 차가 움직인 거리를 차 테일 러다항식을 이용하여 구하여라 분 동안 움직인 거리를 같은 방법으로 구할 수 있는가? 답에 대한 이유를 설명해 보아라 ( ) 8 에서 다음 함수의 테일러급수를 구하여라 (a) e 59 여기서 (b) + r 이 n 보다 크거나 같은 자연수이면 이항계수 r Cn r n(r n) 와 일치한다 따라서 이 정의는 이항계수의 일반화라고 할 수 있다 (c) log( + )

170 66 CHAPTER 함수와미분

171 Chapter 적분과 응용 부정적분 함수 f () 의 도함수는 f () 이다 거꾸로 함수 F 의 도함수가 라면 F 는 어떤 함수일까? 을 미분하면 가 되므로 F () 이라고 할 수 있다 그러나 도함수의 경우와는 달리 이런 성질을 갖는 함수는 무수히 많다 C가 상수일 때 ( + C) 이므로 다음과 같은 형태의 함수 F () + C 는 모두 도함수가 이다 역도함수 함수 f 에 대하여 F () f () 가 되는 미분가능한 함수 F ()를 f ()의 역도함수(antiderivative)라고 한다 예를 들어, 는 의 도함수이고 은 의 역도함수 이다 F ()가 f ()의 역도함수라면 F () + C도 f ()의 역도함수가 된다 또한 G()가 f ()의 다른 역도함수라면 (G() F ()) f () f () 이므로 G() F ()는 상수함수이다 따라서 f 의 역도함수는 항상 F () + C 67

172 CHAPTER 적분과 응용 68 의 꼴로 쓸 수 있다 함수 f ()의 역도함수의 집합을 부정적분(indefinite integral)이라고 하고 f () d 로 나타낸다 그러나 집합 기호는 사용하지 않으며 다음과 같이 이해하기로 한다 F () f () f () d F () + C () 여기서 함수 f ()를 피적분함수(integrand), C를 적분상수(constant of integration)라고 한다 적분상수 C에 대하여 연산을 한 결과는 다시 C 로 쓰기로 한다 예를 들어 f () d F () + C F () + C 로 쓴다 식 ()은 다음과 같이 바꾸어 쓸 수 있다 도함수와 역도함수 d f () d d f () d f () (y f ()는 연속함수) f () + C (y f ()는 미분가능한 함수) 여러 함수의 부정적분 실수 n에 대하여 (n+ ) (n + )n 이므로 n 6 이면 n+ n n+ 이다 따라서 다음 부정적분을 얻는다 n d n+ + C n+ (n 6 ) 의 부정적분은 (log ) 에서 다음과 같다 d log + C n 의 부정적분 엄밀히 d n+ + C (n 6 ), n+ n d log + C 말하자면 역도함수와 부정적분을 구분하여야 하지만 대부분의 교재는 두 용어를 혼용하여 사용한다 연산에서라면 위의 식에서 C 이 된다 그러나 적분상수는 일반적인 실수를 나타내는 수임을 상기하고 새로운 적분상수(C 이나 C 등)를 사용하는 대신 계속 적분상수 C로 나타내기로 한다 일반적인

173 부정적분 69 기초다지기 다음 부정적분을 구하여라 (a) (b) d (c) d (d) d (e) d (f) d (g) d (h) d d 지수함수의 미분법칙으로부터 (e ) e e d e + C 이다 또한 (a ) a log a이므로 다음 결과를 얻는다 a a +C a, 즉, a d log a log a 지수함수의 부정적분 e d e + C, 기초다지기 (a) a d a +C log a 다음 부정적분을 구하여라 d (b) d (c) d 삼각함수의 미분법으로부터는 다음 부정적분을 얻는다 삼각함수의 부정적분 (sin ) cos (cos ) sin (tan ) sec (e) (a) + C (c) + C (b) + C (d) + C (f) (a) (b) log (c) log log cos d sin + C sin d cos + C / 5/ 5 sec d tan + C +C +C (g) / + C (h) / + C

174 CHAPTER 적분과 응용 7 부정적분의 성질 F (), G()가 각각 연속함수 f (), g()의 역도함수라고 하자 그러면 F () f (), G () g() 이므로 임의의 상수 k에 대하여 (kf ()) (F () ± G()) kf () kf () F () ± G () f () ± g() (복호동순) 이다 따라서 부정적분에 대한 다음 공식을 얻는다 부정적분의 성질 kf () d [f () ± g()] d 다음 조건이 성립할 때 주어진 부정적분을 구하여라5 기초다지기 f () (a) f () d 보기 (a) d d ( + e ), g() ( e ) d d (b) [ g()] d (c) [f () + g()] d 다음 부정적분을 구하여라 + d Solution 5 (a) k f () d (k는 상수) f () d ± g() d (복호동순) (b) d + d + (a) 분수식을 간단히 한 후 계산한다 + d / + d / d d + d /+ + log + C / + / + log + C + e + C (b) + e + C (c) + C

175 부정적분 7 (b) 부정적분의 두번째 성질을 이용하면 d d d ( ) d C + + C 을 얻는다 다음 부정적분을 구하여라6 기초다지기 (a) ( + ) d (b) + d (c) y dy y (d) 6t dt t 보기 (e + e ) d (e) (f) ( + ) d + (g) (h) ( ) d ( + ) d + sin d + ( ) d cos d 미분가능한 함수 f ()가 다음 관계식을 만족한다고 할 때 f () 를 구하여라 f () d f () + + C, f () 단, f 은 연속이라고 가정한다 Solution 양변을 에 대하여 미분하면 f () f () + f () + 이고 정리하면 f () f () 이므로 f () ( ) d + C 이다 f () 으로부터 C 이므로 f () + 이다 6 (a) + + C (b) + log + C (c) (d) y + y + C t t + C (e) e e + C (g) + + C (f) + C (h) + C

176 CHAPTER 적분과 응용 7 함수 F ()가 f ()의 역도함수라고 하자 f (a + b)의 도함수는 af (a + b)이다 그렇다면 f (a + b)의 부정적분은 어떻게 될까? [F (a + b)] af (a + b) af (a + b) 이므로 f (a + b)의 부정적분은 다음과 같다7 f (a + b) d F (a + b) + C a 보기 (a) 다음 부정적분을 구하여라 Solution (b) R (a) R (b) e + d 6 + C이므로 6 i h ( + )5 d ( + )6 + C ( + )6 + C 6 5 d e d e + C이므로 + e + d e + C e + + C 기초다지기 (a) (b) (a) (b) 다음 부정적분을 구하여라8 (c) d d + 7 도함수에서는 8 ( + )5 d (d) d ( ) (e) e (c) C (d) d + d (f) 상수 a가 곱해지고 부정적분에서는 상수 a가 나누어진다 ( ) +C log( + ) + ( ) + C ( + ) + C 6 (e) e (f) +C cos( + ) + C sin( + ) d

177 부정적분 7 연습문제 다음 함수의 역도함수를 하나 구하여라 (b) (a) (c) sin k (e) e (d) sec k (f) e + 다음 부정적분을 구하여라 (a) (b) + d + + d (c) (d) sin k d (e) sec k d (f) e d (e + ) d 다음 식이 성립함을 미분을 이용하여 보여라 p (a) d + + C + (b) ( ) d +C ( ) (c) cos d sin + cos + C (d) (e) (f) (g) e d e + C ( )e d e + C e d e + C log d (log ) + C (h) (log + ) d log + C 다음 부정적분을 구하여라 (a) (b) (et + ) d t (e + ) dt ( s + s) ds (d) cos d (e) (e + e ) d (c) (f) (g) ( e) d d ( ) d (i) d d + + (j) (cos + ) d + (sin ) d (h)

178 CHAPTER 적분과 응용 7 5 함수 f 가 다음 조건을 만족할 때 f 를 구하여라 (a) f (), (b) f () cos + sin, f () 7 f () (c) f () e +, f () (d) f () +, >, f () (e) f () 6 +, f (), f () (f) f (), f (), f () (g) f () cos + sin, f (), f () (h) f () e/ + e, f (), f () 6 연속함수 y f ()가 다음 식을 만족할 때 함수 f ()를 구하여라 (a) (b) f () d + C (c) f () d + + C (d) f () d e + e + C f ( ) d + C, 7 y f ()는 미분가능한 함수이고 F ()는 f ()의 역도함수라고 한다 모든 에 대하여 F () f ()가 성립하면 f () 는 상수함수임을 보여라 8 F ()는 미분가능한 함수 f ()의 역도함수이고 다음 관계식이 성립한다 F () f () + + f () 일 때 f ()를 구하여라

179 적분기법 75 적분기법 치환적분법 일반적인 합성함수의 부정적분을 구하기 위해서는 치환법칙을 사용한다 치환법칙은 연쇄법칙 과 밀접한 관계가 있다 예를 들어, e e 이므로 e d e + C, 또는 e d e +C 이 성립한다 또한 log( + ) + 이므로 d log( + ) + C, + 또는, d log( + ) + C + 이다 이렇게 피적분함수가 합성함수의 도함수이면 부정적분을 구할 수가 있다 함수 u g()가 미분가능하고 함수 F (u)가 f (u)의 역도함수라고 하자 연쇄법칙에 의하여 [F (g())] F (g())g () f (g())g () 이므로 F (g())는 f (g())g ()의 역도함수이다 f (g())g () d F (g()) + C F (u) + C F (u) + C R f (u) du이므로 다음 공식을 얻는다 f (g())g () d F (u) + C f (u) du 이렇게 합성함수 f (g())에서 g()를 u로 치환하여 적분하는 방법을 치환적분법(integration by substitution) 이라고 한다 치환적분법 함수 u g()가 미분가능할 때 다음 식이 성립한다 f (g())g () d f (u) du Note 라이프니츠의 표기법 du d g ()를 이용하면 위 식은 다음과 같이 쓸 수 있다 du f (g())g () d f (u) d f (u) du d

180 CHAPTER 적분과 응용 76 따라서 u g()일 때 du d g ()를 du g () d 또는 d du g () 로 나타내기로 한다9 보기 (a) 다음 부정적분을 구하여라 ( + ) d (b) e / d Solution (a) u + 이라고 하면 du d이므로 d du 이다 따라서 i h ( + ) d u du u + C + ( + ) + C 을 얻는다 (b) u /이라고 놓으면 d du 또는 d du이므로 / e d eu du eu + C e / +C 을 얻는다 기초다지기 (a) (b) 다음 부정적분을 구하여라 ( + ) d p (c) d (d) + d ( + ) d (e) (f) e d sin( + ) d 위 보기에서는 무엇을 치환하면 부정적분을 구할 수 있는지가 명백해 보인다 즉, f (an + b)n d 의 꼴의 부정적분은 u an + b로 치환하면 부정적분을 구할 수 있다 그러나 일반적으로는 치환적분에서 어떤 함수를 치환하여야 하는지가 명백히 보이지 않는 경우들이 더 많다 이런 9 이는 연쇄법칙에서는 곱의 형태로 나타난 것이 치환적분에서는 나누기의 형태로 나타남을 보여준다 (a) ( + ) + C (b) ( ) + C (c) + + C 8 (d) 6 ( + ) + C (e) e + C (f) cos( + ) + C 실제로 치환하는 방법은 여러가지 있을 수 있으며 그 중에서 가장 계산이 쉬워지는 방법을 선택한다

181 적분기법 77 경우는 치환하는 함수의 도함수가 무엇인지를 고려하여 적분이 쉬워지는 치환을 하는 것이 중 요하다 보기 (a) 다음 부정적분을 구하여라 d log d e d (c) (b) sin d (d) Solution (a) u 이라고 하면 du d이고 u + 이다 따라서 (u + ) u du u/ du + u/ du d 5/ / u + u +C 5 5/ ( ) + ( )/ + C 5 (b) u 이라고 하면 du d이므로 sin d (c) u log 로 놓으면 du (d) u 로 놓으면 du cos sin u du cos u + C +C d이다 따라서 log d u du u + C (log ) + C d이다 따라서 e d e eu du eu + C +C f ()/f ()의 적분 f () 6 일 때 함수 f () f () 의 부정적분을 구하여 보자 u f () 라고 하면 du f ()d

182 CHAPTER 적분과 응용 78 이므로 f () d f () du log u + C u log f () + C 를 얻는다 f ()/f ()의 적분 보기 (a) f () d log f () + C f () 다음 부정적분을 구하여라 d + (b) tan d (c) et dt + et Solution (a) u + 라고 하면 du d이므로 d + (b) tan sin cos 이므로 d du + u log u + C log( + ) + C u cos 로 치환하자 du sin d이므로 sin d du cos u log u + C log cos + C log sec + C tan d (c) u + et 라고 하면 du et dt 이므로 et dt + et du log u + C u log( + et ) + C

183 적분기법 79 부분적분법 미분가능한 두 함수 f (), g()에 대하여 두 함수의 곱에 대한 미분은 [f ()g()] f ()g() + f ()g () 으로 주어진다 양변을 부정적분을 이용하여 나타내면 f ()g() [f ()g() + f ()g ()] d 이 된다 따라서 다음과 같은 관계식을 얻는다 f ()g () d f ()g() f ()g() d 이 공식에 의한 적분법을 부분적분법(integration by parts)이라고 한다 부분적분법 f ()g () d f ()g() f ()g() d 부분적분법은 다음 형태로 바꾸어 쓰면 기억하기가 더 좋다 Note u f (), v g() 라고 놓으면 du f () d, dv g () d 이다 따라서 치환적분법에 의하면 부분적분법은 다음과 같이 쓸 수 있다 u dv uv v du 보기 다음 부정적분을 구하여라 (a) sin d (b) e d (c) log d Solution (a) u, dv sin d라고 하면 du d, v cos 이다 따라서 부분적분법에 의하면 sin d ( cos ) ( cos ) d cos + cos d cos + sin + C

184 CHAPTER 적분과 응용 8 (b) u, dv e d라고 하면 du d, v e 이다 따라서 부분적분법에 의하면 e d e e d e e + C (c) u log, dv d라고 하면 du d, v 이다 따라서 부분적분법에 의하면 (log ) d log d log d log + C Note 부분적분법은 두 함수의 곱을 적분할 때 유용한 방법이다 그러나 하나의 함수도 두 함수의 곱으로 생각할 수 있다 즉, f () f () () f () 로 놓으면 하나의 함수의 부정적분도 부분적분법을 이용하여 구할 수 있다 보기 5 log d를 구하여라 log () log 로 생각하고 u log, dv d로 놓으면 log d u dv uv v du log d log + C Solution 를 얻는다 두 함수의 곱을 적분하는데 한 함수가 의 다항식이면 부분적분법에 의하여 차수가 하나씩 내려간다 따라서 부분적분을 반복하면 결국 한 함수의 적분 꼴을 계산하면 되는 형태를 얻게 된다 그러나 두 함수 모두 다항식이 아니면 부분적분법을 사용하여도 피적분함수가 더 단순한 형태로 바뀌지 않는다 그러나 피적분함수가 지수함수나 삼각함수의 형태이면 부분적분을 반 복하면 같은 형태의 피적분함수를 얻게 되어 부정적분을 구할 수 있다 보기 6 Solution e sin d를 구하여라 e 나 sin 는 모두 미분에 의하여 더 간단한 형태로 바뀌지 않는다 그러나 우선 u e, dv sin d

185 적분기법 8 로 치환하자 그러면 du e d, v cos 이고 부분적분법에 의하여 e sin d e ( cos ) e ( cos ) d () 를 얻는다 오른쪽 부정적분을 구하기 위하여 다시 u e, dv cos d 로 치환하자 그러면 du e d, v sin 이고 e cos d e sin e sin d 를 얻는다 이 식을 식 () 에 대입하면 e sin d e cos + e sin e sin d 이다 이 식을 정리하면 e sin d e (sin cos ) + C 이다 Note 위 보기에서 u sin, dv e d로 놓고 풀어도 같은 결과를 얻는다

186 CHAPTER 적분과 응용 8 연습문제 주어진 (a) (b) (c) (d) (e) 치환을 이용하여 부정적분을 구하여라 sin d, u cos( ) d, u ( ) d, u 6 ( + ) d, u + dt, u 7t 7t t dt, u 9t (f) 9t (g) t sin t dt, u t ( + ) d, u + (h) sec d, u (i) (j) d, u e (log ) d, u log (k) (log f () ) (a) f () f () 임을 이용하여 다음 부정적분을 구하여라 d + (b) e t dt e t + (c) 부분적분법을 이용하여 다음 부정적분을 구하여라 (a) (b) (c) sin d (d) d e (e) e cos d 다음 부정적분을 구하여라 sec d log d log d (f) cot t dt

187 적분기법 8 d (a) p + d sin (c) d (b) d log + (e) d + + e (f) d e + (d) 5 다음 부정적분을 구하여라 (a) (b) (log ) d (d) t log t dt (e) (c) sin e cos(log t) dt (f) d d e d 6 자연수 n에 대하여 다음 식이 성립함을 보여라 n e d n e + n n e d 7 부분적분을 이용하여 다음 식이 성립함을 보여라 f () d f () f () d

188 CHAPTER 적분과 응용 8 부정적분의 응용: 변수분리법 f ()의 부정적분을 구하는 문제는 y f () 인 함수 y F ()를 모두 구하는 것과 같다 다시 말해서, f ()의 부정적분 정식 y f () R f ()d는 미분방 의 해이다 미분방정식의 해를 모두 구하는 것을 미분방정식을 푼다고 한다 변수분리법 y의 변화율이 다음과 같이 의 함수와 y의 함수의 곱으로 주어지는 경우를 살펴보기로 하자 dy f ()g(y) d () 이런 형태로 주어진 미분방정식을 변수분리방정식이라고 한다 미분방정식 ()에서 g(c) 이 되는 상수 c가 존재하면 상수함수 y c는 ()의 해이다 예를 들어, 상수함수 y 은 미분방정식 dy (y ) () d 을 만족하므로 ()의 해이다 이런 상수해를 평형해 (equilibrium solution)라고 한다 h(y) g(y) 를 ()의 양변에 곱하면 h(y) 이다 양변을 에 대하여 적분하면 h(y()) dy f () d dy d d f () d 이므로 변수분리방정식 ()은 다음 방정식과 동치이다 h(y) dy f () d (5) 이런 의미에서 미분방정식 ()을 다음과 같은 형식으로 쓰기도 한다 h(y)dy f ()d ()의 해를 구하기 위해서는 다음 부정적분 H(y) h(y) dy, 함수를 F () f () d 주어진 미분방정식에 대입하였을 때 등식이 성립하면 해(solution)라고 한다

189 부정적분의 응용: 변수분리법 85 를 구하여야 한다 이렇게 변수분리방정식을 푸는 것을 변수분리법(separation of variable) 이라고 한다 보기 (a) 다음 미분방정식을 풀어라 dy ky d Solution (b) dy y d (a) 상수함수 y 은 평형해이다 변수분리하여 쓰면 dy/y kd이므로 dy k d y 이다 부정적분을 구하면 log y k + C 이다 여기서 C는 적분상수이다 따라서 y ek+c ec ek y ±ec ek 이다 C가 실수이면 ec 는 양의 실수를, ec 는 음의 실수를 의미한다 y ek 이므로 주어진 미분방정식의 해는 y Cek 가 된다 (b) 상수함수 y 은 평형해이다 변수분리하여 쓰면 y dy d이므로 y dy d 이다 부정적분을 구하면 +C 이다 여기서 C는 적분상수이다 A C로 놓고 간단히 하면 y y A 이다 보기 Solution 미분방정식 dy t 을 풀어라 dt y 변수분리하여 쓰면 y dy t dt 이다 양변을 적분하면 다음 결과를 얻는다 t y +C t + y r 여기서 r C이다 즉, 이 미분방정식의 해곡선은 원점이 중심인 원으로 나타난다

190 CHAPTER 적분과 응용 86 y y hhtl r r t y hhtl Note 주어진 미분방정식의 해는 다음과 같이 두 개의 함수로 쓸 수 있다 h (t) p r t, p h (t) r t 따라서 조건 y() 이 주어지면 r 이고 y h (t) p t 만이 주어진 문제의 해가 된다 변수분리법 변수분리방정식 dy f ()g(y) d 는 다음과 같은 과정을 통하여 푼다 모든 평형해를 찾는다 변수를 분리하여 쓰고 양변을 적분하여 음함수해를 찾는다 h(y)dy f ()d h(y)dy f ()d 여기서 h(y) g(y) 이다 Note 조건이 주어진 경우 음함수해에서 상수항을 결정하고 양함수로 고쳐 쓸 수 있는 경우는 양함수로 써준다

191 부정적분의 응용: 변수분리법 87 자연성장, 자연감소 798년 영국의 경제학자 맬더스 는 세계인구증가를 수학적으로 모형화한 논문을 발표하였 다 맬더스는 이 논문에서 인구증가율은 항상 현재의 인구에 비례한다는 가설을 제시하였다 이 가설은 이상적인 환경에서는 인구5 의 증가율이 인구의 크기에 비례한다는 가정에서 출발 한다 다시 말해서, 인구가 많으면 많을수록 인구는 더 많이 증가한다6 이런 원칙을 수학적으로 표현하기 위하여 시점 t에서의 인구를 y y(t)로 나타내기로 하자 각 시점에서 시간에 따른 인구증가율은 dy/dt이므로 인구증가율이 인구에 비례한다는 가정은 다음과 같은 미분방정식으로 표현이 가능하다 dy ky dt (6) 여기서 상수 k는 양의 값을 갖는 실수이며 실험적으로 그 값이 결정된다 따라서 dy/dt y 로 정의되는 인구의 상대성장률(relative growth rate)은 시간에 관계없이 상수 k이다 어떤 시점 t 에서 인구가 y 이라면 인구 y(t)는 다음 두 조건을 만족한다 dy ky, dt y(t ) y 미분방정식과 한 점에서의 함숫값이 주어진 경우 초깃값문제(initial value problem)라고 하고 미분방장식의 해 중에서 주어진 조건을 만족하는 함수를 찾는 것을 초깃값문제를 푼다고 한다 자연성장 초깃값문제 dy ky, dt y() y 에서 미분방정식의 해는7 y(t) Cekt 이므로 조건 y() y 에서 Cek C y 이다 따라서 초깃값문제의 해는 y(t) y ekt Malthus, Thomas Robert(766-8),영국의 경제학자이자 성직자 제목은 An essay on the Principles of Population (Growth) as It Affects the Future Improvement of Society 이다 5 예를 들어 사람, 식물, 박테리아, 곤충 등 6 예를 들어 인구가 만명일 때 년동안 5명이 증가하였다면 인구가 만명이면 년동안 5명이 증가한다 7 보기 (a)의 결과 논문

192 CHAPTER 적분과 응용 88 이다 k > 이면 해가 증가하는 지수함수로 주어지므로 식 (6)은 지수성장방정식(eponential growth equation), 또는 자연성장방정식(natural growth equation)이라 부른다 보기 년 월 세계인구는 6억명이고, 월 한 달 동안 인구는 75만명이 증가하였다 인구증가율이 인구에 비례한다고 가정할 때, 인구가 두 배가 되는 것은 약 몇 년 후인가? Solution 년에서 t년 후의 인구를 P (t)라고 하면 인구증가율이 인구에 비례하므로 P 는 다음 초깃값문제를 만족한다 dp kp, P () 6 dt 여기서 t 은 년에 해당한다 그러므로 P (t) 6ekt 이다 인구가 월 한 달에 75만명 즉 75억명이 늘었으므로 P () ; ) P () P ( 75 9 (억명/년) 이다 따라서 k P (t) P (t) t P () 9 ; 5 P () 6 이다 즉, 년 인구 증가율은 약 5%이고 t년 후의 인구는 P (t) 6e5t (7) 이다 인구가 두 배가 되는 해, 즉, 억이 되는 해를 찾기 위해서는 다음 식을 풀면 된다 P (T ) 6e5T T log(/6) ; 6(년) 5 따라서 6년 인구가 억이 될 것으로 추정된다 인구증가율이 인구에 비례한다는 가정하 에서는 그 후로 6년마다 인구는 두 배로 늘어난다 Note 보기 에서 k는 한 달 후의 인구가 675억명이라는 사실과 (7)을 이용하여 다음과 같이 구할 수도 있다 k 675 P 6e 675 k log ; 99 6 자연감소 아스피린(aspirin)이나 페니실린(penicillin) 같은 약품이 인체에 투여되면 약품은 혈액으 로 흡수되며 시간이 지남에 따라 각 기관으로 흡수된다 약품의 혈액농도의 변화율은 약품의 혈액농도에 비례한다는 것이 알려져 있다 즉, 시점 t에서 약품의 혈액농도를 y(t)라고 하자

193 부정적분의 응용: 변수분리법 89 시점 t에 대한 약품의 혈액농도의 변화량은 dy/dt이므로 약품의 혈액농도의 변화량이 약품의 혈액농도에 비례한다는 가정은 다음과 같은 미분방정식으로 쓸 수 있다 dy ky dt (8) 여기서 상수 k는 양의 값을 갖는 실수이며 실험적으로 그 값이 결정된다 y는 시간에 따라 감 소하므로 음의 부호가 필요하다 어떤 시점 t 에서 약품의 혈액농도가 y 라고 알려져 있다면, 약품의 혈액농도 y(t)는 다음과 같은 초깃값문제를 만족한다 dy ky, dt y(t ) y 이 경우도 해는 다음과 같이 지수함수로 주어진다 y y e k(t t ) 미분방정식 (8)은 지수감소방정식 (eponential decay equation), 또는 자연감소방정 식(natural decay equation)이라 부른다 아래 그림은 y(t)의 전형적인 그래프를 k > 인 경우와 k < 인 경우 각각 보여 주고 있다 y k< k> y t 자연성장과 자연감소 임의의 상수 k에 대하여, 초깃값문제 dy ky, dt y(t ) y 의 해는 다음과 같이 주어진다 y(t) y ek(t t ) 보기 혈중알콜농도는 현재의 농도에 비례하여 감소하며 한 시간이 지나면 혈중알콜농도의 된다고 한다 현재 혈중알콜농도가 mg/l인 사람에 대하여 다음 물음에 답하여라 배가

194 CHAPTER 적분과 응용 9 (a) 시간 후의 혈중알콜농도를 구하여라 (b) 혈중알콜농도가 mg/l 이하가 되는 것은 몇 분 후인지 구하여라 Solution (a) 지금부터 t시간 후의 혈중알콜농도를 y(t)라고 하자 혈중알콜농도는 현재의 농도에 비례하여 감소하므로 초깃값문제 y ky, y() 를 만족한다 따라서 y(t) e kt 이다 한 시간 후 혈중알콜농도는 배가 되므로 y() e k 9 에서 e k 이고 t y 이다 따라서 시간 후의 혈중알콜농도는 y() 675 (mg/l) 이다 (b) y(t) t 를 정리한 후 로그를 취하면 t log t log 에서 log ; 8 log log 을 얻는다 8시간, 즉, 시간 9분이 지나면 혈중알콜농도는 mg/l 이하가 된다 뉴턴의 냉각법칙 어떤 물체의 시점 t에서의 온도를 y(t)라 하자 뉴턴의 냉각법칙(cooling law of Newton) 에 의하면 온도 y(t)의 변화율은 매개체의 온도와 그 물체의 온도 차이에 비례한다 따라서 이 물체를 온도가 A인 매개체에 놓았다면 다음과 같은 미분방정식을 얻는다 dy k(y A) dt (k >, 상수) (9) 여기서 물체의 크기는 매개체에 비하여 상대적으로 작아서 물체에 의한 매개체의 온도 변화는 무시할 수 있는 것으로 간주한다 물체의 초기온도가 y() y 이라고 하자 z y A로 놓으면 z() y A이고 미분방정식 (9)는 dz kz dt

195 부정적분의 응용: 변수분리법 9 가 된다 z z()e kt 이므로 y(t) A (y A)e kt 즉, y(t) A + (y A)e kt 이다 y는 y < A이면 순증가, y > A이면 순감소하며 시간이 흐름에 따라 물체의 온도는 매개체의 온도로 수렴한다 lim y(t) A t 또한 y(t) A는 평형해가 됨을 주목하자 이 사실은 평형해 y A는 모든 해곡선의 수평점근 선이 됨을 의미한다 y y y A A y t t 보기 5 가마에서 꺼낸지 분된 도자기의 온도가 7 C이다 분 후 다시 재 보았더니 도자기의 온도는 8 C이었다면, 도자기를 처음 꺼냈을 때의 온도는 얼마이겠는가? 단, 방의 온도는 C를 유지하고 있다고 한다 Solution t분 후 도자기의 온도를 y(t)라고 하자 그러면 y k(y ), y() 7, y() 8 이 성립한다 미분방정식을 풀면 y + Ce kt 이고 주어진 조건을 이용하면 y() + Ce k 7, y() + Ce k 8 이므로 e k, 5 C 75 을 얻는다 따라서 처음 도자기의 온도는 y() C 이다

196 CHAPTER 적분과 응용 9 연속복리 이자가 연속복리로 지급되는 경우의 자산가치는 자연성장하는 대표적인 예이다 다시 말해서 이자가 연 r(r%)의 연속복리로 지급된다면 현재 A원의 t년 후 자산가치는 y Aert 이므로 y raert ry 을 만족한다 즉 자산의 증가율은 자기자신에 비례한다 이자가 자주 지급되는 경우 예를 들어 일복리로 지급되는 경우 연속복리를 이용하여 근삿값을 구하기도 한다 보기 6 년 월 일에 만원을 저금하고 다음날부터 7년 월 일까지 매일 5 원씩 저금을 하려고 한다 연이율은 5%이고 이자는 일복리로 지급된다고 한다 (a) t년 후 자산의 가치를 A(t)(단위 만원)라고 할 때 A가 만족하는 초깃값문제를 구하여라 (b) 8년 월 일에 저금을 찾는다면 얼마를 수령하겠는가? (c) 자산가치가 만원이 되는 것은 언제인가? Solution (a) 년 월 일 만원을 저금한 직후를 t 이라고 하자 하루가 지난 후의 자산 A(t + 65 )이 증가하는 원인은 이자와 적립금 두 가지이다 이자는 원금에 비례하며 하루의 이율은 5 65 이고 하루의 적립금은 5이므로 하루 후의 자산은 A(t + 5 ) A(t) + A(t) 가 된다 따라서 A(t + da ; dt 65 ) 65 A(t) 5A + 85 이므로 근사적으로 A(t)는 다음 초깃값문제를 만족한다 da 5A + 85, dt A() (b) 미분방정식 ()에서 y A + 65으로 놓으면 dy 5y, dt y() 66 이 된다 따라서 y 66e5t 가 되고 A(t) y(t) 65 66e5t 65 ()

197 부정적분의 응용: 변수분리법 9 이다 그러므로 8년 월 일, 즉, 5년 후의 자산가치는 A(5) e5 ; 95(만원) 이다 (c) 만원이 되는 t의 값을 T 라 하자 그러면 A(T ) e5T 이므로 e5t 95, 66 즉, T 95 ; 55 log 5 66 이다 55년 후, 즉 ; 557 일 후면 자산가치는 만원이 된다 5 시장가격의 결정 모형 자유경쟁시장의 경우 수요와 공급은 가격에 의하여 결정된다8 가격 P 에 대하여 수요함수 Qd 와 공급함수 Qs 가 각각 Qd a + bp, Qs c + dp () 라고 하자 일반적으로 가격이 오르면 수요는 줄고 공급은 늘어나므로 b <, d> 으로 가정한다 수요량과 공급량이 같아지는 가격이 존재하면 그 가격을 균형가격 (equilibrium price)9 이라고 부른다 수요함수와 공급함수가 식 ()로 주어진 경우 균형가격 Pe 는 a c Qd Qs Pe d b 이다 수요와 공급이 다를 때, Qd Qs 를 초과수요라고 한다 보기 7 가격 P 에 대하여 수요함수 Qd 와 공급함수 Qs 가 식 ()로 주어졌다고 하자 시 장에서 가격의 변화율 dp dt 는 초과수요 Qd Qs 에 비례한다고 할 때 가격함수 P P (t)를 구하여라 limt P (t)는 어떻게 되는가? 단, P () P 이라고 하자 Solution dp dt 는 초과수요 Qd Qs 에 비례하므로 양의 상수 k에 대하여 dp k(qd Qs ) dt 8 또한 가격은 수요와 공급에 의하여 결정되기도 한다 평형가격 수요가 공급보다 많으면 가격은 상승하고, 수요가 공급보다 적으면 가격은 하락하므로 k > 이다 9 또는

198 CHAPTER 적분과 응용 9 이다 식 ()로부터 Qd Qs a c + (b d)p 이므로 다음과 같은 미분방정식을 얻는다 dp a c k(a c + (b d)p ) k(b d) P dt d b y P Pe P a c d b 로 놓으면 주어진 미분방정식은 dy k(b d)y, dt y() P Pe 이므로 y y()ek(b d)t P Pe + (P Pe )ek(b d)t 이다 k(b d) < 이므로 lim P (t) Pe t 이다 다시 말해서, 초기조건에 상관없이 가격은 균형가격으로 수렴한다

199 부정적분의 응용: 변수분리법 95 연습문제 다음의 미분방정식을 변수분리법을 이용해서 풀어라 t y (b) ty y (e) (a) y (f) y (c) y y sin t ty ey+ t et y et+y (g) y + t + y + ty (d) ( t )y y 다음 초기값문제를 풀어라 단, 양함수 꼴로 쓸 수 있는 경우는 양함수 꼴로 나타내어라 (a) y yet, (b) yy y() e t t, 6 (c) y 6et y, y(5) y() log 5 (d) y t, y (e) y t + sec t, y (f) ty + y y, y() y() 5 y() 어떤 인구의 변화는 다음과 같은 미분 방정식으로 표현된다 dp P 8P ( ) dt 56 (a) 어떠한 P 의 값에 대해서 인구가 증가하는가? (b) 어떠한 P 의 값에 대해서 인구가 감소하는가? (c) 인구증가율이 가장 클 때의 인구는 얼마인가? ( 5) 반감기 세슘, 탄소-, 우라늄과 같은 방사성 물질들은 불안정하여 안정적인 물질로 붕괴된다 방사성 물질의 상대붕괴율 dm m dt 는 상수임이 알려져 있다 다시 말하면, 시각 t에서 방사성 물질의 양 m(t)는 다음 미분방 정식을 만족한다 dm km (k : 양의 실수 ) dt 방사성 물질의 양이 반이 되는데 걸리는 시간을 반감기(half life)라고 한다 세슘-7(cesium) 의 반감기는 년이다 지금 mg의 세슘 샘플이 있다고 할 때, (a) t년 후에 남아 있는 세슘의 질량을 구하여라 (b) 년 후에는 얼마나 많은 양의 세슘 샘플이 남아 있는가? (c) mg 만이 남게 되는 것은 몇 년 후인가?

200 CHAPTER 적분과 응용 96 5 토리노의 수의는 길이 약 피트, 폭 약 5피트의 직포로 만들어져 있는데 손과 발에 상처 입은 사람의 형상이 찍혀 있고 혈흔과 땀이 분석되었다 가톨릭 교회에서는 이를 예수의 형상이라고 주장하고 있다 988년 행해진 실험 결과 방 사성 탄소-의 9 9 %만이 이 수의에 잔존하는 것으로 밝혀졌다 이 실험 결과에 의하면 가톨릭 교회의 주장은 근거가 있다고 할 수 있는가? 탄소-의 반감기는 57 년이다 ( ) 6 바다의 수면 아래 깊이 미터 지점의 빛의 강도 I는 미분방정식 di I d 를 만족한다 (a) 수면에서의 빛의 강도를 I 라 할 때, 빛의 강도가 절반이 되는 깊이는 얼마인가? (b) m 지점에서의 빛의 강도는 I 의 몇 배인가? (c) 표면에서의 빛의 강도의 %가 되는 깊이는 얼마인가? 7 설탕이 물에 녹을 때, 아직 녹지 않은 설탕의 양을 A라 하면, A는 미분방정식 da/dt ka (k > ) 을 만족시킨다고 한다 설탕을 물에 넣은지 분만에 5%의 설탕이 녹았다고 할 때, 절반의 설탕이 녹는데 걸리는 시간은 얼마인가? 8 냉장고에서 꺼낸지 분 된 음료수의 온도가 8 C 이다 분 후 다시 재 보았더니 음료수 의 온도는 C이었다면, 냉장고 내부의 온도는 얼마이겠는가? 단, 방의 온도는 C를 유지하고 있다고 한다( ) 9 온도가 C인 방안에 놓여 있던 온도계를 C인 집 밖으로 가지고 나갔다 분이 지났을 때, 온도계는 C를 가리키고 있었다 (a) 분이 더 지났을 때, 온도계는 몇 도를 가리키고 있을까? (b) 6 C를 가리킬 때는 집 밖으로 나간지 몇 분 후인가? 함수 P (t)를 학습 시간에 따른 기술의 습득 정도라고 하자 이 함수 P 의 해곡선을 학 습 곡선(learning curve)이라고 한다 함수 P 가 다음의 미분방정식에 의해 주어진다고 하자 dp k(m P ) dt

201 부정적분의 응용: 변수분리법 97 여기서 k, M 은 양수인 상수이다 (a) 미분방정식을 풀고, 그 해를 이용해서 학습 곡선을 그려보아라 단, < P () < M 이다 (b) 어떤 공장의 생산 라인에 두 명의 새 노동자가 투입되었다 A는 첫 시간에 부품 5 개를 조립하였고, 두 번째 시간에는 5개를 조립하였다 B 는 첫 시간에 5개를 조립 하였고, 두 번째 시간에는 5개를 조립하였다 P () 이라고 가정하고 이 모형과 (a)의 결과를 이용하여 둘 다 숙련공이 되었을 때 누가 더 많은 부품을 조립할 수 있는지 판단하여라 (a) 연이율 r%이고, 이자는 연속복리로 계산되는 계좌에 A 원을 입금하였다 그리고 일년에 Q원을 입금하는데 모형을 간단하게 하기 위해서, 이 입금 역시 (일년에 몇 회 일어나는 대신) 연속적으로 이루어진다고 가정하자 A(t)를 t년 후에 이 계좌의 가치라고 할 때, 이 함수가 만족하는 초깃값문제를 구하여라 (b) K씨는 결혼 주년에 세계일주 여행을 계획하고 있다 년 후 세계여행 비용은 5만원으로 예상된다 K씨의 계획은 결혼식 날부터 일년에 Q원씩을 연속적으로 입금하고 주년 되는 날 계좌의 모든 금액을 인출하는 것이다 연 이율이 8%일 때, K의 계획이 성공적이려면 Q는 얼마이어야 하는가? 몸무게가 6kg 삼순이는 하루 섭취열량을 8kcal로 제한하는 다이어트를 시작하였다 삼순이의 기초대사량(basal metabolic rate) 은 kcal 이고 하루에 kg 당 5kcal 의 에너지가 소모된다 kg의 지방은 9kcal의 에너지로 변환된다고 한다 (a) 다이어트를 시작하고 t일 후 삼순이의 몸무게를 y(t)라고 할 때 y(t)가 만족하는 미분방정식을 세우고 해를 구하여라 (b) 삼순이의 몸무게가 처음으로 58kg 이하가 되는 것은 며칠 후인가? 다이어트를 계속 한다면 삼순이의 몸무게는 어떻게 되겠는가? 잠에서 깨어난 상태에서 절대안정을 유지할 때의 대사를 기초대사라 하며, 이 때 개체가 소비하는 에너지량을 기초대사량이라고 한다

202 CHAPTER 적분과 응용 98 정적분과 넓이 도형의 넓이를 구하는 것은 수학에서 가장 오래된 중요한 문제 중의 하나이다 직사각형의 넓 이는 가로와 세로의 곱으로 구할 수 있고 삼각형의 넓이는 밑변과 높이의 곱의 로 구할 수 있다 w l h b bh A A lw 일반적으로 도형의 경계가 직선인 경우는 삼각형의 넓이를 이용하면 항상 그 넓이를 구할 수 있다 그러나 경계가 곡선이면 넓이를 구하는 문제는 그리 단순하지가 않다 이 절에서는 곡선으로 둘러 싸인 도형의 넓이를 구하는 방법을 소개하도록 한다 정적분과 넓이 연속함수 y f ()가 구간 [a, b]에서 f () 일 때, 곡선 y f ()와 축, 그리고 두 수직선 a, b로 둘러싸인 영역의 넓이를 구하여 보 자 구간 [a, b]를 n등분하여 양 끝점과 각 분점의 좌표를 ( a),,,, n, n ( b) a b n 이라 하면 부분구간의 길이는 k k b a n () 이다 구간 h i k k [k, k ] a + (b a), a + (b a) n n 에서 어느 점 k 를 잡더라도 다음 직사각형의 넓이의 합 f ( ) + + f ( n ) n X () f ( k ) k 는 n 일 때 일정한 값으로 수렴한다는 것이 알려져 있다 이 극한값을 구간 [a, b]에서 함수 Rb f ()의 정적분(definite integral)이라고 하고 a f () d 로 나타낸다 b n X f () d lim f ( k ) () a 삼각형은 n 세 변의 길이를 알면 그 넓이를 구할 수 있다 k

203 정적분과 넓이 이때 Rb a 99 f () d가 주어진 영역의 넓이가 됨을 정의로부터 쉽게 이해할 수 있다 곡선에 의하여 정의된 영역의 넓이 y f ()( )가 연속함수일 때, 곡선 y f ()와 축, 그리고 두 수직선 a, b로 둘러 싸인 영역의 넓이는 lim n Rb 으로 정의하고 Note a n X f ( k ), k b a n f () d로 나타낸다 여기서 k k k 이다 연속함수 y f ()가 구간 [a, b]에서 f () 이라고 하자 그러면 b f () d a 이면 f () 이다 보기 다음 극한값을 정적분을 이용하여 나타내고 그 값을 구하여라 n r k X lim n n n k Solution 구간 [, ]을 n등분하였을 때 k번째 구간의 오른쪽 끝점 k k 이고, 이 때 부분구간의 길이는 n 이다 따라서 p y f () k n는 라고 하면 n X lim n n k r k n lim n n X f (k ) 정의역의 다음 적분값은 어떤 영역의 넓이를 나타내는지 설명하고 적분값을 구하여라 모든 에 대하여 f () 이면 f () 로 정의한다 반지름이 인 반원, π (b) 밑변이, 높이가 인 직각삼각형, (c) 윗변과 아랫변이 각각, 이고 높이가 인 직각사다리꼴, (a) - k R 이다 이 극한값은 정적분 d로 나타낼 수 있고, 구간 [,]에서 y f () 과 축 사이의 영역, 즉, 사분원의 넓이이다 따라서 주어진 극한값은 π 이다 기초다지기 y

204 CHAPTER 적분과 응용 (a) p d 보기 ( ) d (b) d (c) 연속함수 f ()에 대하여 f () a, f () b, f ()d A 라고 하자 f 의 역함수 g f 가 존재할 때 a g()d를 구하여라 단, b > a > 이다 Solution y f ()의 그래프는 점 (, a)와 점 (, b)를 지나고 순증가하는 곡선이다 또한 y g()의 그래프는 y f ()의 그래프와 직선 y 에 대하여 대칭이다 정 R Rb 적분 f () d, a g() d의 값은 각각 위의 그림에서 어두운 부분의 넓이 A, B 이고 두 넓이를 합하면 네 꼭지점 (, ), (, ), (, b), (, a)인 직사각형의 넓이 b가 된다 b f () d + g() d b y y f HL b y ghl a Rb a Note 따라서 Rb a a g() d b A이다 b 조각적으로 연속인 함수의 정적분은 구간별 적분의 합으로 정의한다 보기 함수 g() []에 대하여 5 g() d를 구하여라 여기서 []는 를 넘지 않는 최대정수를 나타낸다 y - Solution 주어진 정적분은 어두운 부분의 넓이이다 5 g() d g() d + g() d g() d

205 정적분과 넓이 정적분의 성질 연속인 함수 y f ()에 대하여 식 ()에서 극한값은 항상 존재한다는 것이 알려져 있다 그 극한값을 함수 f 의 [a, b]에서의 리만적분 (Riemann integral), 또는 정적분 (definite integral)이라 한다5 연속함수 f 의 구간 [a, b]에서의 정적분 n X b f () d lim n a 여기서 b a n 이고 f ( k ) (5) k k k k 이다 정적분은 극한값으로 정의되므로 극한값의 성질들로부터 다음과 같은 정적분의 성질들이 성립함을 보일 수 있다 정적분의 성질 I 구간 [a, b]에서 연속인 두 함수 f (), g()에 대하여 다음 관계식이 성립한다 a b f () d f () d b a b c d c(b a) a b [f () ± g()] d f () d ± a b a b 기초다지기 g() d(복호동순) a b f () d(c는 상수) cf () d c a b a 연속인 두 함수 f (), g()에 대하여 f () d, g() d 일 때, 다음을 구하여라6 5 적분을 R 나타내는 기호 는 합을 나타내는 문자 S를 길게 늘어뜨린 모양이다 이 기호는 독일의 수학자 라이프니 츠에 의하여 고안되었다 6 (a) 6 (b) (c) 7

206 CHAPTER 적분과 응용 f () d (a) g() d (b) [f () + g()] d (c) 미분과 적분은 서로 역작용으로 이해할 수 있다 다시 말해서, 어떤 함수를 적분한 후 미분하 면 원래의 함수가 되고, 반대로 어떤 함수를 미분한 것을 다시 적분하면 원래의 함수가 된다7 이러한 원리를 미적분의 기본정리(fundamental theorem of calculus)라고 한다 미적분의 기본정리 연속함수 f ()에 대하여 다음 등식이 성립한다 d f (t) dt f () d a R Note 연속함수 y f ()에 대하여 F () a f (t) dt는 미분가능한 함수이다 미적분의 기본정리는 다음과 같은 형태로 나타낼 수도 있다 u u u 보기 어라 h h 연속함수 y f ()에 대하여 F () d (a) d Solution a f (t) dt Ra f (t) dt d d +h R a f (t) dt f () f (t) dt이다 f 를 이용하여 다음을 나타내 d (b) d (a) f (t) dt lim lim R # f (t) dt a f (t) dt F ()이므로 a " a d f (t) dt [ F ()] f () d 이다 (b) R a f (t) dt F ( )이므로 d d " # f (t) dt a 이다 7 엄밀하게 d [F ( )] f ( )( ) f ( ) d 말하자면 원래의 함수 더하기 상수

207 정적분과 넓이 함수 f ()가 세 점 a, b, c를 포함하는 구간에서 연속이라고 하자 F ()가 f ()의 역도함수 라면 c b f () d + f () d F (c) F (a) + F (b) F (c) a c F (b) F (a) b f () d a 이 성립한다 이 관계식은 a, b, c의 대소관계에 상관없이 항상 성립한다 정적분의 성질 II b c f () d a f () d + a b f () d c 다음 그림은 a c b인 경우 f () 일 때의 상황을 그래프로 나타낸 것이다 y f HL a Note c b 정적분의 성질 II는 다음과 같이 차의 형태로도 나타낼 수 있다 b b c f () d f () d f () d c a a 함수가 구간마다 다르게 정의되어 있으면 전체 구간에서 역도함수를 구하는 것이 어려워진다 이런 경우 구간별로 정적분을 구하여 더하면 전체 정적분의 값을 구할 수 있다 보기 5 정적분 d의 값을 구하여라 ( Solution, 이므로, d ( ) d + ( ) d i i h + i h + h

208 CHAPTER 적분과 응용 기초다지기 구간 [, ]에서 y f ()의 그래프는 원 점에 대하여 대칭이고 다음과 같다 f () d, f () d, f () d y 일 때 (a) 다음 값을 구하여라 i f () d ii f () d iii f () d iv f () d (b) F () f (t) dt에 대하여 F ()가 극댓값을 갖는 의 값과, 극솟값을 갖는 의 값을 구하여라 (c) F ()가 최댓값을 갖는 의 값과, 최솟값을 갖는 의 값을 구하고 최댓값과 최솟값을 구하여라 정적분의 계산 미적분의 기본정리를 이용하여 정적분의 값을 어떻게 구하는지 살펴보기로 한다 f ()의 역도 함수를 하나 찾았다고 하자 다시 말해서, 어떤 함수 F ()가 d F () f () d 를 만족한다고 하자 미적분의 기본정리에서 d f (t) dt f () d a 이므로 모든 에 대하여 d h d i f (t) dt F () f () f () a 이다 도함수가 인 함수는 상수함수이므로 f (t) dt F () C a 이다 a이면 F (a) C이므로 f (t) dt F () F (a) a 가 성립한다 b로 놓으면 다음 결과를 얻는다 8 (a) (i) (ii) (iii) (iv) (b) 극댓값 :, 일 때, 극솟값 :,, 일 때 (c) ±일 때 최댓값, ±일 때 최솟값

209 정적분과 넓이 5 정적분의 계산 함수 f ()가 구간 [a, b]에서 연속이라고 하자 함수 F ()가 f ()의 역도함수이면 b f () d F (b) F (a) a 이다 b F (b) F (a)를 [F ()]ba, Note 또는, F () 로 나타내기도 한다 이 기호에 의하면 a F () f () b f () d [F ()]ba a 로 쓸 수 있다 보기 6 다음 정적분을 구하여라 (a) d log e d (b) 기초다지기 ( ) ( ) e log log e e [9 ] 다음 정적분을 구하여라9 (a) ( + ) d ( + ) d (e) (c) 8 (a) (g) e d log 5 (f) d log 9 ( + + ) d (b) 6 (d) e d Solution d (a) log (b) (b) 8 (h) (c) (i) 8 (d) 96 (j) (e) (k) (e ) (f) (l) 6π e d

210 CHAPTER 적분과 응용 6 π (g) π (h) cos d (i) ( ) d + ( + ) d π (cos + ) d + (l) π (sin + ) d d + + d + (e ) d (e + ) d (k) (j) sin d 두 곡선 사이의 영역의 넓이 양의 함수 y f ()에 대하여 b f () d a 는 구간 [a, b]에서 곡선 y f ()의 그래프와 축 y 사이의 넓이이다 구간 [a, b]에서 두 곡선 사이의 영역의 넓이도 같은 방법으로 구할 수 있다 함수 y f ()와 y g()는 모두 연속이고 f () g()라고 가정하자 구간 [a, b]에 대하여 와 ( a),,,, n, n ( b) 가 ()와 ()로 주어진다고 하자 구간 [k, k ] 의 임의의 점 k 에 대하여 리만합 n X [f ( k ) g( k )] k 은 주어진 영역의 넓이 A의 근삿값이 되며 n이 커질수록 더 좋은 근삿값을 구하여 준다 y y y f HL y f HL f Isk M - f Isk-M a k- k b a b y ghl D y ghl n 일 때, f, g가 연속함수이므로 극한값이 존재하는데 그 극한값을 주어진 영역의 넓이로 정의한다 n X A lim [f ( k ) g( k )] n 이 극한값은 정적분 Rb a k (f () g())d로 정의되므로 다음과 같은 결과를 얻는다

211 정적분과 넓이 7 두 곡선 사이의 넓이 f () g()인 두 연속함수 y f (), y g()와 두 수직선 a, b로 둘러싸인 영역의 넓이는 다음과 같다 b (f () g()) d A a 보기 7 구간 [, ]에서 두 곡선 y e 와 y 넓이를 구하여라 사이의 y Solution 주어진 구간에서 e 이므로 구하는 넓이 는 e d log + e y ã log log + e e log + e e ; 95 이다 y ã- 기초다지기 주어진 구간에서 두 곡선으로 이루어지는 영역의 넓이를 구하여라 (a) y, y, [, ] (c) y, y /, [, ] (b) y, y, [, ] (d) y, y /, [, ] 두 함수의 대소관계가 구간에 따라 변하는 경우는 구간에 따라 넓이를 구하여 모두 더한다 보기 8 두 함수의 그래프 f () + 과 g() + + 으로 둘러싸인 부분의 넓이를 구하여라 Solution 두 곡선이 만나는 점의 좌표를 구하자 f () g() ( )( + ) 이므로 두 곡선은,, 에서 만나고, 두 곡선으로 둘러싸인 영역은 두 개의 영역 S, S 로 분리된다 (a) 6 (b) 5 6 (c) log 8 (d) log

212 CHAPTER 적분과 응용 8 구간 [, ]에서는 f () g()이고 구간 [, ]에서는 f () g()이므로 Si 의 넓이 Ai 는 각각 다음과 같이 구할 수 있다 A (f () g()) d ( ) d ( 8) A (g() f ()) d ( ) d (8 ) y 5 y - + S S - y 따라서 두 곡선으로 둘러싸인 영역의 넓이는 A + A 8이다 Note 구간별로 두 함수의 대소관계가 달라지는 경우, f () g()인 구간에서는 f () g() 를, f () g()인 구간에서는 g() f () (f () g())를 적분한다 따라서 피적분함수는 다음과 같이 절댓값을 이용하여 하나의 함수로 나타낼 수 있다 ( f () g(), f () g() f () g() (f () g()), f () g() 두 곡선 사이의 영역의 넓이 두 곡선 y f (), y g()와 a, b로 둘러싸인 영역의 넓이는 b f () g() d a 이다 주어진 구간에서 두 곡선으로 이루어지는 영역의 넓이를 구하여라 기초다지기 (a) y, y, [, ] (b) y, y /, [, ] 보기 9 f () sin 의 역함수를 y g()라고 하자 구간 [, π]에서 두 곡선 y f () 와 y g()로 둘러싸인 영역의 넓이를 구하여라 Solution (a) 두 곡선은 y 에 대하여 대칭이다 (b) 9 8

213 정적분과 넓이 9 y y Π y ghl y - sinhl Π 그러므로 구하는 영역의 넓이는 다음과 같다 π π ( ( sin )) d sin d ( cos ) 보기 사분면에서 곡선 y π, 축과 y 로 둘러싸인 영역의 넓이를 구하여라 Solution 주어진 영역은 구간 [, ]에서의 영역 S 과 구 간 [, ]에서의 영역 S 로 구분할 수 있다 영역 S 의 넓이 A 은 구간 [, ]에서 y 와 축 사이의 넓이이므로 A d / + / 이다 영역 S 의 넓이 A 는 구간 [, ]에서 y 아래, y 위의 영역의 넓이이므로 y y y - - / A ( ( )) d 이다 따라서 구하는 넓이 A는 A A + A + 이다 5 y에 대한 적분 함수를 나타낼 때 우리는 일반적으로 y f ()로 표시한다 이때, 독립변수 를 수평축에, 종 속변수 y를 수직축에 나타낸다 그러나 를 항상 독립변수로 간주해야 할 필요는 없다 를 y 의 함수로 다루는 것이 더 편할 때에는 y 를 독립변수로 간주해도 된다

214 CHAPTER 적분과 응용 예를 들어, 보기 7의 경우 주어진 영역은 두 곡선 y, 와 y, y 로 둘러싸인 영역으로 나타낼 수 있다 따라서 주어진 영역의 넓이는 다음과 같이 구할 수도 있다 A [(y + ) y ] dy y + y y 8 + y+ y 보기 y - y+ 세 부등식 y, y, y 로 둘러싸인 영역의 넓이를 구하여라 y y y - y y y Solution 주어진 넓이를 에 대한 적분으로 구하려면(위 왼쪽 그림) 두 영역으로( 와 ) 나누어서 구해야 한다 그러나 y에 대한 적분을 이용하면(위 오른쪽 그림) 하나의 영역으로 계산이 가능하다 이때, 주어진 영역은 y의 구간 [, ]에서 두 곡선 y, y 으로 둘러싸인 영역으로 나타낼 수 있으므로 구하는 넓이는 h i (y)/ y ( y y) dy 8 이다

215 정적분과 넓이 연습문제 다음 극한값을 정적분을 이용하여 나타내어라( ) n + n o n n + n + (a) lim n n n n n n + n o n n n + (b) lim n n n n n 다음 극한값을 정적분을 이용하여 나타내어라 (a) lim n n X k n X (n + k) n n n k n (c) lim k n X (k) + n n n n X k + (b) lim n n n (d) lim k k 다음 극한값을 구하여라 lim n n n+n n n+ n n+ 역함수가 존재하는 연속함수 y f ()에 대하여 다음 조건이 성립한다 a f (), f (a) b, f ()d A, a >, b > (a) A의 범위를 구하여라 b (b) g f 일 때 g() d를 구하여라 5 다음 정적분을 구하여라(절의 연습문제 이용) (g) (e + e ) d dy y (b) (et + ) dt (h) ( ) d (e) 6 d π (f) cos d (i) + d (c) ( s + s) ds b 6 f 가 구간 [a, b] (a < b)에서 연속인 함수이고 f () d 이면 f (c) 인 c [a, b]가 (a) (et + ) d (d) 존재함을 보여라 7 다음 정적분의 값을 구하여라 y a

216 CHAPTER 적분과 응용 π (b) ( + ) d + ( ) d (c) ( + d (e) d + (d) cos d (a) d (f) ) d d ++ 8 구간 [, ]에서 함수 y f ()의 그래프는 y축에 대하여 대칭이고 다음과 같다 y - (a) - i f () d f () d 5일 때 다음 값을 구하여라 ii f () d iii f () d 5, f (t) dt 5, - f () d iv f () d (b) F () f () d 에 대하여 F () 가 극댓값을 갖는 의 값과, 극솟값을 갖는 의 값을 구하여라 (c) F () 가 최댓값을 갖는 의 값과, 최솟값을 갖는 의 값을 구하고 최댓값과 최솟값을 구하여라 9 절의 연습문제 의 결과를 이용하여 다음 정적분을 구하여라 (a) + d (d) (b) ( ) d cos d e d (f) log d e ( )e d e (g) log (e) π/ (c) (h) (log + ) d e d 주어진 구간에서 다음 곡선과 축 사이의 영역의 넓이를 구하여라 (a) y +, (b) y cos sin, 두 곡선 y, (c) y e, π (d) y e e, log y 으로 둘러싸인 부분의 넓이를 구하여라( )

217 정적분과 넓이 y y - y - 다음 곡선 또는 직선에 의하여 만들어지는 영역의 넓이를 구하여라 (a) y 5, y (b) y +, y ( + )/ (d) y,, (e) y, + y (f) y, y, y + 6 (c) y, y y 8 주어진 구간에서 두 곡선 사이의 넓이를 구하여라 (a) y, y +, (b) y e, y e+, log 5 (c) y sec, y 8 cos, π π (d) y sec, y tan, π π 다음 부등식에 의하여 만들어지는 영역의 넓이를 구하여라 (a) y /,, (b) + y, (c) y /, (d) y, y, y y ( ) y, + y, y, y 5 f () + 의 역함수가 존재함을 보여라 g() f ()라고 할 때, 사분면에서 두 곡선 y f ()와 y g() 로 둘러싸인 부분의 넓이를 구하여라 R 6 함수 f () t(t )(t ) dt의 극값을 구하여라 R 7 연속함수 f 에 대하여 F () a f (t) dt일 때 다음을 f 를 이용하여 나타내어라 (a) Hint d d f (t) dt + tan sec (b) d d " a # f (t ) dt

218 CHAPTER 적분과 응용 적분기법과 응용 5 치환적분법 5 F (u)가 f (u)의 한 부정적분이라면 부정적분의 치환적분법에 의하면 u g()일 때, f (g())g () d f (u) du F (u) F (g()) 이다 따라서 b f (g())g () d F (g(b)) F (g(a)) a g(b) f (u) du g(a) 을 얻는다 정적분의 치환적분법 구간 (a, b)에서 g 이 연속이고 f 가 u g()의 치역에서 연속이면 다음 식이 성립한다 b g(b) f (g())g () d f (u) du a 보기 g(a) 다음 정적분을 구하여라(절의 보기 5 와 같은 함수들이다) log d e d e (c) d (a) /π (b) /π sin d (d) Solution (a) u g() 이라고 하면 du d이고 g(), g() 이다 따라서 d (u + ) u du 을 얻는다 5 u/ + u/ du 5/ u + u/

219 5 적분기법과 응용 5 (b) u g() 이라고 하면 du d이고 g( π ) π, g( π ) /π π/ sin sin u du d /π π π/ π cos π cos cos u π 이므로 π 을 얻는다 (c) u g() log 로 놓으면 du d이고 g(), g(e) 이다 따라서 e log u du u d 이다 (d) u g() d 이므로 로 놓으면 g(), g() 이다 du e d eu du eu (e e) 이다 기초다지기 다음 정적분을 구하여라 ( ) d (a) (c) (b) e d (e) ( ) d (d) + d d ( ) (f) d + 대칭함수의 적분 5 적분구간이 원점에 대하여 대칭인 경우, 즉, 적분구간이 [ a, a]인 경우 피적분함수의 대칭성을 이용하면 정적분을 간단히 구할 수 있다 연속함수 f 에 대하여 정적분의 성질을 사용하면 a a f () d f () d + f () d a a a f () d + a f () d 이다 마지막 식의 첫번째 정적분에서 u 로 치환하면 du d이고 a일 때 u a 가 된다 따라서 a a f () d f ( u)( du) a f ( u) du (a) 78 (b) 6 (c) 5 (d) (e) e (f) log

220 CHAPTER 적분과 응용 6 Ra Ra 가 성립한다 y f ()가 우함수이면, 즉, f ( u) f (u)이면 f ( u) du f (u) du이므로 a a f () d f () d a Ra Ra 이다 반면에 y f ()가 기함수이면, 즉, f ( ) f ()이면 f ( u) du f (u) du 이므로 a a a f () d f (u) du + f () d a 이다 y y A -A A A 대칭함수의 정적분 구간 [ a, a]에서 연속인 함수 f 에 대하여 Ra Ra f 가 우함수이면(f ( ) f ()), a f () d f () d Ra f 가 기함수이면(f ( ) f ()), a f () d 보기 다음 정적분을 구하여라 (a) ( ) d Solution sin d + (a) y n 은 n이 홀수이면 기함수, 짝수이면 우함수이므로 5 ( + + ) d ( ) d 을 얻는다 (b)

221 5 적분기법과 응용 (b) f () sin + 라고 7 하면 f 는 모든 실수에 대하여 연속이고 sin( ) sin + ( ) + f ( ) 이므로 f 는 기함수이다 따라서 sin d + 이다 다음 정적분을 구하여라 기초다지기 (a) + + d (c) cos d (d) sin d e d (f) log 5 π e d log 5 log 5 log 5 (e) π (b) log( + ) d 정적분의 부분적분법 5 R f ()g() [f ()g() + f ()g ()] d이므로 미적분의 기본정리를 적용하면 b f ()g() b [f ()g() + f ()g ()] d a a b b [f ()g ()] d [f ()g()] d + a a 이다 따라서 정적분의 부분적분법은 다음과 같이 쓸 수 있다 정적분의 부분적분법 b f ()g () d f ()g() 보기 (a) a 다음 정적분의 값을 구하여라 e d (b) (a) 8 f ()g() d a a b b (b) (c) (d) (e) 8 (f) π/ e sin d

222 CHAPTER 적분과 응용 8 Solution (a) u, dv e d로 놓으면 du d, v e 이다 e e e d e e e e + (e + ) d 를 얻는다 (b) 절 보기 6의 결과를 이용하면 π/ e sin d π/ e (sin cos ) i π h π/ π e (sin cos ) e (sin cos ) π/ [e + ] 를 얻는다 5 함수의 평균 수직선 위를 움직이는 물체가 있을 때, 시점 t에서 이 물체의 위치와 속도를 각각 s(t), v(t)라고 하자 v(t) s (t) 이므로 시점 t 에서 시점 t 까지 위치의 변화는 t s(t ) s(t ) v(t) dt t 이다 또한 시점 t 에서 시 t 까지 이 물체가 움직인 거리는 t v(t) dt t 가 된다 평균속력은 주어진 시간 동안 움직인 거리를 시간으로 나누면 구할 수 있다 보기 자동차가 출발한지 t초 후의 속도가 v(t) +t (m/sec) 로 주어진다고 한다 처음 분 동안 이 자동차의 평균속력(초속)을 구하여라

223 5 적분기법과 응용 Solution 9 모든 t 에 대하여 v(t) 이므로 이 경우 분동안 움직인 거리는 dt (t + t) +t ( ) ( ) 이다 즉, 분 동안 움직인 거리는 mkm이다 따라서 자동차의 처음 분 동안의 평균속 력은 분 동안 움직인 거리 km를 분, 즉, (초)로 나눈 것이다 따라서 5 d (m/sec) + 이다 유한 개의 수 y, y,, yn 의 평균 y는 다음과 같이 정의한다 y y + y + + yn n (6) 그렇다면 연속인 구간에서 함수의 평균은 어떻게 정의할까? 만약 a b에서 y f () 이라면 그래프와 축 사이의 영역의 넓이는 b A f () d a A 이다 또한 A는 밑변의 길이가 (b a)이고 높이가 b a 인 직사각형의 넓이이기도 하다 여기서 A b a 을 구간 [a, b]에서 함수 y f ()의 평균(average)으로 정의한다 일반적인 연속함수 y f ()의 평균도 같은 방법으로 정의한다 함수의 평균 연속함수 y f ()의 구간 [a, b]에서의 평균은 다음과 같이 정의한다 f b a Note b f () d a 함수 f 가 다음과 같이 조각적으로 정의되었다고 하자 f () yk, k < k, (k,, n) y yn- y yn y y n- n

224 CHAPTER 적분과 응용 그러면 구간 [a, b] [, n]에서 함수 f 의 평균 f b a b f ()d a 는 식 (6)에서 정의된 y, y,, yn 의 평균 y와 일치한다 기초다지기 주어진 구간에서 다음 함수의 평균을 구하여라5 (a) y, [, ] (b) y, (c) y e, [, ] (d) y sin, [, ] [, π] 보기 5 S 은행에서는 년 만기 예금에 대하여 연 %의 이자를 연속복리로 지급하고 VIP 고객에게는 평균잔고(년 동안)의 %를 보너스로 지급한다 VIP 로 선정된 삼식이가 만원 짜리 년 만기 적금에 가입하였을 만기시 수령금액은 얼마인지 구하여라 Solution 적금 가입 후 t에서 자산의 가치는 A(t) et 이다 따라서 년 후 잔고는 A() e8 ; 887 이다 년 동안의 평균잔고는 t A e et dt 5(e8 ) ; 이므로 총 수령액은 (만원) 이다 55 연속흐름 정유회사의 수입은 유전에서 채취하는 원유의 양에 비례한다고 하면 유전회사의 수입은 연속적 인 흐름으로 간주할 수 있다 다시 말해서, y f (t)가 시간 t에서 원유의 유입량의 비율이라면 시간 k까지 채취된 원유의 양은 k f (t) dt 로 나타낼 수 있다 이렇게 유입 또는 유출이 연속적으로 일어나는 경우 연속흐름(continuous stream)이라고 한다 5 (a) (b) (c) (e e ) (d) π

225 5 적분기법과 응용 보기 6 비율이 정유회사에서 원유를 연속적으로 채취하여 얻는 수입은 시간 t(단위 : 년)에서 그 f (t) 5( t) (억원/년) 으로 주어진다고 한다 이 유전으로부터 년동안 얻을 수 있는 총수입을 구하여라 Solution 년 동안의 총수입은 5( t) dt t ) 5( ) 75 5(t 이다 보기 6에서 k년 동안 원유를 채취할 권리를 판매한다고 하자 연이율은 r이고 연속복리로 지급된다고 하자 그러면 이 권리의 적정한 판매가는 무엇이 될까? 시간의 구간을 n개의 부분구간으로 나누자 i번째 구간에서 얻어지는 수입은 약 f (ti ) t 이고 이 수입의 현재가치는 f (ti ) te rti 이다 따라서 연속수입의 현재가치는 근사적으로 n X f (ti )e rti t i 이고 n 이면 이 값은 k f (t)e rt dt 로 주어진다 연속수입의 현재가치 연속수입율이 f (t)이고 연속복리율이 r일 때 t 부터 t k까지 연속수입의 현재가치는 k f (t)e rt dt 이다 보기 7 보기 6에서 5년 동안 원유를 채취할 권리의 현재가치는 얼마인가? 점단, 이율은 연 5%의 연속복리라고 하고 반올림하여 억 단위까지 구하여라

226 CHAPTER 적분과 응용 Solution r 5이므로 5 년동안 원유를 채취할 권리의 현재가치는 te 5t dt e 5t dt 5 5( t)e 5t dt 5 V + V 이다 V 5 5 e 5t dt ( e 5 5 5t e 5 5 ) 이다 두번째 적분을 구하기 위하여 우선 u 5t로 놓으면 5 5 u 5t te dt eu ( du) ueu du 가 된다 따라서 보기 (b) 의 결과를 이용하면 V 5 te 5t dt 5 ueu du (ueu eu ) 5 ( e 5 + ) 을 얻는다 따라서 구하는 가격은 V + V 이다 e 5 + 5e e 5 ; 6 (억원)

227 5 적분기법과 응용 연습문제 5 다음 정적분을 구하여라 p ( + ) 9 d 다음 정적분을 구하여라 π (a) cos( ) d π cos dt (b) t t + d (c) (d) ( + ) d p a d, a> a (e) (f) p d a (g) a sin cos d + 하루 동안의 온도가 다음과 같은 함수로 주어진다고 πt 6 sin, f (t) 6 cos π(t 6), 9 π(t 5) + 6 cos 8, 한다 t 6 6 t 5 5 t 이 날의 평균온도는 얼마인가? ( ) Solution 온도의 그래프를 그려 보면 다음과 같다 연속함수 y f ()에 대하여 (a) 다음 식을 만족하는 c가 a와 b 사이에 존재함을 보여라6 b f () d f (c) b a a (b) y f ()의 평균 f 에 대하여 f f (c) 인 c가 a와 b 사이에 존재함을 설명하여라 b (c) u(t) (f () t) d라고 하자 u(t)는 t f 일 때 최솟값을 가짐을 보여라7 a 5 함수 f () 의 구간 [, ]에서의 평균 f 을 구하여라 6 어떤 연금상품은 은퇴와 동시에 매년 D의 연금을 년간 지급한다고 한다 연금이 연속 적으로 지급된다고 가정할 때 은퇴와 동시에 일시금으로 받고자 한다면 얼마를 받으면 되는가? 단, 이율은 연 5%의 연속복리로 지급된다고 한다 6 이 결과를 정적분의 평균값정리라고 한다 u(t)는 t에 관한 이차식이다 7 Hint

228 CHAPTER 적분과 응용 7 아이스크림 전문점의 매출(단위 : 백만원)은 f (t) cos πt, ( t ) 로 주어진다고 한다 (t의 단위 : 년) 매출이 연속적으로 일어난다고 가정할 때, (a) 년 동안 총매출은 얼마인가? (b) 매출과 동시에 입금된다면 년 후(t ) 잔고는 얼마인가? 단, 이자는 연 5%의 연속복리로 지급된다고 한다 (c) 년 동안 매출액의 월 일의(t ) 현재가치는 얼마인가? 단, 이자는 연 5%의 연속복리로 지급된다고 한다 반올림하여 만단위까지 구하여라 8 연속함수 f 와 상수 c에 대하여 다음 식이 성립함을 보여라 b (a) b+c f ( + c) d f () d a c (b) c f (c ) d f () d a+c 9 연속함수 f 에 대하여 다음이 성립함을 보여라 π π f (sin ) d (a) f () f (cos ) d R dt일 +t 때 (a) f ()를 구하여라 (b) f 가 존재함을 보여라 (c) (f ) ()을 구하여라 (b) π f (sin ) d π f (sin ) d

229 6 특이적분 6 5 특이적분 Rb 지금까지 우리는 유한한 닫힌 구간 [a, b]에서 정의된 연속함수 f 에 대하여 정적분 a f ()d 을 정의하였다 이 절에서는 두 가지 조건 중에서 적어도 하나가 성립하지 않는 경우의 정적분을 정의한다 이러한 경우의 정적분을 특이적분(improper integral)이라 부른다 6 무한구간에서 정의된 특이적분 사분면에서 곡선 y e 아래 있는 영역을 생각해 보자 y y ã- 임의의 양수 t에 대하여 구간 [, t]에서의 넓이를 A(t)라고 하자 이 영역의 넓이는 t 일 때 A(t)의 극한값으로 정의한다 정적분의 정의에 의하여 A(t) t e d e t e t 이다 t 일 때, A(t)의 값은 로 수렴한다 따라서 빗금친 부분의 넓이는 이다 일반적으로 무한구간 [a, )에서 정의된 연속함수 f 에 대하여 limt R 면 a f () d을 다음과 같이 정의한다 f () d lim a t Rt a f () d가 존재하 t f () d a R 이 극한값이 존재할 때 특이적분 a f () d는 수렴한다(converge)고 하고 극한값이 존재하지 Ra 않을 때는 발산한다(diverge)고 한다 특이적분 f () d도 비슷한 방법으로 정의한다

230 CHAPTER 적분과 응용 6 무한구간에서의 특이적분 무한구간 [a, )에서 정의된 연속함수 f 에 대하여 특이적분 같이 정의한다 t f () d f () d lim t a t a f () d는 다음과 a 무한구간 (, b]에서 정의된 연속함수 f 에 대하여 특이적분 같이 정의한다 b b f () d f () d lim R Rb f () d는 다음과 t R Rb 위의 두 극한값이 존재할 때, 특이적분 a f () d과 f () d 는 수렴한다고 한다 수렴하지 않는 경우는 발산한다고 한다 Ra R 연속함수 f 에 대하여 특이적분 f () d와 a f () d가 모두 수렴하면 R f () d를 다음과 같이 정의한다 a f () d 기초다지기 (a) 보기 f () d + f () d a 다음 특이적분을 구하여라8 d (b) 특이적분 Solution d (c) e d d이 수렴하는 p의 범위를 구하여라 p 이면 /p 는 연속이다 p 일 때 t h it d log log t 이므로 특이적분은 발산한다 p 6 이면 t t h it p p d d p p (t p ) p 이다 p > 이면 p < 이므로 t 일 때 t p 이다 그러므로 t d lim d, p> p p t p 8 (a) (b) (c) (d) (d) e d

231 6 특이적분 7 이다 반면 p < 이면 t 일 때 t p 이므로 특이적분은 발산한다 따라서 주어진 특이적분은 p > 일 때만 수렴한다 보기 특이적분 log d가 수렴하는지 판단하고 수렴한다면 적분값을 구하여라 Solution 구간 [, )에서 피적분함수 log 는 연속이다 u log 로 치환하면 du 또한 eu 이므로 log t log t t u log d du ue u du eu d 이다 을 얻는다 부분적분을 이용하면 u u u u ue du ue ( e )du ue + e u du ue u e u + C 이다 따라서 log t h ue u du ue u e u ilog t log t (log t)e e log t + log t + t t 이고 이 값은 t 일 때 로 수렴한다 따라서 다음 결과를 얻는다 t log log d lim d t 혼동의 위험이 없으면 lim F (t) Note t 순화하여 풀이를 쉽게 해준다 보기 (a) t 대신 F (t) 로 쓰기도 한다 이 표현은 계산을 단 a a 다음 특이적분이 수렴하는지 판단하고 수렴한다면 적분값을 구하여라 log d (b) e d Solution (a) 구간 [, )에서 피적분함수 log 는 연속이다 u log 로 치환하면 일 때, u log 이고 du d 이므로 log d udu u 을 얻는다 따라서 특이적분은 발산한다

232 CHAPTER 적분과 응용 8 R (b) f () e 이라고 하면 f 는 기함수이다 그러므로 f () d가 수렴한다면 주어진 R 특이적분의 값은 이다 이제 f () d이 수렴하는지 살펴 보자 u로 치환하면 일 때, u log 이고 d du이므로 d e u du e e u 이고 구하는 특이적분의 값은 이다 6 불연속함수의 특이적분 특이적분의 다른 형태는 피적분함수가 유한한 구간에서 정의되지만 구간의 한점에서 불연속인 경우에 정의된다 예를 들어, 사분면에서 곡선 y 와 직선 에 의하여 만들어지는 영역을 생각해 보자 y y 우선 < t < 일 때, 구간 [t, ]에서 영역의 넓이를 구하면 다음과 같다 d t + ( t) t 주어진 영역의 넓이는 t +일 때의 극한값으로 정의한다 즉, d lim ( t) t + 이다 일반적으로 함수 f () 가 구간 (a, b]에서 연속이고 a에서 불연속이면9 특이적분 Rb f ()d는 다음과 같이 정의한다 a b f () d lim a t a+ b f () d t 함수 f () 가 구간 [a, b)에서 연속이고 b에서 불연속인 경우도 같은 방법으로 정의한다 9 엄밀히 말하자면 a에서 불연속이거나 정의되지 않으면

233 6 특이적분 9 불연속함수의 특이적분 함수 f () 가 구간 (a, b]에서 연속이고 a에서 불연속이면 특이적분 는 다음과 같이 정의한다 b b t a f ()d f () d f () d lim t b Rb t a f ()d f () d t a+ 함수 f () 가 구간 [a, b)에서 연속이고 b에서 불연속이면 특이적분 는 다음과 같이 정의한다 a b f () d lim a Rb a Rb 위의 두 극한값이 존재할 때, 특이적분 a f ()d은 수렴한다(converge)고 한다 수렴하지 않는 경우는 발산한다(diverge)고 한다 함수 f ()가 구간 [a, c)와 구간 (c, b] (a < c < b) 에서 각각 연속이라고 하자 특이 Rc Rb Rb 적분 a f ()d와 c f ()d가 수렴하면 a f ()d는 다음과 같이 정의한다 b f () d a c f () d + a b f () d c Note 마지막 결과는 a, b 에도 성립한다 즉, f 가 c 에서 불연속이고 특이적 Rc R R 분 f ()d와 c f ()d가 수렴하면 f ()d는 다음과 같이 정의한다 c f ()d f () d + f () d 보기 c 다음 특이적분이 수렴하는지 살펴 보아라 (a) Solution d (b) d (a) 에서 는 정의되지 않으므로 d lim d lim log t + t + t lim ( log t) t t + 이고 주어진 특이적분은 발산한다 (b) 특이적분 Note R d이 발산하므로 주어진 특이적분 위 보기의 (b)에서 R d도 발산한다

234 CHAPTER 적분과 응용 f () 는 기함수이므로 R d 이다 Rc 라고 결론을 내려서는 안된다 불연속점 c가 구간 안에 있는 경우, 특이적분 a f ()d와 Rb Rb f ()d가 모두 수렴하는 경우에만 특이적분 a f () d 는 수렴한다고 정의한다 c 보기 5 / d을 구하여라 함수 f () / 는 에서 정의되지 않으며 수직점근선을 갖는다 따라서 Solution / d / / d d + 을 계산하여야 한다 / d / d t / d lim t lim t / / t / lim (t ( )/ ) lim / d t t + t / t + / lim / lim ( t + t / (t) ) (/ ) 이므로 다음 값을 얻는다 / d + (/ ) ( + / ) 다음 특이적분이 수렴하면 그 값을 구하여라 기초다지기 (a) d (c) (b) d (d) 6 d d (e) (f) d d 특이적분의 비교판정법 특이적분의 정확한 값을 구하는 것은 대부분의 경우 아주 어렵거나 불가능하다 이런 경우 특이적분의 수렴여부는 다른 함수의 특이적분과 비교하여 판정할 수 있다 이러한 판정법을 (a) (b) (c) 발산 (d) 발산 (e) (f) 발산

235 6 특이적분 특이적분에 대한 비교판정법(comparison test)이라고 한다 비교판정법 두 연속함수 f, g가 a일 때 f () g() 이라고 하자 그러면 f () d가 수렴하면 g() d도 수렴한다 a a g() d가 발산하면 f () d도 발산한다 a a a, 또는, b 인 경우에도 비교판정법은 성립한다 Note 양의 값을 갖는 함수의 적분은 함수의 그래프와 축 사이의 넓이로 이해할 수 있으므로 비교판정법은 작은 함수의 넓이가 큰 함수의 넓이보다 작다는 사실로부터 유추할 수 있다 보기 6 다음 특이적분의 수렴여부를 판정하여라 (a) Solution d sin (a) (b) d sin < < 이면 < sin 이므로 sin 이다 d lim log lim ( log(t)) t + t t + t + t 이므로 비교판정법에 의하여 주어진 특이적분은 발산한다 lim (b) R d는 수렴하지만 이 경우 부등식 sin 은 비교판정법에서는 아무 역할을 하지 못한다 작은 함수의 특이적분이 수렴한다고 해서 큰 함수의 특이적분도 수렴한다고는 할 수 없기 때문이다 따라서 다른 방향의 부등식이 필요하다 y sin 는 구간 [, π/] 에서 (sin ) sin 이므로 위로 볼록하다 따라서 y sin 의 그래프는 두 점 (, )과 (π/, sin(π/)) (π/, ) 를 잇는 직선 y π 보다 위에 있다 즉, 다음 부등식이 성립한다 sin, π [, π ]

236 CHAPTER 적분과 응용 y y Π Π 따라서 sin pπ Π 이고 d < 이므로 주어진 특이적분은 비교판정법에 의하여 수렴한다 보기 7 특이적분 e / d는 수렴함을 보여라 Solution R e / d는 유한한 구간에서 연속인 함수의 정적분이므로 유한한 값을 갖는다 e / 이므로 R e / d와 e / d + R d e / e / d d는 동시에 수렴하거나 동시에 발산한다 이면 e 이고 / e / e / d e / 이므로 비교판정법에 의하여 특이적분 e / < e d 는 수렴한다

237 6 특이적분 연습문제 6 다음 적분이 발산하면 D라고 적고, 수렴하면 그 값을 구하여라 (a) d ( + ) (b) sin d (c) log d d π tan d (g) e d (d) log d (f) (e) (h) d + 비교판정법을 이용하여 다음 적분이 수렴하면 C, 발산하면 D라고 적어라 (a) (b) d sin d sin (c) e d (g) sin d (h) sin (d) d d (e) + e e (f) e d (i) e d (j) e d d e + 다음 특이적분이 수렴하면 C, 발산하면 D라고 적어라 (a) (b) d sin e d e d (c) sin d (d) 다음 특이적분이 수렴하는 p 의 범위를 구하고 적분값을 구하여라 (a) e d (log )p (b) p log d 5 다음 관계식이 성립함을 보이고 n이 자연수일 때 왼쪽 특이적분의 값을 구하여라 n e d n n e d

238 CHAPTER 적분과 응용 7 연속확률변수 7 확률밀도함수 연속확률변수에 대하여는 어느 한 점에서의 확률을 계산하는 것이 아니라 어느 구간에 속할 확률을 구한다 어떤 함수 f : R R에 대하여 f () R f () d P(a X b) Rb a f () d 가 성립하면 X를 연속확률변수(continuous random variable)라고 하고, f 를 X 의 확률밀도 함수(probability density function)라고 한다 확률밀도함수는 어는 값에서 그 확률변수가 일어날 가능성을 수치적으로 나타낸다 보기 지하철 역에서 지하철이 5분에 한 대씩 들어온다고 한다 당신이 지하철에 도착하였 을 때, 바로 앞의 지하철이 언제 떠났는지 모른다면 분 이내에 다음 지하철이 도착할 확률은 얼마인가? Solution X를 지하철역 도착 후 다음 지하철이 올 때까지의 시간이라고 하자 그러면 X 5이고 X는 이 구간에서 모두 같은 정도의 가능성을 갖고 있다 따라서 X의 확률밀도함 수를 f ()라고 하면 ( c, [, 5] f (), 기타 이다 f ()는 확률밀도함수이므로 5 f () d c d 5c c 5 이다 따라서 분 이내에 치하철이 도착할 확률은 P( X ) f () d 5 5 이다 보기 은 균일분포의 한 예이다 일반적인 정의는 다음과 같다

239 7 연속확률변수 5 균일분포 연속확률변수 X가 어떤 구간 [a, b]의 값을 취하며 구간 안의 어느 점에서도 일어날 가능 성이 같을 때, X는 구간 [a, b] 에서 균일분포(uniform distribution)를 갖는다고 한다 이때 확률밀도함수는 다음과 같다 ( b a, [a, b] f (), 기타 보기 확률변수 X의 확률밀도함수 f 가 다음과 같다 ( f () ce,,, 기타 c의 값을 구하고 확률 P(X )을 구하여라 Solution f 는 확률밀도함수이므로 f () d 이어야 한다 즉, f () d R c e d i h c c e 이므로 c 이고 구하는 확률은 P(X ) e d e e ; 867 이 된다 y y ã- 보기 의 확률밀도함수를 갖는 확률변수는 모수가 인 지수분포(eponential distribution)를 따른다고 한다 일반적인 지수분포의 정의는 다음과 같다

240 CHAPTER 적분과 응용 6 지수분포 X의 확률밀도함수가 양수 λ > 에 대하여 ( λe λ, f (), 기타 일 때, X는 모수가 λ인 지수분포를 따른다고 한다 Note 지수분포는 자동차 배터리의 수명, 서비스 시간, 암 수술을 받은 환자의 생존기간 등 수명이나 어떤 사건이 일어날 때까지의 시간을 모형화하기에 적합하다 7 누적분포함수 연속확률변수 X의 확률밀도함수가 f 라고 하자 X의 누적분포함수(cumulative distribution function)는 F () P(X ) f (t) dt 로 정의한다 F 는 증가함수이고 다음 성질을 갖는다 F () f () lim F () lim F () 누적분포함수의 정의로부터 P(a < X b) F (b) F (a) 이다 누적분포함수와 확률밀도함수 f (), F ()가 각각 연속확률변수 X의 확률밀도함수와 누적분포함수이면 F () f (t)dt, F () f () 이고 X의 확률은 다음과 같이 구할 수 있다 P(a X b) P(a < X b) f ()d a 일반적인 확률변수에 대하여는 첫번째 등식으로 정의한다 b

241 7 연속확률변수 보기 7 연속확률변수 X가 모수가 인 지수분포를 따를 때 (보기 참조) (a) X의 누적분포함수를 구하여라 (b) Y X 의 확률밀도함수를 구하여라 Solution (a) X의 누적분포함수를 F () P(X ) 라고 하면 < 에 대하여 F () 이다 이면 F () t e e t dt f (t)dt e 이다 (b) Y 의 누적분포함수를 G(y) P(Y y), 확률밀도함수를 g(y) G (y)라고 하자 y 이면 G(y) P(X y) g(y) 이다 y > 이면 G(y) P(X y) P( y X y) P(X y) F ( y) e y 이고 g(y) G (y) ( e 이다 정리하면 ( g(y) ) e y y y e y, y y >,, 기타 이다 7 기댓값과 분산 f ()가 연속확률변수 X의 확률밀도함수일 때 X의 기댓값 µ E(X) 는 다음과 같이 정의된다 µ E(X) f () d 마찬가지로 g(x)의 기댓값은 다음과 같이 구할 수 있다 E(g(X)) g()f () d

242 CHAPTER 적분과 응용 8 연속확률변수의 기댓값 f ()가 연속확률변수 X의 확률밀도함수일 때 X의 기댓값 µ E(X) 는 다음과 같이 정의된다 f () d µ E(X) g(x)의 기댓값은 다음과 같이 구할 수 있다 E(g(X)) g()f () d Note R X의 확률밀도함수가 f ()라고 하자 f () d 이므로 E(aX + b) (a + b)f () d a f () d + b f () d ae(x) + b 이다 분산 X의 기댓값 µ E(X)는 확률분포의 무게중심을 나타낸다 다시 말해서 E(X)는 분포의 중심위치에 대한 측도라고 할 수 있다 그러나 같은 기댓값을 갖는 확률변수도 확률분포에 따라 그 성질이 상당히 다를 수 있다 확률밀도함수가 다음과 같이 주어진 확률변수 X, Y 를 생각 해 보자 X, Y 의 기댓값은 모두 이지만 Y 의 확률분포가 X의 그것보다 훨씬 중심(기댓값) 으로부터 훨씬 넓게 퍼져 있음을 알 수 있다 y - y Y 의 분포 X의 분포 이러한 분포가 중심위치 µ E(X)로부터 퍼져 있는 정도를 나타내는 측도로써 E((X µ) ) 을 사용한다 이를 X의 분산(variance) 이라고 하고 기호로는 σ, 또는, Var(X)

243 7 연속확률변수 9 로 나타낸다 X의 확률밀도함수가 f ()이면 분산은 다음과 같이 구할 수 있다 ( µ) f () d Var(X) E((X µ) ) f () d f () d µ f () d + µ E(X ) µ + µ E(X ) µ 분산 µ E(X)일 때 X의 분산은 σ Var(X)은 다음과 같이 정의한다 σ Var(X) E((X µ) ) E(X ) µ Note Var(aX + b) E((aX + b) ) (aµ + b) a E(X ) a µ a Var(X) 보기 Solution 보기 에서 확률변수 X의 기댓값과 분산을 구하여라 확률변수 X의 기댓값은 E(X) e d f () d 이다 부분적분법을 이용하면(e d dv, u ) e E(X) e d + e 이다 X의 분산을 구하기 위해서는 E(X ) e d 을 구하여야 한다 다시 부분적분법을 이용하면(e d dv, u ) E(X ) e + e d + E(X) 이다 따라서 분산은 Var(X) E(X ) (E(X)) 이다

244 CHAPTER 적분과 응용 보기 의 결과는 다음과 같이 일반화할 수 있다 증명은 연습문제로 남긴다 지수분포의 평균과 분산 X가 모수가 λ인 지수분포를 따르면 평균과 분산은 각각 다음과 같다 E(X), λ Var(X) λ Note 모수 λ는 기댓값의 역수이다 따라서 λ가 커지면 기댓값은 작아지고 λ가 작아지면 기댓 값은 커진다 X가 어떤 위험한 사건이 일어날 때까지의 시간이나 사망할 때까지의 시간이라고 하자 그러면 X의 기댓값이 크면 사건이 일어날 가능성은 작아지고 따라서 위험도는 줄어든다 반대로 E(X)가 작다면, 즉, λ가 크다면 사건이 짧은 시간 내에 일어남을 의미하므로 위험도는 커진다 이런 의미에서 모수 λ 를 위험계수(hazard rate) 라고 부른다 7 정규분포 연속확률분포 X의 확률밀도함수가 f () ( µ) ep, σ πσ < < 일 때 X는 모수가 (µ, σ )인 정규분포(normal distribution)를 따른다고 하고 X N(µ, σ ) 으로 나타낸다 이 때 X의 기댓값과 분산은 각각 µ, σ 이라는 것이 알려져 있다 정규분포의 기댓값과 분산 X N (µ, σ )이면 E(X) µ, Var(X) σ 이다 정규분포의 확률밀도함수는 기댓값과 분산에 의하여 결정되는데 곡선의 형태가 µ와 σ가 달라질 때 어떻게 변화하는지 아래 그림에서 보기로 하자 왼쪽 그림은 기댓값(µ )은 동 일하지만 분산이 다른 세 정규분포(σ, σ, σ 9)를 보여 준다 분산이 작을수록 분포는 기댓값축에 집중된다 오른쪽 그림은 분산이 같고(σ ) 기댓값이 다른 경우인데 세 경우(µ, µ, µ ) 모두 곡선의 모양은 같고 중심축의 위치만 다르다

245 7 연속확률변수 f HL f HL - - 위의 그림에서 볼 수 있듯이 정규분포의 확률밀도함수는 다음과 같은 특징을 갖는다 종 모양의 연속함수이다 µ를 대칭축으로 하여 좌우대칭이다 µ에서 최대값 /( π σ)을 갖는다 X가 평균 µ, 분산 σ 인 확률변수라하자 E(aX + b) ae(x) + b Var(aX + b) a Var(X) 이므로 확률변수 X µ σ 의 평균은, 분산은 이다 이러한 변환을 하는 과정을 표준화한다(standardize)고 표현한다 보기 5 X가 평균이 µ, 분산이 σ 인 정규분포를 따를 때, 정규분포를 따름을 보여라 Solution X µ σ 는 평균이, 분산이 인 의 확률밀도함수가 z f (z) e π 임을 보이면 된다 의 누적분포함수를 F (z) P( z) 라고 하자 P( z) P X µ z σ σz+µ P(X σz + µ) ( µ) e σ d σ π

246 CHAPTER 적분과 응용 에서 u µ σ 로 치환하면 d σdu 이고 P( z) z u e σdu σ π z u e du π 이다 따라서 f (z) F (z) z u d e du dz π z e π 이다 보기 5처럼 평균이 이고 분산이 인 정규분포를 특히 표준정규분포(standard normal distribution)라 하고, 그 확률변수를 로 표기한다 N (, ) 표준정규확률변수 가 구간 [a, b]의 값을 취할 확률은 b e d P(a b) π a 이고, 이 값은 다음 그림에서 어두운 영역의 면적에 해당한다 f HL a b 일반적으로 위의 적분은 초등적인 방법으로는 구할 수 없다 그러나 이에 대한 수표(부록 정 규분포표)가 만들어져 있어 수표를 이용하여 표준정규분포의 확률을 구할 수 있다 표에 주어진 값은 주어진 점 z( )의 왼쪽 부분의 면적이다 즉, 주어진 값 z에 대하여 z Φ(z) P( z) e d π 의 값이 구하여져 있다 FHzL z

247 7 연속확률변수 일반적으로 표준정규분포의 확률값은 표준확률분포표와 다음 관계식을 이용하여 찾을 수 있다 표준정규분포의 성질 P( > z) P( < z) P( < z) P( > z) P( < z) P(a < < b) P( < b) P( < a) 보기 6 가 표준정규확률변수일 때 다음의 확률을 구하여라 (a) P( < ) (b) P( < ) (c) P( < ) Solution (a) 표준정규분포표에서 이 확률은 약 8 이다 (b) P( < ) P( < ) P( < ) P( < ) 95 (c) P( < ) P( > ) P( < )

248 CHAPTER 적분과 응용 연습문제 7 X가 구간 [a, b]에서 균일분포를 갖는다 (a) X의 누적분포함수 F () P(X )를 구하여라 (b) X의 평균과 분산을 구하여라 나누어심기를 한 나무의 수명은 확률밀도함수가 다음과 같다 ( ce, f () 기타 (a) 상수 c를 구하여라 (b) 이 나무의 평균수명을 구하여라 어떤 전기제품의 수명(단위 : 년)의 확률밀도함수가 다음과 같다 8, a f (), < a (a) 상수 a를 구하여라 (b) 이 제품의 수명의 평균을 구하여라 (c) 신상품을 구입하였을 경우 년 이상 사용할 수 있을 확률을 구하여라 X가 모수가 인 지수분포를 따를 때, (a) Y X 의 확률밀도함수를 구하여라 (b) Y 의 평균과 분산을 구하여라 5 X가 모수가 λ인 지수분포를 따르면 (a) s, t 에 대하여 다음 관계식이 성립함을 보여라 P(X > t + s) P(X > t)p(x > s) (b) 평균과 분산은 각각 다음과 같음을 보여라 E(X), λ Var(X) λ 6 정규분포 X N (µ, σ ) 의 확률밀도함수 f 에 대하여 µ ± σ 는 변곡점이 됨을 보여라 7 (a) 표준정규분포의 평균이 임을 보여라 다시 말해서 다음 등식이 성립함을 보여라 e / d π

249 7 연속확률변수 5 (b) 표준정규분포의 분산이 임을 보여라 다시 말해서 다음 등식이 성립함을 보여라 e / d π 8 가 표준정규분포를 따른다 (a) Y 의 확률밀도함수를 구하여라 (b) Y 의 평균과 분산을 구하여라 9 다음 등식을 이용하여 주어진 적분값을 구하여라 / e e / d d, π π (a) e d (b) e d 확률변수 X 의 누적분포함수가 다음과 같다 ( e, F (), < (a) P(X > ), P( < X < ) 를 구하여라 (b) X 의 평균과 분산을 구하여라 Hint R e / d q π 확률밀도함수는 f () e 이다 π q R R π e / d e / d 의

250 CHAPTER 적분과 응용 6 8 부피 가로, 세로, 높이가 각각 w, l, h인 직육면체의 부피는 V wlh 이다 또한 반지름이 r, 높이가 h인 원기둥의 부피는 V πr h 이다 r h h l w 직육면체나 원기둥과 같이 윗면과 밑면이 합동이고 두 면을 수직으로 연결한 입체도형을 기둥면(cylinder)이라고 한다 기둥면의 부피는 밑면의 넓이 A와 높이 h의 곱으로 주어진다 V A h 8 부피 이 사실을 이용하여 일반적인 입체도형의 부피를 구하여 보기로 한다 a와 b 사이에 있는 입체도형 S의 축에 수직인 단면의 넓이가 연속함수 A()로 주어졌다고 하자 구간 [a, b] 을 n등분하여 양 끝점과 각 분점의 좌표를 ( a),,,, n, n ( b) 이라 하면 부분구간의 길이는 b a n 이다 k번째 구간의 입체 Sk 의 부피는 단면적이 A( k ) ( k [k, k ])이고 높이는 인 기둥면의 부피로 근사시킬 수 있다 V (Sk ) ; A( k ) y AHHk L*L k- k

251 8 부피 7 따라서 입체의 부피는 이 부분기둥들의 부피의 합으로 근사시킬 수 있다 V ; Vn n X A( k ) k n이 커질수록 근삿값은 좋아지며 그 극한값이 존재하는데 그 극한값으로 입체의 부피를 정의 한다 입체의 부피 a와 b 사이에 있는 입체도형 S의 축에 수직인 단면의 넓이를 A()라고 하자 A()가 연속함수이면 S의 부피는 다음과 같다 V lim n 보기 Solution n X A( k ) b A() d a k 밑면의 한 변이 r인 정사각형이고 높이가 h인 사각뿔(pyramid)의 부피를 구하여라 사각뿔을 다음과 같이 그리자 y y r r s h h 에서 단면의 한 변의 길이를 s라고 하면 삼각형의 닯음비에서 r h:r:s s h 이다 따라서 단면의 넓이는 A() 이고 사각뿔의 부피는 다음과 같다5 h A() d 5 일반적으로 r h h r d h r h d r h h 기둥면과 같은 모양의 밑면을 갖는 뿔면의 부피는 기둥면 부피의 이다

252 CHAPTER 적분과 응용 8 보기 반지름이 r인 구의 부피를 구하여라 Solution 이다 구의 중심을 원점으로 잡자 에서의 단면은 원이고 이 원의 반지름은 y r y r r - s r 단면의 넓이는 p A() π( r ) π(r ) 이다 따라서 구의 부피는 r A() d π (r ) d r r r h ir π (r ) d π r πr V r 이 된다 8 회전체의 부피 I 구는 원을 회전하면 얻어진다 이렇게 곡선을 회전하여 얻어지는 회전체의 부피는 단면이 항상 원이므로 단면의 넓이를 쉽게 구할 수 있다 다시 말해서, 곡선 y f (), a b를 축으로 회전하였을 때 생기는 회전체의 에서의 단면적은 A() π[f ()] 이므로 이 회전체의 부피는 다음과 같다 V b A() d π a a b [f ()] d

253 8 부피 9 회전체의 부피 I y f (), a b를 축으로 회전하여 생기는 회전체의 부피는 다음과 같다 b b [f ()] d y d π V π a a Note 마찬가지로 곡선 g(y), c y d를 y축 둘레로 회전한 회전체의 부피는 다음과 같이 주어진다 d d [g(y)] dy dy π V π c c 보기 다음 곡선을 각각 축과 y축 둘레로 회전하여 생기는 회전체의 부피를 구하여라 y Solution, 축 둘레로 회전하면 단면은 반지름이 yy y 인 원이다 y y 부피는 V π ( ) d π d 8π 이다 반면 y축 둘레로 회전하면 생기는 단면의 반지름은 y 이고 이 때 y 이다 y y y y y y 그러므로 부피는 V π 가 된다 dy π (y ) dy 5 π π 5 5

254 CHAPTER 적분과 응용 5 두 곡선으로 둘러 싸인 영역을 회전하여 생긴 회전체의 부피는 외부곡선으로 생긴 회전체의 부피에서 내부곡선에 의하여 생긴 회전체의 부피를 빼면 된다 보기 두 곡선 y, y, 으로 둘러 싸인 영역을 다음과 같이 회전했을 때 생기는 회전체의 부피를 구하여라 (a) 축 둘레 (b) y축 둘레 y y y Solution (a) 축 둘레로 회전하면 y 가 외부곡선이 되고 y 이 내부곡선이 된다 y y y y 따라서 첫번째 입체의 부피에서 두번째 입체의 부피를 빼면 V π () d π ( ) d h i 6 π π 5 5 y

255 8 부피 5 가 된다 (b) y축 둘레로 회전하면 y가 외부곡선이 되고 y 가 내부곡선이 된다 y y y y y는 부터 까지 변하므로 V π ( y) dy π y dy π π y ( ) dy y dy i y 6 8 π 8 π π h y 이다 8 회전체의 부피 II 직사각형 R을 회전하여 생기는 회전체의 부피는 가운 데 구멍이 난 원기둥 모양이다 따라서 그 부피는 바깥 원기둥의 부피에서 안쪽 원기둥의 부피를 빼면 된다 안 쪽 원의 반지름이 r, 바깥 원의 반지름이 r, 높이가 h 라면 이 회전체의 부피는 y r r V R πr h πr h π(r r )(r + r )h π rrh 가 된다 여기서 r r r, r r + r

256 CHAPTER 적분과 응용 5 이다 이 사실을 이용하여 부피를 구하는 새로운 방법을 생각해 보자 곡선 y f (), a b 을 y축 둘레로 회전한 회전체의 부피를 구하여 보기로 하자 f 가 일대일 함수이어서 역함수 g가 존재하면 g(y), c y d로 쓸 수 있고 d V π (g(y)) dy c 로 부피를 구할 수 있다 그러나 f 가 다음과 같이 일대일 함수가 아닌 경우는 이 방법을 쓸 수 없고 문제가 훨씬 복잡해진다6 y y y f HL y f HL a a b b 이제 회전체를 다음과 같이 잘라서 생각해 보자 구간 [a, b]을 n등분하여 양 끝점과 각 분점의 좌표를 ( a),,,, n, n ( b) 이라 하면 부분구간의 길이는 b a n 이다 k번째 영역을 높이가 f (k )인 사각형으로 근사시킬 수 있다 여기서 k k + k 이다 이 사각형을 y축 둘레로 회전하여 생긴 회전체는 바깥원의 반지름이 k, 안쪽 원의 반지 름은 k 이고 높이는 f (k )인 구멍이 난 원기둥이다 y D f H k L a 6 또한 k- k b 일대일함수에 대하여 역함수를 항상 구할 수 있는 것도 아니다

257 8 부피 5 따라서 그 부피는 Vk πk f (k ) 로 근사시킬 수 있다 따라서 회전체의 부피는 V ; n X Vk π k n X k f (k ) k 로 근사시킬 수 있고 n이 커지면 이 값은 b π f () d a 으로 수렴한다 따라서 다음 결과를 얻는다 회전체의 부피 II 곡선 y f (), a b를 y축으로 회전하여 생긴 회전체의 부피는 다음과 같다 V π b f () d a 보기 5 다음 곡선으로 둘러싸인 영역을 y축 둘레로 회전한 회전체의 부피를 구하여라 y,,, y y y Solution 이다 회전체의 부피를 구하는 두번째 공식을 이용하면 회전체의 부피는 π d π 보기 6 다음 곡선과 축으로 둘러 싸인 영역을 y축 둘레로 회전하여 생기는 회전체의 부피를 두 가지 방법으로 구하여라 y

258 CHAPTER 적분과 응용 5 Solution y 방법 외부곡선은 의 구간 [, ]에서 + p y, y 이고 이 곡선을 y축으로 회전한 회전체의 부피는 p V π ( + y) dy 이다 내부곡선은 의 구간 [, ]에서 p y, y 이고 이 곡선을 y축으로 회전한 회전체의 부피는 p V π ( y) dy 이다 따라서 구하는 부피는 V V V π p y dy π u du 8π 이다 방법 곡선은 축과, 에서 만나므로 구하는 부피는 V π ( ) d π ( ) d i π 6 8π π( ) h 이다

259 8 부피 55 연습문제 8 다음 곡선에 의하여 만들어지는 영역을 -축 둘레로 회전하여 생기는 회전체의 부피를 구하여라 (a) y, y π,, y π π (c) y sec, y,, (d) y e,,, y (b) y cos, (e) y,,, y (f) y, y ( ) + y 로 둘러싸인 영역을 y축 둘레로 회전시켜 생기는 회전체의 부피를 구하여라 다음 도형을 축 둘레로 회전시켜 생기는 입체의 부피를 구하여라 y e e, log 9 다음 곡선으로 둘러싸인 영역을 축과 y축 둘레로 회전시켜 생긴 회전체의 부피를 각각 구하여라 (a) y,, y (c) y, y (b) y /,,, y (d) y, y 5 다음 곡선 또는 직선에 의하여 만들어지는 영역을 주어진 직선 둘레로 회전하여 생기는 회전체의 부피를 구하여라 (a) y, y, y 둘레 (b) y, y, y 둘레 (c) y, y, y 둘레 (d) y,,, y, y 둘레 6 다음 곡선으로 둘러싸인 영역을 y축 둘레로 회전한 회전체의 부피를 구하여라 ( ) y sin, π, y

260 CHAPTER 적분과 응용 56 y y sinhl Π 7 원 ( b) + y a, ( < a < b)을 y축 둘레로 회전하면 도넛 모양의 회전체가 생긴다 이 회전체를 원환체(torus)라고 부른다 원환체의 부피를 구하여라 a b 8 다음 그래프를 y축 둘레로 회전한 회전체의 부피를 구하여라 sin, < π f (), y y Π sinhl Π 9 다음 곡선으로 둘러 싸인 영역을 y축 둘레로 회전한 회전체의 부피를 구하여라 (a) y +, y (b) y ( ), y (c) y sin( ) ( π), y (d) y e ( ), y

1 경영학을 위한 수학 Final Exam 2015/12/12(토) 13:00-15:00 풀이과정을 모두 명시하시오. 정리를 사용할 경우 명시하시오. 1. (각 6점) 다음 적분을 구하시오 Z 1 4 Z 1 (x + 1) dx (a) 1 (x 1)4 dx 1 Solut

1 경영학을 위한 수학 Final Exam 2015/12/12(토) 13:00-15:00 풀이과정을 모두 명시하시오. 정리를 사용할 경우 명시하시오. 1. (각 6점) 다음 적분을 구하시오 Z 1 4 Z 1 (x + 1) dx (a) 1 (x 1)4 dx 1 Solut 경영학을 위한 수학 Fial Eam 5//(토) :-5: 풀이과정을 모두 명시하시오. 정리를 사용할 경우 명시하시오.. (각 6점) 다음 적분을 구하시오 4 ( ) (a) ( )4 8 8 (b) d이 성립한다. d C C log log (c) 이다. 양변에 적분을 취하면 log C (d) 라 하자. 그러면 d 4이다. 9 9 4 / si (e) cos si

More information

<B4EBC7D0BCF6C7D02DBBEFB0A2C7D4BCF62E687770>

<B4EBC7D0BCF6C7D02DBBEFB0A2C7D4BCF62E687770> 삼각함수. 삼각함수의덧셈정리 삼각함수의덧셈정리 삼각함수 sin (α + β ), cos (α + β ), tan (α + β ) 등을 α 또는 β 의삼각함수로나 타낼수있다. 각 α 와각 β 에대하여 α >0, β >0이고 0 α - β < β 를만족한다고가정하 자. 다른경우에도같은방법으로증명할수있다. 각 α 와각 β 에대하여 θ = α - β 라고놓자. 위의그림에서원점에서거리가

More information

슬라이드 1

슬라이드 1 전자기학 도함수와미분법 도함수의응용 Prof. Jae Young Choi 전자기학 (015 Fall) Prof. Jae Young Choi 미분을배우는이유 영화속의미분과적분 스피드 3 3.1.1 함수의극한 극한 f(a) 의존재성과무관하게 a 의부근에있는 에서함수 f() 가정의될때 a f() L 이면, 가 a 에가까워질수록함숫값 f() 는 L 에수렴한다. lim

More information

= ``...(2011), , (.)''

= ``...(2011), , (.)'' Finance Lecture Note Series 사회과학과 수학 제2강. 미분 조 승 모2 영남대학교 경제금융학부 학습목표. 미분의 개념: 미분과 도함수의 개념에 대해 알아본다. : 실제로 미분을 어떻게 하는지 알아본다. : 극값의 개념을 알아보고 미분을 통해 어떻게 구하는지 알아본다. 4. 미분과 극한: 미분을 이용하여 극한값을 구하는 방법에 대해 알아본다.

More information

1 1 장. 함수와극한 1.1 함수를표현하는네가지방법 1.2 수학적모형 : 필수함수의목록 1.3 기존함수로부터새로운함수구하기 1.4 접선문제와속도문제 1.5 함수의극한 1.6 극한법칙을이용한극한계산 1.7 극한의엄밀한정의 1.8 연속

1 1 장. 함수와극한 1.1 함수를표현하는네가지방법 1.2 수학적모형 : 필수함수의목록 1.3 기존함수로부터새로운함수구하기 1.4 접선문제와속도문제 1.5 함수의극한 1.6 극한법칙을이용한극한계산 1.7 극한의엄밀한정의 1.8 연속 1 1 장. 함수와극한 1.1 함수를표현하는네가지방법 1.2 수학적모형 : 필수함수의목록 1.3 기존함수로부터새로운함수구하기 1.4 접선문제와속도문제 1.5 함수의극한 1.6 극한법칙을이용한극한계산 1.7 극한의엄밀한정의 1.8 연속 2 1.1 함수를표현하는네가지방법 함수 f : D E 는집합 D 의각원소 x 에집합 E 에속하는단하나의원소 f(x) 를 대응시키는규칙이다.

More information

제 3강 역함수의 미분과 로피탈의 정리

제 3강 역함수의 미분과 로피탈의 정리 제 3 강역함수의미분과로피탈의정리 역함수의미분 : 두실수 a b 와폐구갂 [ ab, ] 에서 -이고연속인함수 f 가 ( a, b) 미분가능하다고가정하자. 만일 f '( ) 0 이면역함수 f 은실수 f( ) 에서미분가능하고 ( f )'( f ( )) 이다. f '( ) 에서 증명 : 폐구갂 [ ab, ] 에서 -이고연속인함수 f 는증가함수이거나감소함수이다 (

More information

<3235B0AD20BCF6BFADC0C720B1D8C7D120C2FC20B0C5C1FE20322E687770>

<3235B0AD20BCF6BFADC0C720B1D8C7D120C2FC20B0C5C1FE20322E687770> 25 강. 수열의극한참거짓 2 두수열 { }, {b n } 의극한에대한 < 보기 > 의설명중옳은것을모두고르면? Ⅰ. < b n 이고 lim = 이면 lim b n =이다. Ⅱ. 두수열 { }, {b n } 이수렴할때 < b n 이면 lim < lim b n 이다. Ⅲ. lim b n =0이면 lim =0또는 lim b n =0이다. Ⅰ 2Ⅱ 3Ⅲ 4Ⅰ,Ⅱ 5Ⅰ,Ⅲ

More information

<B0F8BDC4C1A4B8AE2838C2F720BCF6C7D032292E687770>

<B0F8BDC4C1A4B8AE2838C2F720BCF6C7D032292E687770> 제 1 과방정식과부등식 분수방정식과고차방정식의연립방정식, 10단계와융합된계산문제, 고차부등식과분수부등식의연립부등식등다른내용과융합된계산문제를중심으로공부를해야한다. 방정식과부등식의풀이법을이해하고있는가를중심으로공부한다. 추론문제의경우증명과같은괄호를채우는문제를중심으로연습하는것이좋다 분수방정식, 무리방정식, 고차부등식, 분수부등식의각주제별로외적문제를구분지어연습해두어야한다.

More information

Vector Differential: 벡터 미분 Yonghee Lee October 17, 벡터미분의 표기 스칼라미분 벡터미분(Vector diffrential) 또는 행렬미분(Matrix differential)은 벡터와 행렬의 미분식에 대 한 표

Vector Differential: 벡터 미분 Yonghee Lee October 17, 벡터미분의 표기 스칼라미분 벡터미분(Vector diffrential) 또는 행렬미분(Matrix differential)은 벡터와 행렬의 미분식에 대 한 표 Vector Differential: 벡터 미분 Yonhee Lee October 7, 08 벡터미분의 표기 스칼라미분 벡터미분(Vector diffrential) 또는 행렬미분(Matrix differential)은 벡터와 행렬의 미분식에 대 한 표기법을 정의하는 방법이다 보통 스칼라(scalar)에 대한 미분은 일분수 함수 f : < < 또는 다변수 함수(function

More information

문제지 제시문 2 보이지 않는 영역에 대한 정보를 얻기 위하여 관측된 다른 정보를 분석하여 역으로 미 관측 영역 에 대한 정보를 얻을 수 있다. 가령 주어진 영역에 장애물이 있는 경우 한 끝 점에서 출발하여 다른 끝 점에 도달하는 최단 경로의 개수를 분석하여 장애물의

문제지 제시문 2 보이지 않는 영역에 대한 정보를 얻기 위하여 관측된 다른 정보를 분석하여 역으로 미 관측 영역 에 대한 정보를 얻을 수 있다. 가령 주어진 영역에 장애물이 있는 경우 한 끝 점에서 출발하여 다른 끝 점에 도달하는 최단 경로의 개수를 분석하여 장애물의 제시문 문제지 2015학년도 대학 신입학생 수시모집 일반전형 면접 및 구술고사 수학 제시문 1 하나의 동전을 던질 때, 앞면이나 뒷면이 나온다. 번째 던지기 전까지 뒷면이 나온 횟수를 라 하자( ). 처음 던지기 전 가진 점수를 점이라 하고, 번째 던졌을 때, 동전의 뒷면이 나오면 가지고 있던 점수를 그대로 두고, 동전의 앞면이 나오면 가지고 있던 점수를 배

More information

01

01 2019 학년도대학수학능력시험 9 월모의평가문제및정답 2019 학년도대학수학능력시험 9 월모의평가문제지 1 제 2 교시 5 지선다형 1. 두벡터, 모든성분의합은? [2 점 ] 에대하여벡터 의 3. 좌표공간의두점 A, B 에대하여선분 AB 를 로외분하는점의좌표가 일때, 의값은? [2점] 1 2 3 4 5 1 2 3 4 5 2. lim 의값은? [2점] 4. 두사건,

More information

<B1B9BEEE412E687770>

<B1B9BEEE412E687770> 201 학년도대학수학능력시험 6 월모의평가문제및정답 2016 학년도대학수학능력시험 6 월모의평가문제지 1 제 2 교시 5 지선다형 1. 두행렬 성분은? [2 점 ] 에대하여행렬 의 3. lim 의값은? [2점] 1 2 3 4 5 1 2 3 4 5 2. 의값은? [2점] 1 2 3 4 5 4. 공차가 인등차수열 에대하여 의값은? [3 점 ] 1 2 3 4 5

More information

연습문제 및 정답 연습문제 1.1, 주어진 벡터 a, b에 대하여 다음 벡터를 그려 보아라. (a) a + b (c) a b (b) 2a + b (d) b 2a b a Answer 생략 2. P, Q가 다음과 같을 때 벡터 P Q, QP 를 구하고 kp Q

연습문제 및 정답 연습문제 1.1, 주어진 벡터 a, b에 대하여 다음 벡터를 그려 보아라. (a) a + b (c) a b (b) 2a + b (d) b 2a b a Answer 생략 2. P, Q가 다음과 같을 때 벡터 P Q, QP 를 구하고 kp Q 연습문제 및 정답 연습문제.,.. 주어진 벡터 a, b에 대하여 다음 벡터를 그려 보아라. (a) a + b (c) a b a + b (d) b a b a Answer 생략. P, Q가 다음과 같을 때 벡터 P Q, QP 를 구하고 kp Qk, kqp k을 구하여라. (a) P = (, ), Q = (, ), P = (5, 7, ), Q = (, 9, ) Answer

More information

= " (2014), `` ,'' .." " (2011), `` ,'' (.)"

=  (2014), `` ,'' ..  (2011), `` ,'' (.) 학습목표 Finance Lectue Note Seies 파생금융상품의 이해 화폐의 시간가치(time value of money): 화폐의 시간가치에 대해 알아본다 제강 화폐의 시간가치 연금의 시간가치(time value of annuity): 일정기간 매년 동일금액을 지급하는 연금의 시간가치에 대해 알아본다 조 승 모 3 영구연금의 시간가치(time value

More information

5. 두함수 log 에대하여옳은것을 < 보기 > 에서모두고르면?5 ) ㄱ. ㄴ. ㄷ. < 보기 > 1 ㄴ 2 ㄷ 3 ㄱ, ㄴ 4 ㄴ, ㄷ 5 ㄱ, ㄴ, ㄷ 7. 인실수 에대하여 log 의지표를 이라할때, 옳 은것을보기에서모두고르면? ( 단, 는 를넘지않는최대의정수이다.

5. 두함수 log 에대하여옳은것을 < 보기 > 에서모두고르면?5 ) ㄱ. ㄴ. ㄷ. < 보기 > 1 ㄴ 2 ㄷ 3 ㄱ, ㄴ 4 ㄴ, ㄷ 5 ㄱ, ㄴ, ㄷ 7. 인실수 에대하여 log 의지표를 이라할때, 옳 은것을보기에서모두고르면? ( 단, 는 를넘지않는최대의정수이다. 제 2 교시 2008 년 5 월고 3 모의고사문제지 성명수험번호 3 1 먼저수험생이선택한응시유형의문제지인지확인하시오. 문제지에성명과수험번호를정확히기입하시오. 답안지에수험번호, 응시유형및답을표기할때는반드시 수험생이지켜야할일 에따라표기하시오. 단답형답의숫자에 0 이포함된경우, 0 을 OMR 답안지에반드시표기해야합니다. 문항에따라배점이다르니, 각물음의끝에표시된배점을참고하시오.

More information

FGB-P 학번수학과권혁준 2008 년 5 월 19 일 Lemma 1 p 를 C([0, 1]) 에속하는음수가되지않는함수라하자. 이때 y C 2 (0, 1) C([0, 1]) 가미분방정식 y (t) + p(t)y(t) = 0, t (0, 1), y(0)

FGB-P 학번수학과권혁준 2008 년 5 월 19 일 Lemma 1 p 를 C([0, 1]) 에속하는음수가되지않는함수라하자. 이때 y C 2 (0, 1) C([0, 1]) 가미분방정식 y (t) + p(t)y(t) = 0, t (0, 1), y(0) FGB-P8-3 8 학번수학과권혁준 8 년 5 월 9 일 Lemma p 를 C[, ] 에속하는음수가되지않는함수라하자. 이때 y C, C[, ] 가미분방정식 y t + ptyt, t,, y y 을만족하는해라고하면, y 는, 에서연속적인이계도함수를가지게확 장될수있다. Proof y 은 y 의도함수이므로미적분학의기본정리에의하여, y 은 y 의어떤원시 함수와적분상수의합으로표시될수있다.

More information

일반각과호도법 l 삼각함수와미분 1. 일반각 시초선 OX 로부터원점 O 를중심으로 만큼회전이동한위치에동경 OP 가있을때, XOP 의크기를나타내는각들을 ( 은정수 ) 로나타내고 OP 의일반각이라한다. 2. 라디안 rad 반지름과같은길이의호에대한중심각의 크기를 라디안이라한

일반각과호도법 l 삼각함수와미분 1. 일반각 시초선 OX 로부터원점 O 를중심으로 만큼회전이동한위치에동경 OP 가있을때, XOP 의크기를나타내는각들을 ( 은정수 ) 로나타내고 OP 의일반각이라한다. 2. 라디안 rad 반지름과같은길이의호에대한중심각의 크기를 라디안이라한 일반각과호도법 l 1. 일반각 시초선 OX 로부터원점 O 를중심으로 만큼회전이동한위치에동경 OP 가있을때, XOP 의크기를나타내는각들을 ( 은정수 ) 로나타내고 OP 의일반각이라한다. 2. 라디안 rad 반지름과같은길이의호에대한중심각의 크기를 라디안이라한다. 3. 호도법과육십분법 라디안 라디안 4. 부채꼴의호의길이와넓이 반지를의길이가 인원에서중심각이 인 부채꼴의호의길이를

More information

함수 좌표평면에서 함수 미적분 Ⅱ 1. 여러가지적분법 삼각함수의부정적분 의도함수가 sin 일때, 의값 은? [3점][2011( 가 ) 10월 / 교육청 4] 지수함수의부정적분 가모든실수에서연속일때, 도함수 가 > 이다. 일때, 의

함수 좌표평면에서 함수 미적분 Ⅱ 1. 여러가지적분법 삼각함수의부정적분 의도함수가 sin 일때, 의값 은? [3점][2011( 가 ) 10월 / 교육청 4] 지수함수의부정적분 가모든실수에서연속일때, 도함수 가 > 이다. 일때, 의 모든 연속함수 함수 1. 여러가지적분법 Ⅳ 적분법 1. 1. 여러가지적분법 01 부정적분과미분계수 02 ( 은실수 ) 의부정적분 실수 에서연속인함수 에대하여 이다. 일때, 의값을구하시오. [3점][2015(B) 4월 / 교육청 25] 4. 03 유리함수의부정적분 에대하여함수 이다. 함수 는다음조건을만족시킨다. ( 가 ) 두직선 는함수 의그래프의점근선이 다.

More information

제 12강 함수수열의 평등수렴

제 12강 함수수열의 평등수렴 제 강함수수열의평등수렴 함수의수열과극한 정의 ( 점별수렴 ): 주어진집합 과각각의자연수 에대하여함수 f : 이있다고가정하자. 이때 을집합 에서로가는함수의수열이라고한다. 모든 x 에대하여 f 수열 f ( x) lim f ( x) 가성립할때함수수열 { f } 이집합 에서함수 f 로수렴한다고한다. 또 함수 f 을집합 에서의함수수열 { f } 의극한 ( 함수 ) 이라고한다.

More information

제1장 군 제1절 소개와 예 제2절 이항연산 2.1 보기. 다음은 정수방정식 a + x = b를 푸는 과정이다. (1) 준식에 a를 더하여 ( a) + (a + x) = ( a) + b. (2) 결합법칙을 사용하면 (( a) + a) + x = ( a) + b. (3)

제1장 군 제1절 소개와 예 제2절 이항연산 2.1 보기. 다음은 정수방정식 a + x = b를 푸는 과정이다. (1) 준식에 a를 더하여 ( a) + (a + x) = ( a) + b. (2) 결합법칙을 사용하면 (( a) + a) + x = ( a) + b. (3) 제장 군 제절 소개와 예 제절 이항연산. 보기. 다음은 정수방정식 + x = b를 푸는 과정이다. () 준식에 를 더하여 ( ) + ( + x) = ( ) + b. () 결합법칙을 사용하면 (( ) + ) + x = ( ) + b. () ( ) + = 임을 이용하면 + x = ( ) + b. (4) + x = x 이므로 x = ( ) + b. 이를 유리수방정식

More information

6.6) 7.7) tan 8.8) 자연수 10.10) 부등식 두 의전개식에서 의계수는? ) 사건 에대하여 P P 일때, P 의값은? ( 단, 은 의여사건이다.) 일때, tan 의값은? log log 을만족시키

6.6) 7.7) tan 8.8) 자연수 10.10) 부등식 두 의전개식에서 의계수는? ) 사건 에대하여 P P 일때, P 의값은? ( 단, 은 의여사건이다.) 일때, tan 의값은? log log 을만족시키 1.1) 벡터 2.2) cos 함수 제 2 교시 2016 년 6 월고 3 모의고사문제지 성명수험번호 3 1 먼저수험생이선택한응시유형의문제지인지확인하시오. 문제지에성명과수험번호를정확히기입하시오. 답안지에수험번호, 응시유형및답을표기할때는반드시 수험생이지켜야할일 에따라표기하시오. 단답형답의숫자에 0 이포함된경우, 0 을 OMR 답안지에반드시표기해야합니다. 문항에따라배점이다르니,

More information

7. 인실수 에대하여 log 의지표를 이라할때, 옳 은것을보기에서모두고르면? ( 단, 는 를넘지않는최대의정수이다.) 7 ) ㄱ. log ㄴ. log 의지표는 이다. ㄷ. log log 이면 은 자리의정수 이다. 10. 다음은어느인터넷사이트의지도상단에있는버튼의기능을설명한

7. 인실수 에대하여 log 의지표를 이라할때, 옳 은것을보기에서모두고르면? ( 단, 는 를넘지않는최대의정수이다.) 7 ) ㄱ. log ㄴ. log 의지표는 이다. ㄷ. log log 이면 은 자리의정수 이다. 10. 다음은어느인터넷사이트의지도상단에있는버튼의기능을설명한 제 2 교시 2008 년 5 월고 3 모의고사문제지 성명수험번호 3 1 먼저수험생이선택한응시유형의문제지인지확인하시오. 문제지에성명과수험번호를정확히기입하시오. 답안지에수험번호, 응시유형및답을표기할때는반드시 수험생이지켜야할일 에따라표기하시오. 단답형답의숫자에 0 이포함된경우, 0 을 OMR 답안지에반드시표기해야합니다. 문항에따라배점이다르니, 각물음의끝에표시된배점을참고하시오.

More information

PSFZWLOTGJYU.hwp

PSFZWLOTGJYU.hwp 학년도대수능 9 월모의평가 ( 수리영역 - 가형 AH AT sin 8. log 9 log. log log 일때, ( 분모 ( 분자 이어야한다. 즉, ( +a-b+a-b a - b - ᄀ +a+b - (-(-b (-( ++ -b + + - b -b 9 ᄂ ᄀ, ᄂ에서 a, b 8 a+ b 5. log log X AB -B ( ( - - ( - ( 5 - -8

More information

함수공간 함수공간, 점열린위상 Definition 0.1. X와 Y 는임의의집합이고 F(X, Y ) 를 X에서 Y 로의모든함수족이라하자. 집합 F(X, Y ) 에위상을정의할때이것을함수공간 (function space) 이라한다. F(X, Y ) 는다음과같이적당한적집합과

함수공간 함수공간, 점열린위상 Definition 0.1. X와 Y 는임의의집합이고 F(X, Y ) 를 X에서 Y 로의모든함수족이라하자. 집합 F(X, Y ) 에위상을정의할때이것을함수공간 (function space) 이라한다. F(X, Y ) 는다음과같이적당한적집합과 함수공간 함수공간, 점열린위상 Definition.1. X와 Y 는임의의집합이고 F(X, Y ) 를 X에서 Y 로의모든함수족이라하자. 집합 F(X, Y ) 에위상을정의할때이것을함수공간 (function spce) 이라한다. F(X, Y ) 는다음과같이적당한적집합과같음을볼수있다. 각 x X에대해 Y x = Y 라하자. 그리고 F := Y x x X 이라하자.

More information

생존분석의 추정과 비교 : 보충자료 이용희 December 12, 2018 Contents 1 생존함수와 위험함수 생존함수와 위험함수 예제: 지수분포

생존분석의 추정과 비교 : 보충자료 이용희 December 12, 2018 Contents 1 생존함수와 위험함수 생존함수와 위험함수 예제: 지수분포 생존분석의 추정과 비교 : 보충자료 이용희 December, 8 Cotets 생존함수와 위험함수. 생존함수와 위험함수....................................... 예제: 지수분포.......................................... 예제: 와이블분포.........................................

More information

7.7) 정의역이 8.8) 연속확률변수 10.10) 원점을 좌표평면에서 인함수 의그래프가그림 과같다. 9.9 ) 함수 의그래프와함수 의 그래프가만나는점을 라할때, 옳은것만을 < 보기 > 에서있는대로고른것은? lim lim 의값은? < 보기 > ㄱ. ㄴ

7.7) 정의역이 8.8) 연속확률변수 10.10) 원점을 좌표평면에서 인함수 의그래프가그림 과같다. 9.9 ) 함수 의그래프와함수 의 그래프가만나는점을 라할때, 옳은것만을 < 보기 > 에서있는대로고른것은? lim lim 의값은? < 보기 > ㄱ. ㄴ 1.1) 2.2) 두 두 로그부등식 제 2 교시 2012 년 5 월고 2 모의평가문제지 성명수험번호 3 1 먼저수험생이선택한응시유형의문제지인지확인하시오. 문제지에성명과수험번호를정확히기입하시오. 답안지에수험번호, 응시유형및답을표기할때는반드시 수험생이지켜야할일 에따라표기하시오. 단답형답의숫자에 0 이포함된경우, 0 을 OMR 답안지에반드시표기해야합니다. 문항에따라배점이다르니,

More information

7) 다음의 다음 9) 남학생과 9. zb 여학생 각각 명이 갖고 있는 여름 티 셔츠의 개수를 조사하여 꺾은선그래프로 나타낸 것 이다. 이 두 그래프의 설명으로 옳지 않은 것은? ㄱ. ㄴ. 회째의 수학 점수는 점이다. 수학 점수의 분산은 이다. ㄷ. 영어점수가 수학 점

7) 다음의 다음 9) 남학생과 9. zb 여학생 각각 명이 갖고 있는 여름 티 셔츠의 개수를 조사하여 꺾은선그래프로 나타낸 것 이다. 이 두 그래프의 설명으로 옳지 않은 것은? ㄱ. ㄴ. 회째의 수학 점수는 점이다. 수학 점수의 분산은 이다. ㄷ. 영어점수가 수학 점 1) 은경이네 2) 어느 3) 다음은 자연수 그림은 6) 학생 학년 고사종류 과목 과목코드번호 성명 3 2012 2학기 중간고사 대비 수학 201 대청중 콘텐츠산업 진흥법 시행령 제33조에 의한 표시 1) 제작연월일 : 2012-08-27 2) 제작자 : 교육지대 3) 이 콘텐츠는 콘텐츠산업 진흥법 에 따라 최초 제작일부터 년간 보호됩니다. 콘텐츠산업 진흥법

More information

(001~006)개념RPM3-2(부속)

(001~006)개념RPM3-2(부속) www.imth.tv - (~9)개념RPM-(본문).. : PM RPM - 대푯값 페이지 다민 PI LPI 알피엠 대푯값과산포도 유형 ⑴ 대푯값 자료 전체의 중심적인 경향이나 특징을 하나의 수로 나타낸 값 ⑵ 평균 (평균)= Ⅰ 통계 (변량)의 총합 (변량의 개수) 개념플러스 대푯값에는 평균, 중앙값, 최 빈값 등이 있다. ⑶ 중앙값 자료를 작은 값부터 크기순으로

More information

제 2 교시 2019 학년도 3 월고 1 전국연합학력평가문제지수학영역 1 5 지선다형 1. 의값은? [2점] 일차방정식 의해는? [2 점 ] 두수, 의최대공약수는? [2 점 ] 일차함수 의그래프에서

제 2 교시 2019 학년도 3 월고 1 전국연합학력평가문제지수학영역 1 5 지선다형 1. 의값은? [2점] 일차방정식 의해는? [2 점 ] 두수, 의최대공약수는? [2 점 ] 일차함수 의그래프에서 제 2 교시 2019 학년도 3 월고 1 전국연합학력평가문제지 1 5 지선다형 1. 의값은? [2점] 1 2 3 4 5 3. 일차방정식 의해는? [2 점 ] 1 2 3 4 5 2. 두수, 의최대공약수는? [2 점 ] 1 2 3 4 5 4. 일차함수 의그래프에서 절편과 절편의합은? [3 점 ] 1 2 3 4 5 1 12 2 5. 함수 의그래프가두점, 를지날때,

More information

편입수학만점공식 위드유편입 1

편입수학만점공식 위드유편입 1 편입수학만점공식 위드유편입 1 2 MATH DICTIONARY MATH DICTIONARY 편입수학사전 편입수학만점공식 편입수학만점공식 1. 그리스어문자 대문자소문자읽기대문자소문자읽기 Α alpha nu Β beta xi Γ gamma omicron Δ delta pi Ε epsilon rho Ζ zeta sigma Η eta tau Θ theta upsilon

More information

<BCF6B8AEBFB5BFAA28B0A1C7FC295FC2A6BCF62E687770>

<BCF6B8AEBFB5BFAA28B0A1C7FC295FC2A6BCF62E687770> 제 2 교시 2013 학년도대학수학능력시험문제지 수리영역 ( 가형 ) 1 짝수형 5 지선다형 1. 두행렬, 모든성분의합은? [2 점 ] 에대하여행렬 의 3. 좌표공간에서두점 A, B 에대하여선분 AB 를 로내분하는점의좌표가 이다. 의값은? [2점] 1 2 3 4 5 1 2 3 4 5 2. sin 일때, sin 의값은? ( 단, 이다.) [2 점 ] 1 2 3

More information

미분기하학 II-16 복소평면의선형분수변환과쌍곡평면의등장사상 김영욱 (ÑñÁ) 강의양성덕 (zû ) 의강의록 Ø 'x! xxñ 2007 년 김영욱 (ÑñÁ) 강의양성덕 (zû ) 의강의록 (Ø 'x!) 미분기하 II 2007 년 1 / 26

미분기하학 II-16 복소평면의선형분수변환과쌍곡평면의등장사상 김영욱 (ÑñÁ) 강의양성덕 (zû ) 의강의록 Ø 'x! xxñ 2007 년 김영욱 (ÑñÁ) 강의양성덕 (zû ) 의강의록 (Ø 'x!) 미분기하 II 2007 년 1 / 26 미분기하학 II-16 복소평면의 김영욱 (ÑñÁ) 강의양성덕 (zû ) 의강의록 Ø 'x! xxñ 2007 년 김영욱 (ÑñÁ) 강의양성덕 (zû ) 의강의록 (Ø 'x!) 미분기하 II 2007 년 1 / 26 자, 이제 H 2 의등장사상에대해좀더자세히알아보자. Definition 선형분수변환이란다음형식의사상을뜻한다. Example f (z) = az +

More information

MGFRSQQFNTOD.hwp

MGFRSQQFNTOD.hwp 접선의방정식과평균값의정리 1. 접선의기울기와미분계수 곡선 위의점 에서의접선의기울기는 2. 접선의방정식 (1) 접선의방정식 곡선 위의점 에서의접선의방정식은 ( 단, y 1 = f (x 1 ) ) (2) 법선의방정식 곡선 위의점 에서의법선의방정식은 3. 두곡선의공통접선 두곡선 가 (1) 점 에서접할조건 1 (2) 점 에서직교할조건 1 2 2 4. 롤(Rolle)

More information

벡터(0.6)-----.hwp

벡터(0.6)-----.hwp 만점을위한 수학전문가남언우 - 벡터 1강 _ 분점의위치벡터 2강 _ 벡터의일차결합 3강 _ 벡터의연산 4강 _ 내적의도형적의미 5강 _ 좌표를잡아라 6강 _ 내적의활용 7강 _ 공간도형의방정식 8강 _ 구의방정식 9강 _2014년수능최고난도문제 좌표공간에 orbi.kr 1 강 _ 분점의위치벡터 01. 1) 두점 A B 이있다. 평면 에있는점 P 에대하여 PA

More information

스무살, 마음껏날아오르기위해, 일년만꾹참자! 2014학년도대학수학능력시험 9월모의평가 18번두이차정사각행렬 가 를만족시킬때, 옳은것만을 < 보기 > 에서있는대로고른것은? ( 단, 는단위행렬이다.) [4점] < 보기 > ㄱ. ㄴ. ㄷ. 2013학년도대학수학능력시험 16번

스무살, 마음껏날아오르기위해, 일년만꾹참자! 2014학년도대학수학능력시험 9월모의평가 18번두이차정사각행렬 가 를만족시킬때, 옳은것만을 < 보기 > 에서있는대로고른것은? ( 단, 는단위행렬이다.) [4점] < 보기 > ㄱ. ㄴ. ㄷ. 2013학년도대학수학능력시험 16번 친절한하영쌤의 수학 A형 약점체크집중공략오답률 Best 5 정복 하기! - 보충문제 행렬 2015학년도대학수학능력시험 9월모의평가 19번두이차정사각행렬 가 를만족시킬때, < 보기 > 에서옳은것만을있는대로고른것은? ( 단, 는단위행렬이고, 는영행렬이다.) [4점] < 보기 > ㄱ. 의역행렬이존재한다. ㄴ. ㄷ. 2015학년도대학수학능력시험 6월모의평가 19번두이차정사각행렬

More information

3.2 함수의정의 Theorem 6 함수 f : X Y 와 Y W 인집합 W 에대하여 f : X W 는함수이다. Proof. f : X Y 가함수이므로 f X Y 이고, Y W 이므로 f X W 이므로 F0이만족된다. 함수의정의 F1, F2은 f : X Y 가함수이므로

3.2 함수의정의 Theorem 6 함수 f : X Y 와 Y W 인집합 W 에대하여 f : X W 는함수이다. Proof. f : X Y 가함수이므로 f X Y 이고, Y W 이므로 f X W 이므로 F0이만족된다. 함수의정의 F1, F2은 f : X Y 가함수이므로 3.2 함수의정의 Theorem 6 함수 f : X Y 와 Y W 인집합 W 에대하여 f : X W 는함수이다. Proof. f : X Y 가함수이므로 f X Y 이고, Y W 이므로 f X W 이므로 F0이만족된다. 함수의정의 F1, F2은 f : X Y 가함수이므로성립한다. Theorem 7 두함수 f : X Y 와 g : X Y 에대하여, f = g f(x)

More information

<B1B9BEEE412E687770>

<B1B9BEEE412E687770> 2015 학년도대학수학능력시험문제및정답 2015 학년도대학수학능력시험문제지 1 제 2 교시 홀수형 5 지선다형 1. 의값은? [2점] 3. lim 의값은? [2점] 1 2 3 4 5 1 2 3 4 5 2. 두행렬 성분의합은? [2 점 ], 에대하여행렬 의모든 4. 다음그래프의각꼭짓점사이의연결관계를나타내는행렬의성분중 의개수는? [3점] 1 2 3 4 5 1 2

More information

제 5강 리만적분

제 5강 리만적분 제 5 강리만적분 리만적분 정의 : 두실수, 가 을만족핚다고가정하자.. 만일 P [, ] 이고 P 가두끝점, 을모두포함하는유핚집합일때, P 을 [, ] 의분핛 (prtitio) 이라고핚다. 주로 P { x x x } 로나타낸다.. 분핛 P { x x x } 의노름을다음과같이정의핚다. P x x x. 3. [, ] 의두분핛 P 와 Q 에대하여만일 P Q이면 Q

More information

5.5) cos 6.6) 두 coscos 일때, sinsin 의값은? [3점] ) 일때, 방정식 의모든해의합은? [3 점 ] 1 4 sin cos 의값은? [3점] 1 ln 2 ln 3 ln 4 ln 5 ln 8.8 ) 벡터 에대하여

5.5) cos 6.6) 두 coscos 일때, sinsin 의값은? [3점] ) 일때, 방정식 의모든해의합은? [3 점 ] 1 4 sin cos 의값은? [3점] 1 ln 2 ln 3 ln 4 ln 5 ln 8.8 ) 벡터 에대하여 1.1) 두 2.2) 방정식 좌표공간에서 두 제 2 교시 2016 년 9 월고 3 모의고사문제지 성명수험번호 3 1 먼저수험생이선택한응시유형의문제지인지확인하시오. 문제지에성명과수험번호를정확히기입하시오. 답안지에수험번호, 응시유형및답을표기할때는반드시 수험생이지켜야할일 에따라표기하시오. 단답형답의숫자에 0 이포함된경우, 0 을 OMR 답안지에반드시표기해야합니다.

More information

01 2 NK-Math 평면좌표

01 2 NK-Math 평면좌표 01 평면좌표 NK-Math 1 01 2 NK-Math 평면좌표 01 평면좌표 NK-Math 3 테마1. 테마1. 두 점 사이의 거리 1. 1.세 점 O A B 에 대하여 삼각형 OAB 의 외심의 좌표가 일 때, 양수 의 합 의 값을 구하여라. 2. 2.두 점 A B 과 직선 위의 점 P 에 대하여 AP BP 일 때, 상수 의 곱 의 값은? ① ② ④ ⑤ 3.

More information

2018년 수학성취도 측정시험 모범답안/채점기준/채점소감 (2018학년도 수시모집, 정시모집 및 외국인특별전형 합격자 대상) 2018년 2월 13일, 고사시간 90분 2018년 1번 x3 + x2 + x 3 = x 1 x2 1 lim. [풀이] x3 + x2 + x 3

2018년 수학성취도 측정시험 모범답안/채점기준/채점소감 (2018학년도 수시모집, 정시모집 및 외국인특별전형 합격자 대상) 2018년 2월 13일, 고사시간 90분 2018년 1번 x3 + x2 + x 3 = x 1 x2 1 lim. [풀이] x3 + x2 + x 3 8년 수학성취도 측정시험 모범답안/채점기준/채점소감 (8학년도 수시모집, 정시모집 및 외국인특별전형 합격자 대상) 8년 월 일, 고사시간 9분 8년 번 x + x + x x x lim. [풀이] x + x + x (x )(x + x + ) lim x x x (x )(x + ) x + x + lim x x+ limx x + x + limx x + 6 lim 8년

More information

statistics

statistics 수치를이용한자료요약 statistics hmkang@hallym.ac.kr 한림대학교 통계학 강희모 ( 한림대학교 ) 수치를이용한자료요약 1 / 26 수치를 통한 자료의 요약 요약 방대한 자료를 몇 개의 의미있는 수치로 요약 자료의 분포상태를 알 수 있는 통계기법 사용 중심위치의 측도(measure of center) : 어떤 값을 중심으로 분포되어 있는지

More information

2013unihangulchar {45380} 2unihangulchar {54617}unihangulchar {44592} unihangulchar {49328}unihangulchar {50629}unihangulchar {51312}unihangulchar {51

2013unihangulchar {45380} 2unihangulchar {54617}unihangulchar {44592} unihangulchar {49328}unihangulchar {50629}unihangulchar {51312}unihangulchar {51 Proem Se 4 산업조직론 (ECM004N) Fall 03. 독점기업이 다음과 같은 수요함수를 각각 가지고 있는 두 개의 소비자 그룹에게 제품을 공급한다고 하자. 한 단위 제품을 생산하는 데 드는 비용은 상수 이다. 다음 질문에 답하시오. P = A B Q P = A B Q () 두 그룹에 대하여 가격차별을 하고자 할 때 각 그룹의 균형생산량(Q, Q )과

More information

31. 을전개한식에서 의계수는? 를전개한식이 일 때, 의값은? 을전개했을때, 의계수와상수항의합을구하면? 을전개했을때, 의 계수는? 를전개했을때, 상수항을 구하여라. 37

31. 을전개한식에서 의계수는? 를전개한식이 일 때, 의값은? 을전개했을때, 의계수와상수항의합을구하면? 을전개했을때, 의 계수는? 를전개했을때, 상수항을 구하여라. 37 21. 다음식의값이유리수가되도록유리수 의값을 정하면? 1 4 2 5 3 26. 을전개하면상수항을 제외한각항의계수의총합이 이다. 이때, 의값은? 1 2 3 4 5 22. 일때, 의값은? 1 2 3 4 5 27. 를전개하여간단히 하였을때, 의계수는? 1 2 3 4 5 23. 를전개하여 간단히하였을때, 상수항은? 1 2 3 4 5 28. 두자연수 와 를 로나누면나머지가각각

More information

고 학년도 9월고수학 1 전국연합학력평가영역문제지 1 1 제 2 교시 수학영역 5 지선다형 3. 두다항식, 에대하여 는? [ 점 ] 1. 의값은? ( 단, ) [ 점 ] 다항식 이 로인수분해될때, 의값은? ( 단,,

고 학년도 9월고수학 1 전국연합학력평가영역문제지 1 1 제 2 교시 수학영역 5 지선다형 3. 두다항식, 에대하여 는? [ 점 ] 1. 의값은? ( 단, ) [ 점 ] 다항식 이 로인수분해될때, 의값은? ( 단,, 고 208학년도 9월고수학 전국연합학력평가영역문제지 제 2 교시 수학영역 5 지선다형 3. 두다항식, 에대하여 는? [ 점 ]. 의값은? ( 단, ) [ 점 ] 2 3 2 3 4 5 4 5 2. 다항식 이 로인수분해될때, 의값은? ( 단,, 는상수이다.) [ 점 ] 4. 좌표평면위의두점 A, B 사이의거리가 일때, 양수 의값은? [ 점 ] 2 3 4 5 2

More information

Microsoft PowerPoint Relations.pptx

Microsoft PowerPoint Relations.pptx 이산수학 () 관계와그특성 (Relations and Its Properties) 2010년봄학기강원대학교컴퓨터과학전공문양세 Binary Relations ( 이진관계 ) Let A, B be any two sets. A binary relation R from A to B, written R:A B, is a subset of A B. (A 에서 B 로의이진관계

More information

지구에서달까지의거리는얼마일까? ( Hipparchos ;? ~? B. C. 125 ) ( Rheticus, G. K. ; 1514~1576 ) ( Fourier, J. B. J. ; 1768 ~ 1830 )

지구에서달까지의거리는얼마일까? ( Hipparchos ;? ~? B. C. 125 ) ( Rheticus, G. K. ; 1514~1576 ) ( Fourier, J. B. J. ; 1768 ~ 1830 ) Ⅶ 삼각함수 1 삼각함수 2 삼각형에의응용 지구에서달까지의거리는얼마일까? ( Hipparchos ;? ~? B. C. 125 ) ( Rheticus, G. K. ; 1514~1576 ) ( Fourier, J. B. J. ; 1768 ~ 1830 ) 수학의명언 1 : 의사선생님, 무엇을보고계세요? : 심전도그래프를보고있단다. : 심전도그래프가무엇인가요? :

More information

= Fisher, I. (1930), ``The Theory of Interest,'' Macmillan ,

= Fisher, I. (1930), ``The Theory of Interest,'' Macmillan , Finance Lecture Note Series 학습목표 제4강 소유와 경영의 분리 효용함수(utility function): 효용함수, 한계효용(marginal utility), 한계대체율(marginal rate of substitution) 의 개념에 대해 알아본다 조 승 모2 (production possibility curve): 생산가능곡선과 한계변환율(marginal

More information

<30325FBCF6C7D05FB9AEC7D7C1F62E687770>

<30325FBCF6C7D05FB9AEC7D7C1F62E687770> 고1 2015학년도 9월고수학 1 전국연합학력평가영역문제지 1 1 제 2 교시 수학영역 1. 두복소수, 에대하여 의값은? ( 단, ) [2 점 ] 1 2 3 4 5 3. 좌표평면위의두점 P, Q 사이의거리는? [2 점 ] 1 2 3 4 5 2. 두다항식, 에대하여 를간단히하면? [2점] 4. 에서이차함수 의최댓값을, 최솟값을 이라할때, 의값은? [3점] 1

More information

inance Lectue Note Seies 금융시장과 투자분석 제강. 화폐의 시간가치 조 승 모 영남대학교 경제금융학부 04학년도 학기 학습목표. 화폐의 시간가치(tie value of oney): 동일한 금액의 화폐라도 시점에 따라 다른 가치를 가지게 되는 화폐의 시간가치에 대해 알아본다.. 수익률(ate of etun): 단순 수익률과 로그 수익률을 정의하고

More information

2019 학년도대학수학능력시험문제및정답

2019 학년도대학수학능력시험문제및정답 2019 학년도대학수학능력시험문제및정답 2019 학년도대학수학능력시험문제지 1 제 2 교시 홀수형 5 지선다형 1. 두벡터, 에대하여 벡터 의모든성분의합은? [2 점 ] 3. 좌표공간의두점 A, B 에대하여선분 AB 를 로내분하는점이 축위에있을때, 의값은? [2점] 1 2 3 4 5 1 2 3 4 5 2. lim 의값은? [2점] ln 4. 두사건, 에대하여

More information

[Real Analysis]4.1

[Real Analysis]4.1 정동명해석학 4.1 수열의수렴성 1. 다음의수열 중에서어느것이수렴하는가를조사하여라. 또, 그이유를밝혀라. (1) 수렴한다. 임의의 에대하여 아르키메데스성질에의하여 을만족하는 을택하면 일때, 이성립한다. 여기서 이므로 이성립한다. 따라서 은 1 로수렴한다. (2) 수렴한다. 임의의 에대하여 아르키메데스성질에의하여 을만족하는 을택하면 일때, 이성립한다. 따라서

More information

<A1DAA1DAA1DA20C6DBC5AC20BCF6C7D020BFCFB7E E687770>

<A1DAA1DAA1DA20C6DBC5AC20BCF6C7D020BFCFB7E E687770> 수리이과 1 강 이과 1 강 삼차함수그래프의특징 01 삼차함수의그래프 1. 기울기가같은두접선 수리영역이상빈 1 에서극댓값, 에서극솟값 을가진다. 2 에서변곡점을가지고 3 극댓점과극솟점에서 축과평행한접선을그었을때 와만나는점을 이라하면, 은차례대로등차수열을이룬다. ( 간격이모두같다.) 4 극댓점 와접선과의교점 을 2:1로내분한점이극솟점 가된다. 5 같은기울기를가진두접선과교점,

More information

(Microsoft PowerPoint - Ch19_NumAnalysis.ppt [\310\243\310\257 \270\360\265\345])

(Microsoft PowerPoint - Ch19_NumAnalysis.ppt [\310\243\310\257 \270\360\265\345]) 수치해석 6009 Ch9. Numerical Itegratio Formulas Part 5. 소개 / 미적분 미분 : 독립변수에대한종속변수의변화율 d vt yt dt yt 임의의물체의시간에따른위치, vt 속도 함수의구배 적분 : 미분의역, 어떤구간내에서시간 / 공간에따라변화하는정보를합하여전체결과를구함. t yt vt dt 0 에서 t 까지의구간에서곡선 vt

More information

수리영역 5. 서로다른두개의주사위를동시에던져서나온두눈의수의곱 이짝수일때, 나온두눈의수의합이 또는 일확률은? 5) 의전개식에서상수항이존재하도록하는모든자 연수 의값의합은? 7) 다음순서도에서인쇄되는 의값은? 6) 8. 어떤특산

수리영역 5. 서로다른두개의주사위를동시에던져서나온두눈의수의곱 이짝수일때, 나온두눈의수의합이 또는 일확률은? 5) 의전개식에서상수항이존재하도록하는모든자 연수 의값의합은? 7) 다음순서도에서인쇄되는 의값은? 6) 8. 어떤특산 제 2 교시 2008 학년도 10 월고 3 전국연합학력평가문제지 수리영역 성명수험번호 3 1 먼저수험생이선택한응시유형의문제지인지확인하시오. 문제지에성명과수험번호를정확히기입하시오. 답안지에수험번호, 응시유형및답을표기할때는반드시 수험생이지켜야할일 에따라표기하시오. 단답형답의숫자에 0 이포함된경우, 0 을 OMR 답안지에반드시표기해야합니다. 문항에따라배점이다르니,

More information

2018 년수학임용고시기출풀이 ( 대수학, 해석학, 복소해석, 위상수학, 정수론, 선형대수, 미적분학 ) - 하이어에듀 - 구준모강사 1

2018 년수학임용고시기출풀이 ( 대수학, 해석학, 복소해석, 위상수학, 정수론, 선형대수, 미적분학 ) - 하이어에듀 - 구준모강사 1 8 년수학임용고시기출풀이 ( 대수학 해석학 복소해석 위상수학 정수론 선형대수 미적분학 ) - 하이어에듀 - 구준모강사 8년 수학 임용고시 기출풀이 (안내) 제가 작성한 8년 수학 임용시험 기출 풀이 참고 답안입니다. 8년 임용 시험을 치르신 분들과 앞으로 준비 하시는 분들께 참고가 되었으면 좋겠습니다. 혹시 풀이에 오류가 있다면 제 이메일(junmomath8@gmail.com)

More information

체의원소를계수로가지는다항식환 Theorem 0.1. ( 나눗셈알고리듬 (Division Algorithm)) F 가체일때 F [x] 의두다항식 f(x) = a 0 + a 1 x + + a n x n, a n 0 F 와 g(x) = b 0 + b 1 x + + b m x

체의원소를계수로가지는다항식환 Theorem 0.1. ( 나눗셈알고리듬 (Division Algorithm)) F 가체일때 F [x] 의두다항식 f(x) = a 0 + a 1 x + + a n x n, a n 0 F 와 g(x) = b 0 + b 1 x + + b m x 체의원소를계수로가지는다항식환 Theorem 0.1. ( 나눗셈알고리듬 (Division Algorithm)) F 가체일때 F [x] 의두다항식 f(x) = a 0 + a 1 x + + a n x n, a n 0 F 와 g(x) = b 0 + b 1 x + + b m x m, b m 0 F, m > 0 에대해 f(x) = g(x)q(x) + r(x) 을만족하는

More information

1.1) 등비수열 전체집합 제 2 교시 나 형 2016 년 3 월고 3 모의고사문제지 수리영역 성명수험번호 3 1 먼저수험생이선택한응시유형의문제지인지확인하시오. 문제지에성명과수험번호를정확히기입하시오. 답안지에수험번호, 응시유형및답을표기할때는반드시 수험생이지켜야할일 에따

1.1) 등비수열 전체집합 제 2 교시 나 형 2016 년 3 월고 3 모의고사문제지 수리영역 성명수험번호 3 1 먼저수험생이선택한응시유형의문제지인지확인하시오. 문제지에성명과수험번호를정확히기입하시오. 답안지에수험번호, 응시유형및답을표기할때는반드시 수험생이지켜야할일 에따 1.1) 등비수열 전체집합 제 2 교시 2016 년 3 월고 3 모의고사문제지 성명수험번호 3 1 먼저수험생이선택한응시유형의문제지인지확인하시오. 문제지에성명과수험번호를정확히기입하시오. 답안지에수험번호, 응시유형및답을표기할때는반드시 수험생이지켜야할일 에따라표기하시오. 단답형답의숫자에 0 이포함된경우, 0 을 OMR 답안지에반드시표기해야합니다. 문항에따라배점이다르니,

More information

<C1DF29BCF6C7D020315FB1B3BBE7BFEB20C1F6B5B5BCAD2E706466>

<C1DF29BCF6C7D020315FB1B3BBE7BFEB20C1F6B5B5BCAD2E706466> 84 85 86 87 88 89 1 12 1 1 2 + + + 11=60 9 19 21 + + + 19 17 13 11=60 + 5 7 + 5 + 10 + 8 + 4+ 6 + 3=48 1 2 90 1 13 1 91 2 3 14 1 2 92 4 1 2 15 2 3 4 93 1 5 2 6 1 2 1 16 6 5 94 1 1 22 33 55 1 2 3 4 5 6

More information

미시경제학을위한기초수학 조남운 March 20, 함수 1.1 함수란무엇인가 여러분이미시경제학을배우면서미분을배우는이유는계산을통해함수의최대값이나최소값을구해야하기때문이다. 최대값이나최소값을구하기위해서는함수의미분을알

미시경제학을위한기초수학 조남운 March 20, 함수 1.1 함수란무엇인가 여러분이미시경제학을배우면서미분을배우는이유는계산을통해함수의최대값이나최소값을구해야하기때문이다. 최대값이나최소값을구하기위해서는함수의미분을알 미시경제학을위한기초수학 조남운 mailto:namun.cho@gmail.com March 20, 2008 1 함수 1.1 함수란무엇인가 여러분이미시경제학을배우면서미분을배우는이유는계산을통해함수의최대값이나최소값을구해야하기때문이다. 최대값이나최소값을구하기위해서는함수의미분을알아야하며, 함수의미분을알기위해서는함수의연속과극한을알아야한다. 그중에서도가장먼저알아야할것은 함수

More information

완벽한개념정립 _ 행렬의참, 거짓 수학전문가 NAMU 선생 1. 행렬의참, 거짓개념정리 1. 교환법칙과관련한내용, 는항상성립하지만 는항상성립하지는않는다. < 참인명제 > (1),, (2) ( ) 인경우에는 가성립한다.,,, (3) 다음과같은관계식을만족하는두행렬 A,B에

완벽한개념정립 _ 행렬의참, 거짓 수학전문가 NAMU 선생 1. 행렬의참, 거짓개념정리 1. 교환법칙과관련한내용, 는항상성립하지만 는항상성립하지는않는다. < 참인명제 > (1),, (2) ( ) 인경우에는 가성립한다.,,, (3) 다음과같은관계식을만족하는두행렬 A,B에 1. 행렬의참, 거짓개념정리 1. 교환법칙과관련한내용, 는항상성립하지만 는항상성립하지는않는다. < 참인명제 > (1),, (2) ( ) 인경우에는 가성립한다.,,, (3) 다음과같은관계식을만족하는두행렬 A,B에대하여 AB=BA 1 가성립한다 2 3 (4) 이면 1 곱셈공식및변형공식성립 ± ± ( 복호동순 ), 2 지수법칙성립 (은자연수 ) < 거짓인명제 >

More information

= Fisher, I. (1930), ``The Theory of Interest,'' Macmillan ,

= Fisher, I. (1930), ``The Theory of Interest,'' Macmillan , Finance Lecture Note Series 금융시장과 투자분석 연구 제4강. 소유와 경영의 분리1 조 승 모2 영남대학교 대학원 경제학과 2015학년도 2학기 Copyright 2015 Cho, Seung Mo 1 기본적으로 Fisher, I. (1930), The Theory of Interest, Macmillan의 내용을 바탕으로 작성되었으며,

More information

와플-4년-2호-본문-15.ps

와플-4년-2호-본문-15.ps 1 2 1+2 + = = 1 1 1 +2 =(1+2)+& + *=+ = + 8 2 + = = =1 6 6 6 6 6 2 2 1 1 1 + =(1+)+& + *=+ =+1 = 2 6 1 21 1 + = + = = 1 1 1 + 1-1 1 1 + 6 6 0 1 + 1 + = = + 7 7 2 1 2 1 + =(+ )+& + *= + = 2-1 2 +2 9 9 2

More information

2018 학년도대학수학능력시험문제지 1 제 2 교시 홀수형 5 지선다형 1. 두벡터, 모든성분의합은? [2 점 ] 에대하여벡터 의 3. 좌표공간의두점 A, B 에대하여선분 AB 를 으로내분하는점의좌표가 이다. 의값은? [2점] ln

2018 학년도대학수학능력시험문제지 1 제 2 교시 홀수형 5 지선다형 1. 두벡터, 모든성분의합은? [2 점 ] 에대하여벡터 의 3. 좌표공간의두점 A, B 에대하여선분 AB 를 으로내분하는점의좌표가 이다. 의값은? [2점] ln 2018 학년도대학수학능력시험문제및정답 2018 학년도대학수학능력시험문제지 1 제 2 교시 홀수형 5 지선다형 1. 두벡터, 모든성분의합은? [2 점 ] 에대하여벡터 의 3. 좌표공간의두점 A, B 에대하여선분 AB 를 으로내분하는점의좌표가 이다. 의값은? [2점] 1 2 3 4 5 1 2 3 4 5 ln 2. lim 의값은? [2점] 4. 두사건 와 는서로독립이고

More information

Python과 함께 배우는 신호 해석 제 5 강. 복소수 연산 및 Python을 이용한 복소수 연산 (제 2 장. 복소수 기초)

Python과 함께 배우는 신호 해석 제 5 강. 복소수 연산 및 Python을 이용한 복소수 연산      (제 2 장. 복소수 기초) 제 5 강. 복소수연산및 을이용한복소수연산 ( 제 2 장. 복소수기초 ) 한림대학교전자공학과 한림대학교 제 5 강. 복소수연산및 을이용한복소수연산 1 배울내용 복소수의기본개념복소수의표현오일러 (Euler) 공식복소수의대수연산 1의 N 승근 한림대학교 제 5 강. 복소수연산및 을이용한복소수연산 2 복소수의 4 칙연산 복소수의덧셈과뺄셈에는직각좌표계표현을사용하고,

More information

Microsoft PowerPoint - 26.pptx

Microsoft PowerPoint - 26.pptx 이산수학 () 관계와그특성 (Relations and Its Properties) 2011년봄학기 강원대학교컴퓨터과학전공문양세 Binary Relations ( 이진관계 ) Let A, B be any two sets. A binary relation R from A to B, written R:A B, is a subset of A B. (A 에서 B 로의이진관계

More information

문항코드 EBS 수능완성수학영역수학 1 A 형 주어진그래프의꼭짓점에 를그림과같이 정하고꼭짓점사이의연결관계를행렬로나타내면다 음과같다. ( 나 ) 세수, 12, 는이순서대로등비수열을이룬다. 의값은? 문

문항코드 EBS 수능완성수학영역수학 1 A 형 주어진그래프의꼭짓점에 를그림과같이 정하고꼭짓점사이의연결관계를행렬로나타내면다 음과같다. ( 나 ) 세수, 12, 는이순서대로등비수열을이룬다. 의값은? 문 곽정원의수능필수아이템! 2,3 점은다내꺼 + 4 점도전 ~ 실전모의고사 1. 두행렬 의모든성분의합은? 1 9 2 10 3 11 4 12 5 13 배점 2 문항코드 3-182-365 기 따라서행렬 의모든성분의합은 7+(-4)+4+5=12 2. log l 의값은? 에대하여행렬 3. lim 의값은? 1 2 3 1 4 2 5 4 배점 2 문항코드 3-179-239

More information

2014 학년도수학성취도측정시험 (2014 학년도정시모집합격자대상 ) 2014 년 2 월 17 일, 고사시간 90 분 1번부터 11번까지는단답형이고, 12번부터 16번까지는서술형입니다. 답안지는깨끗한글씨로바르게작성하되, 단답형은답만쓰고, 서술형은풀이과정과답을명시하시오.

2014 학년도수학성취도측정시험 (2014 학년도정시모집합격자대상 ) 2014 년 2 월 17 일, 고사시간 90 분 1번부터 11번까지는단답형이고, 12번부터 16번까지는서술형입니다. 답안지는깨끗한글씨로바르게작성하되, 단답형은답만쓰고, 서술형은풀이과정과답을명시하시오. 학년도수학성취도측정시험 ( 학년도정시모집합격자대상 년 월 7 일, 고사시간 9 분 번부터 번까지는단답형이고, 번부터 6번까지는서술형입니다. 답안지는깨끗한글씨로바르게작성하되, 단답형은답만쓰고, 서술형은풀이과정과답을명시하시오. 총배점은 점이고, 각문항의배점은, 기본문제 (-6번 각 점, 발전문제 (7-번 각 7점, 심화문제 (번-6번 각 점입니다. 년정시 번 lim

More information

수열의극한 수열의극한에서활용되는방법은크게다섯가지이다. ] 거미줄도형 ] 유계이론 ] 일반항 ] 부동점( 극한값) 활용 ] 샌드위치이론 ] 거미줄도형 가장첫번째로거미줄도형은대부분의경우수열의극한문제에서엄밀한증명을위해활용되기보다는수열이수렴하는지여부를판단하고수열의극한이존재한다

수열의극한 수열의극한에서활용되는방법은크게다섯가지이다. ] 거미줄도형 ] 유계이론 ] 일반항 ] 부동점( 극한값) 활용 ] 샌드위치이론 ] 거미줄도형 가장첫번째로거미줄도형은대부분의경우수열의극한문제에서엄밀한증명을위해활용되기보다는수열이수렴하는지여부를판단하고수열의극한이존재한다 수열의극한 수열의극한에서활용되는방법은크게다섯가지이다. ] 거미줄도형 ] 유계이론 ] 일반항 ] 부동점( 극한값) 활용 ] 샌드위치이론 ] 거미줄도형 가장첫번째로거미줄도형은대부분의경우수열의극한문제에서엄밀한증명을위해활용되기보다는수열이수렴하는지여부를판단하고수열의극한이존재한다면어디로수렴해야하는지를판단하는데에활용된다. 예를들어보도록하자. 수열이다음과같이정의되어있을때,

More information

Mathema Barista Type Daily Quiz 20 수1_기하과 벡터- part1.hwp

Mathema Barista Type Daily Quiz 20 수1_기하과 벡터- part1.hwp Mathema Barista Type Daily Quiz 20 수Ⅰ 기하와 벡터 [ 자료번호 1 ] 1. 답 5 정류장 에 번, 번이 정차하므로 정류장 에 번, 번이 정차하므로 정류장 에 번이 정차하므로 2. 답 두 원 를 좌표평면 위에 나타내면 다음 그림과 같다. 어두운 부분과 같으므로 구하는 영역의 넓이는 4. 답 이므로 이때, 에서 이므로 행렬이 서로

More information

집합 집합 오른쪽 l 3. (1) 집합 X 의각원소에대응하는집합 Y 의원소가단하나만인대응을 라할때, 이대응 를 X 에서 Y 로의라고하고이것을기호로 X Y 와같이나타낸다. (2) 정의역과공역정의역 : X Y 에서집합 X, 공역 : X Y 에서집합 Y (3) 의개수 X Y

집합 집합 오른쪽 l 3. (1) 집합 X 의각원소에대응하는집합 Y 의원소가단하나만인대응을 라할때, 이대응 를 X 에서 Y 로의라고하고이것을기호로 X Y 와같이나타낸다. (2) 정의역과공역정의역 : X Y 에서집합 X, 공역 : X Y 에서집합 Y (3) 의개수 X Y 어떤 다음 X 대응 1. 대응 (1) 어떤주어진관계에의하여집합 X 의원소에집합 Y 의원소를짝지어주는것을집합 X 에서집합 Y 로의대응이라고한다. l (2) 집합 X 의원소 에집합 Y 의원소 가짝지어지면 에 가대응한다고하며이것을기호로 와같이나타낸다. 2. 일대일대응 (1) 집합 A 의모든원소와집합 B 의모든원소가하나도빠짐없이꼭한개씩서로대응되는것을집합 A 에서집합

More information

2013 학년도수학성취도측정시험 (2013학년도수시모집및외국인특별전형합격자대상 ) 2012년 12월 18일, 고사시간 90분 1번부터 11번까지는단답형이고, 12번부터 16번까지는서술형입니다. 답안지는깨끗한글씨로바르게작성하되, 단답형은답만쓰고, 서술형은풀이과정과답을명시

2013 학년도수학성취도측정시험 (2013학년도수시모집및외국인특별전형합격자대상 ) 2012년 12월 18일, 고사시간 90분 1번부터 11번까지는단답형이고, 12번부터 16번까지는서술형입니다. 답안지는깨끗한글씨로바르게작성하되, 단답형은답만쓰고, 서술형은풀이과정과답을명시 학년도수학성취도측정시험 (학년도수시모집및외국인특별전형합격자대상 년 월 8일, 고사시간 9분 번부터 번까지는단답형이고, 번부터 번까지는서술형입니다. 답안지는깨끗한글씨로바르게작성하되, 단답형은답만쓰고, 서술형은풀이과정과답을명시하시오. 총배점은 점이고, 각문항의배점은, 기본문제 (-번 각 점, 발전문제 (7-번 각 7점, 심화문제 (4번-번 각 점입니다. x x

More information

마지막 변경일 2018년 5월 7일 ** 도형의 자취 문제 ** Geogebra와 수학의 시각화 책의 4.1소절 내용임. http://min7014.iptime.org/math/2017063002.htm 가장 최근 파일은 링크를 누르면 받아 보실 수 있습니다. https://goo.gl/tywcbz http://min7014.iptime.org/math/2018010402.pdf

More information

곡선 7.7. 오른쪽그림과같이반지름의길이가각각 이고중심이같은세원으로이루어진과녁에총을쏠때, 색칠한부분을맞힐확률은? ( 단, 총알은과녁을벗어나지않고, 경계선에맞지않는다.) [3점] [PP 난이도중 ] [PP 18 문

곡선 7.7. 오른쪽그림과같이반지름의길이가각각 이고중심이같은세원으로이루어진과녁에총을쏠때, 색칠한부분을맞힐확률은? ( 단, 총알은과녁을벗어나지않고, 경계선에맞지않는다.) [3점] [PP  난이도중 ] [PP 18 문 등차수열 함수 2017 학년도수능대비 9 월모의고사 FINAL 1 회 ( 나형 ) 제 2 교시 1 1. lim 의값은? 1 2 [PP 07 0006@ 문과 @ 고 3@ 수열의극한 @ 난이도하 ] 3 [2 점 ] 4.4. [PP 05 0010@ 문과 @ 고 3@ 수열 @ 난이도중 ] 에대하여 일때, 의값은? [3점] 1 2 3 4 5 4 5 [PP 08 0007@

More information

Chapter 연습문제답안. y *sin-*cos*^ep-*/sqrt. y [ ; sinpi/ ; sin*pi ; ] 혹은 [ sinpi/ sin*pi ]. a ais[- ] b et.,., sin. c.. a A는주어진행렬 M의 번째열만을표시하는새로운행렬을나타낸다.

Chapter 연습문제답안. y *sin-*cos*^ep-*/sqrt. y [ ; sinpi/ ; sin*pi ; ] 혹은 [ sinpi/ sin*pi ]. a ais[- ] b et.,., sin. c.. a A는주어진행렬 M의 번째열만을표시하는새로운행렬을나타낸다. IT CookBook, MATLAB 으로배우는공학수치해석 ] : 핵심개념부터응용까지 [ 연습문제답안이용안내 ] 본연습문제답안의저작권은한빛아카데미 주 에있습니다. 이자료를무단으로전제하거나배포할경우저작권법 조에의거하여최고 년이하의징역또는 천만원이하의벌금에처할수있고이를병과 倂科 할수도있습니다. - - Chapter 연습문제답안. y *sin-*cos*^ep-*/sqrt.

More information

(Hyunoo Shim) 1 / 24 (Discrete-time Markov Chain) * 그림 이산시간이다연쇄 (chain) 이다왜 Markov? (See below) ➀ 이산시간연쇄 (Discrete-time chain): : Y Y 의상태공간 = {0, 1, 2,..., n} Y n Y 의 n 시점상태 {Y n = j} Y 가 n 시점에상태 j 에있는사건

More information

2020 학년도랑데뷰실전모의고사문제지 - 시즌 6 제 1 회 제 2 교시 수학영역 ( 가형 ) 1 5 지선다형 학년도 9월모의평가가형과싱크로율 99% 학년도수학영역대비랑데뷰실전모의고사가형-시즌1~ 시즌6, 나형-시즌1~ 시즌2 ( 각시즌 4회분 )

2020 학년도랑데뷰실전모의고사문제지 - 시즌 6 제 1 회 제 2 교시 수학영역 ( 가형 ) 1 5 지선다형 학년도 9월모의평가가형과싱크로율 99% 학년도수학영역대비랑데뷰실전모의고사가형-시즌1~ 시즌6, 나형-시즌1~ 시즌2 ( 각시즌 4회분 ) 2020 학년도랑데뷰실전모의고사문제지 - 시즌 6 제 1 회 제 2 교시 1 5 지선다형 - 2020학년도 9월모의평가가형과싱크로율 99% - 2020학년도수학영역대비랑데뷰실전모의고사가형-시즌1~ 시즌6, 나형-시즌1~ 시즌2 ( 각시즌 4회분 ) 오르비전자책에서구매가능 - 오타, 오류수정파일은랑데뷰수학카페자료실에서무료다운로드가능 (cafe.daum.net/baekipsi)

More information

두 두 두 두 두 lim 1. 수열의극한 수열의극한에대한기본성질 1. 수열의극한 Ⅰ 수열의극한 5. 수열, 에대하여 lim, lim 이성 립할때, lim 의값은? [3 점 ][2015(A) 7 월 / 교육청 5] 의값은? [2 점 ][200

두 두 두 두 두 lim 1. 수열의극한 수열의극한에대한기본성질 1. 수열의극한 Ⅰ 수열의극한 5. 수열, 에대하여 lim, lim 이성 립할때, lim 의값은? [3 점 ][2015(A) 7 월 / 교육청 5] 의값은? [2 점 ][200 두 두 두 두 두 1. 01 1. 수열의극한 수열의극한에대한기본성질 1. 수열의극한 Ⅰ 수열의극한 5. 수열, 에대하여, 이성 립할때, 의값은? 1 2 3 4 5 [3 점 ][2015(A) 7 월 / 교육청 5] 의값은? [2 점 ][2006( 나 ) 9 월 / 평가원 3] 1 2 3 4 5 6. 수열, 이, 를만족할 때, 의값을구하시오. [3 점 ][2005(

More information

7. 다음그림과같이한변의길이 가 4 6 인마름모의넓이를구 하여라. 10. 다음그림과같이모선의길이가 6 cm 인원뿔의밑면의 둘레의길이가 6π cm 일때, 원뿔의높이와부피를구한 것은? 1 6 cm, 6 π cm 6 cm, 6π cm 8. 다음과같이한변의길이가 8 인정육 면

7. 다음그림과같이한변의길이 가 4 6 인마름모의넓이를구 하여라. 10. 다음그림과같이모선의길이가 6 cm 인원뿔의밑면의 둘레의길이가 6π cm 일때, 원뿔의높이와부피를구한 것은? 1 6 cm, 6 π cm 6 cm, 6π cm 8. 다음과같이한변의길이가 8 인정육 면 . 단원테스트 범위 : 피타고라스의정리 피타고라스의정리의활용 50 문항 / 저반 : 이름 : 출제자 : 박지연. 1. 다음그림에서 x 의값으로적절한것은? 4. 세변의길이가 6 cm, 5 cm, 10 cm 인삼각형은어떤삼 각형인가? 1 직각삼각형 이등변삼각형 직각이등변삼각형 4 예각삼각형 5 둔각삼각형 1 9 9 9 4 4 9 5 5 9. 삼각형의세변의길이가다음보기와같을때직각삼각

More information

0 cm (++x)=0 x= R QR Q =R =Q = cm =Q =-=(cm) =R =x cm (x+) = +(x+) x= x= (cm) =+=0 (cm) =+=8 (cm) + =0+_8= (cm) cm + = + = _= (cm) 7+x= x= +y= y=8,, Q

0 cm (++x)=0 x= R QR Q =R =Q = cm =Q =-=(cm) =R =x cm (x+) = +(x+) x= x= (cm) =+=0 (cm) =+=8 (cm) + =0+_8= (cm) cm + = + = _= (cm) 7+x= x= +y= y=8,, Q . 09~ cm 7 0 8 9 8'-p 0 cm x=, y=8 cm 0' 7 cm 8 cm 9 'åcm 90 'åcm T T=90 T T =" 8 - =' (cm) T= T= _T _T _'_ T=8' (cm ) 7 = == =80 -_ =0 = = _=(cm) M = = _0= (cm) M M =" - = (cm) r cm rcm (r-)cm H 8cm cm

More information

2017 학년도대학수학능력시험문제지 1 제 2 교시 홀수형 5 지선다형 3. sin 의값은? [2점] 1. 두벡터, 모든성분의합은? [2 점 ] 에대하여벡터 의 lim 의값은? [2점] ln 두사건 와 는

2017 학년도대학수학능력시험문제지 1 제 2 교시 홀수형 5 지선다형 3. sin 의값은? [2점] 1. 두벡터, 모든성분의합은? [2 점 ] 에대하여벡터 의 lim 의값은? [2점] ln 두사건 와 는 2017 학년도대학수학능력시험문제및정답 2017 학년도대학수학능력시험문제지 1 제 2 교시 홀수형 5 지선다형 3. sin 의값은? [2점] 1. 두벡터, 모든성분의합은? [2 점 ] 에대하여벡터 의 1 2 3 4 5 1 2 3 4 5 2. lim 의값은? [2점] ln 1 2 3 4 5 4. 두사건 와 는서로독립이고 P P 일때, PP 의값은? ( 단, 은

More information

완비거리공간 완비거리공간 Definition 0.1. (X, d) 는거리공간일때 X의점렬 < a n > 이모든 ɛ > 0에대해 n o N such that n, m > n o = d(a n, a m ) < ɛ 을만족하면이점렬을코시열 (Cauchy sequence) 이라

완비거리공간 완비거리공간 Definition 0.1. (X, d) 는거리공간일때 X의점렬 < a n > 이모든 ɛ > 0에대해 n o N such that n, m > n o = d(a n, a m ) < ɛ 을만족하면이점렬을코시열 (Cauchy sequence) 이라 완비거리공간 완비거리공간 Definition 0.1. (X, d) 는거리공간일때 X의점렬 < a n > 이모든 ɛ > 0에대해 n o N such that n, m > n o = d(a n, a m ) < ɛ 을만족하면이점렬을코시열 (Cauchy sequence) 이라한다. Example 0.2. < a n > 이 p에수렴하는점렬이면모든 ɛ > 0에대해 n

More information

Microsoft PowerPoint - chap04-연산자.pptx

Microsoft PowerPoint - chap04-연산자.pptx int num; printf( Please enter an integer: "); scanf("%d", &num); if ( num < 0 ) printf("is negative.\n"); printf("num = %d\n", num); } 1 학습목표 수식의 개념과 연산자, 피연산자에 대해서 알아본다. C의 를 알아본다. 연산자의 우선 순위와 결합 방향에

More information

소성해석

소성해석 3 강유한요소법 3 강목차 3. 미분방정식의근사해법-Ritz법 3. 미분방정식의근사해법 가중오차법 3.3 유한요소법개념 3.4 편미분방정식의유한요소법 . CAD 전처리프로그램 (Preprocessor) DXF, STL 파일 입력데이타 유한요소솔버 (Finite Element Solver) 자연법칙지배방정식유한요소방정식파생변수의계산 질량보존법칙 연속방정식 뉴톤의운동법칙평형방정식대수방정식

More information

<4D F736F F F696E74202D2035BBF3C6F2C7FC5FBCF8BCF6B9B0C1FA2E BC8A3C8AF20B8F0B5E55D>

<4D F736F F F696E74202D2035BBF3C6F2C7FC5FBCF8BCF6B9B0C1FA2E BC8A3C8AF20B8F0B5E55D> 5. 상평형 : 순수물질 이광남 5. 상평형 : 순수물질 상전이 phase transition 서론 ~ 조성의변화없는상변화 5. 상평형 : 순수물질 전이열역학 5. 안정성조건 G ng ng n G G 자발적변화 G < 0 G > G or 물질은가장낮은몰Gibbs 에너지를갖는상 가장안정한상 으로변화하려는경향 5. 상평형 : 순수물질 3 5. 압력에따른Gibbs

More information

4-Ç×°ø¿ìÁÖÀ̾߱â¨ç(30-39)

4-Ç×°ø¿ìÁÖÀ̾߱â¨ç(30-39) 항공우주 이야기 항공기에 숨어 있는 과학 및 비밀장치 항공기에는 비행 중에 발생하는 현상을 효율적으로 이용하기 위해 과 학이 스며들어 있다. 특별히 관심을 갖고 관찰하지 않으면 쉽게 발견할 수 없지만, 유심히 살펴보면 객실 창문에 아주 작은 구멍이 있고, 주 날 개를 보면 뒷전(trailing edge) 부분이 꺾어져 있다. 또 비행기 전체 형 상을 보면 수직꼬리날개가

More information

수리가-20일-최종-인쇄.hwp

수리가-20일-최종-인쇄.hwp 2012학년도 대학수학능력시험 수리영역에서는 와 관련된 연계율이 70%로 상향되었다. 수리영역에서는 수리 가형에서 EBS 수능교재 문제의 개념원리를 활용한 문 항이 1문항, 자료 상황을 활용한 문항이 10문항, 문항을 확대 또는 축소시켜 변형한 문항 이 10문항 출제되는 등 총 21문항이 연계되어 출제되었다. 또한 수리 나형에서 개념원리 활용한 문항이 7문항,

More information

슬라이드 1

슬라이드 1 장연립방정식을 풀기위한반복법. 선형시스템 : Guss-Sedel. 비선형시스템 . 선형시스템 : Guss-Sedel (/0) 반복법은초기근을가정한후에더좋은근의값을추정하는체계적인절차를이용한다. G-S 방법은선형대수방정식을푸는반복법중에서 가장보편적으로사용되는방법이다. 개의방정식에서 인 ( 대각원소들이모두 0 이아닌 ) 경우를다루자. j j b j b j j j

More information

TOPOLOGY-WEEK 6 & 7 KI-HEON YUN 1. Quotient space( 상공간 ) X 가위상공간이고 Y 가집합이며 f : X Y 가전사함수일때, X 의위상을사용하여 Y 에위상을정의할수있는방법은? Definition 1.1. X 가위상공간, f : X

TOPOLOGY-WEEK 6 & 7 KI-HEON YUN 1. Quotient space( 상공간 ) X 가위상공간이고 Y 가집합이며 f : X Y 가전사함수일때, X 의위상을사용하여 Y 에위상을정의할수있는방법은? Definition 1.1. X 가위상공간, f : X TOPOLOGY-WEEK 6 & 7 KI-HEON YUN 1. Quotient space( 상공간 ) X 가위상공간이고 Y 가집합이며 f : X Y 가전사함수일때, X 의위상을사용하여 Y 에위상을정의할수있는방법은? Definition 1.1. X 가위상공간, f : X Y 가전사함수일때, T Y = {U Y f 1 (U) is open set in X} 로정의하면

More information

2

2 rev 2004/1/12 KAIST 2 6 7 1 13 11 13 111 13 112 18 113 19 114 21 12 24 121 24 122 26 13 28 131 28 132 30 133 (recurrence) 34 134 35 4 2 39 21 39 211 39 212 40 22 42 221, 42 222 43 223, 45 224 46 225, 48

More information

기초 해석학 강의 노트

기초 해석학 강의 노트 기초해석학강의노트 Sooji Shin soojishin@live.com 1. 순서체의성질 집합 에대하여, 로부터 로의함수를 에서의이항연산이라고부른다. 즉이항연산이란두값의연산결과를한값에대응시키는함수이다. 정의 1.1 집합 에서의이항연산 이세조건 G1., G2., G3. 를모두만족시킬때, 집합 를연산 에대한군 (group) 이라고부른다. 이때군 를집합 와연산 를묶어

More information

(Microsoft PowerPoint - Ch21_NumAnalysis.ppt [\310\243\310\257 \270\360\265\345])

(Microsoft PowerPoint - Ch21_NumAnalysis.ppt [\310\243\310\257 \270\360\265\345]) 수치해석 161009 Ch21. Numerical Differentiation 21.1 소개및배경 (1/2) 미분 도함수 : 독립변수에대한종속변수의변화율 y = x f ( xi + x) f ( xi ) x dy dx f ( xi + x) f ( xi ) = lim = y = f ( xi ) x 0 x 차분근사 도함수 1 차도함수 : 곡선의한점에서접선의구배 21.1

More information

mathna_hsj.hwp

mathna_hsj.hwp 2008 학년도 6 월모의평가 ( 수리영역 - 가형 ) 정답및해설 1. 4 4 4. 2. 로놓으면 ᄀ - ᄂ 양변을제곱하면 3. 5 따라서 방정식ᄀ의근은이다. 일때 ( 분모 ) ( 분자 ) 이어야한다. 따라서 따라서 두식ᄀ ᄂ을동시에만족하는실수의값은구하는합은 ( 준식 ) 5 5. 는최고차항의계수가 1인삼차함수 로놓으면 - 1 - 따라서 ㄷ. 3 < 다른풀이

More information

슬라이드 1

슬라이드 1 . Fourier Series, Itegrl, d Trsorms Bog-Kee ee Chom Ntiol Uiversity. Fourier Series 주기함수 (periodi utio) 함수 (), 모든실수 에대하여정의주기 (period) 어떤양수 p가존재하여, 모든 에대하여 ( + p)=() 주기함수 (periodi utio) 예. si, ( 주기 π) 주기함수가아닌예.,,,

More information

슬라이드 1

슬라이드 1 1 장수치미분 1.1 소개및배경 1. 고정확도미분공식 1.3 Richardson 외삽법 1.4 부등간격의미분 1.5 오차가있는데이터의도함수와적분 1.6 MATLAB 을이용한수치미분 1.1 소개및배경 (1/4) 미분이란무엇인가? 도함수 : 독립변수에대한종속변수의변화율 y f( xi + x) f( xi) dy f( x = i + x) f( xi) = lim =

More information

마지막 변경일 2018년 5월 7일 ** 이항분포와 정규분포의 관계 ** Geogebra와 수학의 시각화 책의 3.2소절 내용임. 가장 최근 파일은 링크를 누르면 받아 보실 수 있습니다.

마지막 변경일 2018년 5월 7일 ** 이항분포와 정규분포의 관계 ** Geogebra와 수학의 시각화 책의 3.2소절 내용임.   가장 최근 파일은 링크를 누르면 받아 보실 수 있습니다. 마지막 변경일 2018년 5월 7일 ** 이항분포와 정규분포의 관계 ** Geogebra와 수학의 시각화 책의 3.2소절 내용임. http://min7014.iptime.org/math/2017063002.htm 가장 최근 파일은 링크를 누르면 받아 보실 수 있습니다. https://goo.gl/edxsm7 http://min7014.iptime.org/math/2018010602.pdf

More information

5. 정적분 의값과반지름의길이가 인원의넓 이가같을때, 의값은? 7. 곡선 ln 와 축및 축으로둘러싸인도형의넓이 가 일때, 상수 의값은? ( 단, ) 에서정의된함수 의 그래프가오른쪽그림과같을때, 정적분 의값을구하면? 8. 함수 의

5. 정적분 의값과반지름의길이가 인원의넓 이가같을때, 의값은? 7. 곡선 ln 와 축및 축으로둘러싸인도형의넓이 가 일때, 상수 의값은? ( 단, ) 에서정의된함수 의 그래프가오른쪽그림과같을때, 정적분 의값을구하면? 8. 함수 의 1. lim sin 의값은? 3. 함수 cos cos ( ) 는 에서극솟값 를갖는다. 이때 의값은? 1 2 3 1 2 3 4 5 4 5 2. 아래쪽그림과같이중심이 C 이고반지름의길이가 인원이있다. 직선 가원점 O 를지나고기울기가양수인직선 과만나는점을 P 축과만나는점을 Q 라하고, 직선 이원과만나는원점이아닌점을 R 라하자. 직선 이 축의양의방향과이루는각의크기를

More information

장연립방정식을풀기위한반복법 12.1 선형시스템 : Gauss-Seidel 12.2 비선형시스템 12.1 선형시스템 : Gauss-Seidel (1/10) 반복법은초기근을가정한후에더좋은근의값을추정하는체계적인절차를이용한다. G-S 방법은선형대수방정

장연립방정식을풀기위한반복법 12.1 선형시스템 : Gauss-Seidel 12.2 비선형시스템 12.1 선형시스템 : Gauss-Seidel (1/10) 반복법은초기근을가정한후에더좋은근의값을추정하는체계적인절차를이용한다. G-S 방법은선형대수방정 . 선형시스템 : GussSedel. 비선형시스템. 선형시스템 : GussSedel (/0) 반복법은초기근을가정한후에더좋은근의값을추정하는체계적인절차를이용한다. GS 방법은선형대수방정식을푸는반복법중에서 가장보편적으로사용되는방법이다. 개의방정식에서 인 ( 대각원소들이모두 0 이아닌 ) 경우를다루자. j j b j j b j j 여기서 j b j j j 현재반복단계

More information

Microsoft PowerPoint - ch02-1.ppt

Microsoft PowerPoint - ch02-1.ppt 2. Coodinte Sstems nd Tnsfomtion 20 20 2.2 Ctesin Coodintes (,, ) () (b) Figue 1.1 () Unit vectos,, nd, (b) components of long,, nd. 직각좌표계에서각변수 (,, ) 들의범위 < < < < < < (2.1) 직각좌표계에서임의의벡터 는,, 가그림 1.1 에서와같이,,

More information

슬라이드 1

슬라이드 1 16 장 Fourier 해석 16.1 사인함수를이용한곡선접합 16.2 연속 Fourier 급수 16.3 주파수영역과시간영역 16.4 Fourier 적분과변환 16.5 이산 Fourier 변환 (DFT) 16.6 파워스펙트럼 16.1 사인함수를이용한곡선접합 (1/5) 주기가 T 인주기함수 f() t = f( t+ T) 주기운동의가장기본 : 원운동 ( 코사인,

More information